ltd compilation as of september 8

372
PartI 1.VALIAOv. REPUBLIC GRNo. 170757, 28November 2011 PERALTA,J.: FACTS: PetitionersValiao filed forthe registration their parcel of land (Lot No. 2372) whichthey acquired fromtheir uncle in 1947. The Solicitor General and privateoppositors, Zafra andYusay, opposed said registration on the followinggrounds: (1) the land appliedfor has not been declared alienable anddisposable; (2) there was no proof thatthe applicants had in open, continuous,exclusive and notorious possession of theland in question since June 12, 1945or prior thereto. In the petitioners’defense, they alleged that the possessionof their uncle since 1916 until 1966had been open, continuous anduninterrupted; thus, converting the said land intoa private land. Issue:Whether or not Lot No. 2372 is an alienable land of the PublicDomain under theRegalian Doctrine. HELD: No,Lot No. 2372 isinalienable.

Upload: whatrich

Post on 09-Dec-2015

371 views

Category:

Documents


44 download

DESCRIPTION

case digest

TRANSCRIPT

PartI

 

1.VALIAOv. REPUBLIC

GRNo. 170757, 28November 2011

PERALTA,J.:

FACTS:

               PetitionersValiao filed forthe registration their parcel of land (Lot No. 2372) whichthey acquired fromtheir uncle in 1947. The Solicitor General and privateoppositors, Zafra andYusay, opposed said registration on the followinggrounds: (1) the land appliedfor has not been declared alienable anddisposable; (2) there was no proof thatthe applicants had in open, continuous,exclusive and notorious possession of theland in question since June 12, 1945or prior thereto. In the petitioners’defense, they alleged that the possessionof their uncle since 1916 until 1966had been open, continuous anduninterrupted; thus, converting the said land intoa private land.

Issue:Whether or not Lot No. 2372 is an alienable land of the PublicDomain under theRegalian Doctrine.

HELD:

               No,Lot No. 2372 isinalienable.

               Underthe Regalian doctrine,all lands of the public domain belong to the State. Alllands not appearing tobe clearly within private ownership are presumed tobelong to the State.Accordingly, public lands not shown to have beenreclassified or released asalienable agricultural land or alienated to a privateperson by the State,remain part of the inalienable public domain.

               Propertyof the public domainis beyond the commerce of man and not susceptible ofprivate appropriation andacquisitive prescription. Occupation thereof in theconcept of owner no matterhow long cannot ripen into ownership and

beregistered as a title. To prove thatthe <a></a>landsubject of an application for registration isalienable, the applicant mustestablish the existence of a positive act of thegovernment, such as apresidential proclamation or an executive order. Theapplicant may also securea certification from the government that the<a></a>landclaimed to have been possessed for the required numberof y<a>earsis alienable and disposable.</a>

               Inthis case, no such evidencewas offered by the petitioners to show that the <a></a>landinquestion has been classified as alienable and disposable<a></a>landof the <a></a>public domain. In the absenceofincontrovertible evidence to prove that the subject property isalreadyclassified as alienable and disposable, Lot 2372 is still an inalienablepublicdomain.

2. Secretary of the DENR vs. YapG.R. 167707 October 8, 2008Sacay et. al and Secretary of DENRG.R. 173775 Reyes, R.T., J.:Facts: There are two consolidated cases in this petition.G.R. 167707On November 10, 1978, then President Ferdinand Marcos issued Proclamation No. 1801 declaring Boracay Island, among other islands, caves and peninsulas in the Philippines, as tourist zones and marine reserves under the administration of the Philippine Tourism Authority (PTA). President Marcos later approved the issuance of PTA Circular 3-82 dated September 3, 1982, to implement Proclamation No. 1801. Claiming that Proclamation No. 1801 and PTA Circular No 3-82 precluded them from filing an application for judicial confirmation of imperfect title or survey of land for titling purposes, respondents-claimants Mayor Jose S. Yap, Jr., Libertad Talapian, Mila Y. Sumndad, and Aniceto Yap filed a petition for declaratory relief with the RTC in Kalibo, Aklan.In their petition, respondents-claimants alleged that Proclamation No. 1801 and PTA Circular No. 3-82 raised doubts on their right to secure titles over their occupied lands.They declared that they themselves, or through their predecessors-in-interest, had been in open, continuous, exclusive, and notorious possession and occupation in Boracay since June 12, 1945, or earlier since time immemorial. On July 14, 1999, the RTC rendered a decision in favor of respondents-claimants, The RTC upheld respondents-claimants right to have their occupied lands titled in their name. It ruled that neither Proclamation No. 1801 nor PTA Circular No. 3-82 mentioned that lands in Boracay were inalienable or could not be the subject of disposition. The Circular itself recognized private ownership of lands. The trial court cited Sections 87 and 53 of the Public Land Act as basis for acknowledging private ownership of lands in Boracay and that only those forested areas in public lands were declared as part of the forest reserve. On December 9, 2004, the appellate court affirmed in toto the RTC decision, The CA held that respondents-claimants could not be prejudiced

by a declaration that the lands they occupied since time immemorial were part of a forest reserve.G.R. 173775On May 22, 2006, during the pendency of G.R. No. 167707, President Gloria Macapagal-Arroyo issued Proclamation No. 1064 classifying Boracay Island into four hundred (400) hectares of reserved forest land (protection purposes) and six hundred twenty-eight and 96/100 (628.96) hectares of agricultural land (alienable and disposable). The Proclamation likewise provided for a fifteen-meter buffer zone on each side of the centerline of roads and trails, reserved for right-of-way and which shall form part of the area reserved for forest land protection purposes. Petitioners-claimants contended that there is no need for a proclamation reclassifying Boracay into agricultural land. Being classified as neither mineral nor timber land, the island is deemed agricultural pursuant to the Philippine Bill of 1902 and Act No. 926, known as the first Public Land Act. Thus, their possession in the concept of owner for the required period entitled them to judicial confirmation of imperfect title. Issue: Whether private claimants (respondents-claimants in G.R. No. 167707 and petitioners-claimants in G.R. No. 173775) have a right to secure titles over their occupied portions in Boracay.Held:NO. The Regalian Doctrine dictates that all lands of the public domain belong to the State, that the State is the source of any asserted right to ownership of land and charged with the conservation of such patrimony. The doctrine has been consistently adopted under the 1935, 1973, and 1987 Constitutions. All lands not otherwise appearing to be clearly within private ownership are presumed to belong to the State. Thus, all lands that have not been acquired from the government, either by purchase or by grant, belong to the State as part of the inalienable public domain. Necessarily, it is up to the State to determine if lands of the public domain will be disposed of for private ownership. The government, as the agent of the state, is possessed of the plenary power as the persona in law to determine who shall be the favored recipients of public lands, as well as under what terms they may be granted such privilege, not excluding the placing of obstacles in the way of their exercise of what otherwise would be ordinary acts of ownership.A positive act declaring land as alienable and disposable is required. In keeping with the presumption of State ownership, the Court has time and again emphasized that there must be a positive act of the government, such as an official proclamation, declassifying inalienable public land into disposable land for agricultural or other purposes. In fact, Section 8 of CA No. 141 limits alienable or disposable lands only to those lands which have been officially delimited and classified. In the case at bar, no such proclamation, executive order, administrative action, report, statute, or certification was presented to the Court. The records are bereft of evidence showing that, prior to 2006, the portions of Boracay occupied by private claimants were subject of a government proclamation that the land is alienable and disposable. Absent such well-nigh incontrovertible evidence, the Court cannot accept the submission that lands occupied by private claimants were already open to disposition before 2006.Matters of land classification or reclassification cannot be assumed.

3(Gina)

FERNANDAARBIAS - versus - THE REPUBLIC OF THE PHILIPPINES,

 G.R. No. 173808           September 17, 2008

 CHICO-NAZARIO, J

Facts:

On 12March 1993, Lourdes T. Jardeleza (Jardeleza) executed a Deed of Absolute Sale sellingto petitioner a parcel of unregistered land situated at Poblacion, Estancia,Iloilo,   for the sum of P33,000.00. According to theDeed, the subject property was residential and consisted of 600 square meters,more or less.

Three years thereafter, on 17 June 1996,petitioner filed with the RTC a verified Application for Registration of Titleover the subject property. She attached to her application the Tracing Clothwith Blue Print copies, the Deed of Absolute Sale involving the subjectproperty, the Surveyors Certification, the Technical Description of the land,and Declaration of Real Property in the name of petitioner and her spouse Jimmy.

 Thereafter,the respondent filed an Opposition to petitioners application for registrationof the subject property. In its Brief, respondent questioned  the failure of petitioner to prove thecontinuous, open, exclusive and notorious possession by theirpredecessor-in-interest.

On 26June 2000, the RTC granted the petitioners application for registration in thiswise:

As tothe issue that muniments of title and/or tax declarations and taxreceipts/payments do not constitute competent and sufficient evidence

ofownership, the same cannot hold through (sic) anymore it appearing from therecords that the muniments of titles as presented by the herein applicant arecoupled with open, adverse and continuous possession in the concept of anowner, hence, it can be given greater weight in support of the claim forownership. The [herein petitioner] is a private individual who is qualifiedunder the law being a purchaser in good faith and for value. The adverse, open,continuous and exclusive possession of the land in the concept of owner of the[petitioner] started as early as in 1992 when their predecessors in interestfrom Lourdes Jardeleza then to the herein [petitioner] without any disturbanceof their possession as well as claim of ownership. Hence, uninterruptedpossession and claim of ownership has ripen (sic) into an incontrovertibleproof in favor of the [petitioner].

It wason the issue of possession, however, that the Court of Appeals digressed fromthe ruling of the RTC. The appellate court found that other than petitionersown general statements and tax declarations, no other evidence was presented toprove her possession of the subject property for the period required by law.Likewise, petitioner failed to establish the classification of the subjectproperty as an alienable and disposable land of the public domain. The Court ofAppeals reversed and set aside the decision of the RTC.

 Issue:

whether ornot the petitioner has proved that the subject land is an alienable land.

Held:

No. Underthe Regalian doctrine, all lands of the public domain belong to the State, andthe State is the source of any asserted right to ownership of land and chargedwith the conservation of such patrimony. This same doctrine also states thatall lands not otherwise appearing to be clearly within private ownership arepresumed to belong to the State. Hence, the burden of proof in overcoming thepresumption of State ownership of lands of the public domain is on the personapplying for registration. The applicant must show that the land subject of theapplication is alienable or disposable.

Section14, paragraph 1 of Presidential Decree No. 1529 states the requirementsnecessary for a judicial confirmation of imperfect title to be issued. Theapplicant for registration under said statutory provision must specificallyprove:

1) possession of the subject land under a bona fide claim of ownershipfrom 12 June 1945 or earlier; and 2) the classification of the land as analienable and disposable land of the public domain.

In thecase at bar, petitioner miserably failed to discharge the burden of proofimposed on her by the law. The Deed of Sale did not state the duration of timeduring which the vendor (or her predecessors-in-interest) possessed the subjectproperty in the concept of an owner. Well-settled is the rule that taxdeclarations and receipts are not conclusive evidence of ownership or of theright to possess land when not supported by any other evidence. The Survey Planand Technical Description of the subject property submitted by petitionermerely plot the location, area and boundaries thereof.

For theoriginal registration of title, the applicant must overcome the presumptionthat the land sought to be rehgtered forms part of the public domain. Unlesspublic land is shown to have been reclassified or alienated to a private personby the State, it remains part of the inalienable public domain. Indeed,occupation thereof in the concept of owner, no matter how long, cannot ripeninto ownership and be registered as a title. To overcome such presumption,incontrovertible evidence must be shown by the applicant. Absent such evidence,the land sought to be registered remains inalienable.

4(Zhon)

Case Number I.A.d.Alcantara vs. DENR

G.R. No. 161881, July 31, 2008

Facts:Petitioner is a lessee under FLGLA No. 542 , issued by the DENR, of 923 hectares of public forest land (subject land) located in the vicinity of Sitio Lanton, Barrio Apopong, General Santos City, since 1983.

The subject land, is being claimed as the ancestral land of the indigenous B'laan and Maguindanao people, who maintain that they and their predecessors have been cultivating, possessing and occupying it since time immemorial.

On April 10, 1990, private respondents, representing the B'laan and Maguindanao tribes, filed a complaint against petitioner before the Commission on the Settlement of Land Problems (COSLAP) seeking the cancellation of FLGLA No. 542 and the reversion of the land to the indigenous communities.

In 1993, despite the pendency of the COSLAP case, and despite opposition from private respondents, petitioner was able to renew FLGLA No. 542 when it expired that year. The renewal given to petitioner was for another 25 years, or until December 31, 2018.

On October 29, 1997, Congress passed Republic Act No. 8371, or the Indigenous People's Rights Act (IPRA), which was intended to recognize andpromote all the rights of the country's Indigenous Cultural Communities/Indigenous Peoples (ICCs/IPs) within the framework of the Constitution.

On August 3, 1998, the COSLAP rendered its decision, recommending to DENR Sec., inter alia, cancellation of respondents renewed FLGLA No. 542 and placing in possession the subject land to the petitioners in order to start cultivation and plant crops for their food. COSLAP denied petitioner’s MR. Upon appeal, the CA affirmed said decision in toto.

Petitioner filed a petition for review on certiorari before SC, docketed as G.R. No. 145838. SC upheld the CA and the COSLAP, holding that a) COSLAP had jurisdiction to decide the case; b) FLGLA No. 542 was issued in violation of the law, and; c) the 923 hectares covered by FLGLA No. 542 were ancestral land of the private respondents.

On July 29, 2002, the COSLAP issued a writ of execution of its decision, ordering the DENR Sec. to implement the August 3, 1998 decision as affirmed by SC. DENR Sec. ordered the DENR Director of Region 12 toconduct a review and investigation of

FLGLA No. 542. The latter found violations by petitioner of the terms of the FLGLA. On August 15, 2002, Sec. Alvarez cancelled FLGLA No. 542. Petitioner’s MR was denied.

On November 26, 2002, Community Environment and Natural Resources Officer (CENRO) Andrew B. Patricio Jr. sent a letter to petitioner, advising him to vacate and remove all improvements in the area within 10 days from receipt of the letter.

On November 27, 2002, CENRO Patricio issued an Installation Order, which directed the immediate installation and occupation of the area by the private respondents indigenous communities.

Upon appeal, the CA dismissed petitioner’s appeal, holding that issue was already decided by SC in GR No. G.R. No. 145838. Hence, this instant petition via Rule 45.

Issue: w/n petitioner has residual rights over subject property until the expiration of FLGLA 542 (Dec. 31, 2018)

Held:No. Petitioner has no residual rights to speak of because such right presupposes an existing right. In the case at bar, FLGLA No. 542 which is supposed to be the source of petitioner’s right was already cancelled and held contrary to law.

Further, FLGLA 542, even if not cancelled, does not confer any right. FLGLA No. 542 is a mere license or privilege granted by the State to petitioner for the use or exploitation of natural resources and public lands over which the State has sovereign ownership under the Regalian Doctrine. Like timber or mining licenses, FLGA is a mere permit which, by executive action, can be revoked, rescinded, cancelled, amended or modified, whenever public welfare or public interest so requires. Thus, a privilege or license is not in the nature of a contract that enjoys protection under the due process and non-impairment clauses of the Constitution. In cases in which the license or privilege is in conflict with the people's welfare, the former must yield to the supremacy of the latter, as well as to the police power of the State.

5(May Ann)

6(Rhea B.)

REPUBLIC OF THE PHILIPPINES VS CANDY MAKER, INC.GR.NO. 163766, June 22, 2006Facts:On April, 29, 1999, Antonia, Eladia, and Felisa, all surnamed Cruz, executed a Deed of Absolute Sale in favor of Candy Maker, Inc. for a parcel of land located below the reglementary lake elevation of 12.50m, about 900 meters away the Laguna de Bay. Candy Maker, Inc. as applicant, filed an application with the MTC of Taytay, Rizal for registration of it’s alleged title over the lot.The CENRO of Antipolo City declared the land to fall within the alienable and disposable zone. On the other hand, the Land Registration Authority recommended the exclusion of lot no. 3138-B on the ground that it is a legal easement and intended for public use, hence, inalienable and indisposable. On July 2001, the Republic of thePhilippines, the LLDA filed its opposition which alleged that the lot subject of the application for registration may not be alienated and disposed since it is considered part of the Laguna Lake Bed, a public land within, its jurisdiction.Issue:Whether or not the property subject of the amended application is alienable and disposable property of the State, and if so, whether respondent adduced the requisite quantum of evidence to prove it’s ownership over the property.Ruling:The property subject of this application was alienable and disposable public agricultural land. However, respondent failed to prove that it possesses registrable title over the property. The statute of limitations with regard to public agricultural lands does not operate against the statute unless the occupant proves possession and occupation of the same after a claim ofownership for the required number of years to constitute a grant from the State. A mere casual cultivation of portions of the land by the claimant does not constitute sufficient basis for a claim of ownership, such possession is not exclusive and notorious as to give rise to presumptive grant from the state. In light of the foregoing, the petition of the Republic of the Philippines is granted.

7(Angel)

8. Cariño vs Insular Government, 41 Phil 935

Facts: Carino, an Igorot, applied for the registration of a certain land situated in the town of Baguio, Province of Benguet. He and his ancestors had held the land as owners for more than 50 years, which he inherited under Igorot customs. There was no document of title issued for the land when he applied for registration. The government contends that the land in question belonged to the state. Under the Spanish Law, all lands belonged to the Spanish Crown except those with permit private titles. Moreover, there is no prescription against the Crown.

Issue: Wether the land in question belonged to the Spanish Crown under the Regalian Doctrine? Held: No. Law and justice require that the applicant should be granted title to his land.

The United States Supreme Court, through Justice Holmes declared:

“It might perhaps, be proper and sufficient to say that when, as far as testimony or memory goes, the land has been held by individuals under a claim of private ownership, it will be presumed to have been held in the same way from before the Spanish conquest, and never to have been public land.”

There is an existence of native title to land, or ownership of land by Filipinos by virtue of possession under a claim of ownership since time immemorial and independent of any grant from the Spanish Crown, as an exception to the theory of jura regalia.

9. Cruzvs Secretary ofDENR

GR.No. 135385, Dec.6, 2000

FACTS:

               Petitioners Isagani CruzandCesar Europa filed a suit for prohibition and mandamus as citizensandtaxpayers, assailing the constitutionality of certain

provisions ofRepublicAct No. 8371, otherwise known as the Indigenous People’s Rights Act of1997(IPRA) and its implementing rules and regulations (IRR). The petitionersassailcertain provisions of the IPRA and its IRR on the ground that theseamount toan unlawful deprivation of the State’s ownership over lands of thepublicdomain as well as minerals and other natural resources therein, inviolation ofthe regalian doctrine embodied in section 2, Article XII of the Constitution.

ISSUE:

Dotheprovisions of IPRA contravene the Constitution?

HELD:

No,theprovisions of IPRA do not contravene the Constitution. Examining theIPRA,there is nothing in the law that grants to the ICCs/IPs ownership overthenatural resources within their ancestral domain. Ownership over thenaturalresources in the ancestral domains remains with the State and therightsgranted by the IPRA to the ICCs/IPs over the natural resources intheirancestral domains merely gives them, as owners and occupants of the landonwhich the resources are found, the right to the small scale utilizationofthese resources, and at the same time, a priority in their largescaledevelopment and exploitation.

Additionally,ancestrallands and ancestral domains are not part of the lands of the publicdomain. Theyare private lands and belong to the ICCs/IPs by native title,which is a conceptof private land title that existed irrespective of any royalgrant from theState. However, the right of ownership and possession by theICCs/IPs of theirancestral domains is a limited form of ownership and does notinclude the rightto alienate the same.

10(Lou)

Secretaryof DENR vs. Yap (568 SCRA 164) G.R. No. 167707 (8October 2008)

 

Facts:On November 10, 1978, then President Ferdinand Marcos issuedProclamation No. 1801 declaring Boracay Island, amongother islands, caves and peninsulas in the Philippines, as touristzones and marine reserves under the administration of the PhilippineTourism Authority (PTA). President Marcos later approved the issuanceof PTA Circular 3-82 dated September 3, 1982, toimplement Proclamation No. 1801.

On July14, 1999, the RTC rendered a decision in favor ofrespondents-claimants.

The RTC upheldrespondents-claimants right to have their occupied lands titled in their name.The Office of the Solicitor General moved for reconsideration but itsmotion was denied. The Republic then appealed to the CA. On December9, 2004, the appellate court affirmed in toto the RTC decision.The CA held that respondents-claimants could not be prejudiced by a declarationthat the lands they occupied since time immemorial were part of a forestreserve. Again, the OSG sought reconsideration but it was similarlydenied. Hence, the present petition under Rule 45.

G.R.No. 173775

On May22, 2006, during the pendency of G.R. No. 167707, President GloriaMacapagal-Arroyo issued Proclamation No. 1064[26] classifying BoracayIsland into four hundred (400) hectares of reserved forest land (protectionpurposes) and six hundred twenty-eight and 96/100 (628.96) hectares ofagricultural land (alienable and disposable). The Proclamation likewiseprovided for a fifteen-meter buffer zone on each side of the center line ofroads and trails, reserved for right-of-way and which shall form part of thearea reserved for forest land protection purposes.

On November21, 2006, this Court ordered the consolidation of the two petitions (G.R.No. 167707 and G.R. no. 173775) as they principally involve thesame issues on the land classification of Boracay Island.

Respondents-claimants allegedthat they themselves, or through their predecessors-in-interest, had been inopen, continuous, exclusive, and notorious possession and occupation in Boracaysince June 12, 1945, or earlier since time immemorial. They declared theirlands for tax purposes and paid realty taxes on them. Also they alleged thatthe proclamation and its implementing circulars did not place Boracay beyondthe commerce of men and was classified as a tourist zone, it was susceptible ofprivate ownership. Under Section 48(b) of Commonwealth Act (CA) No. 141,otherwise known as the Public Land Act, they had the right to have the lotsregistered in their names through judicial confirmation of imperfect titles.

TheOffice of the Solicitor General (OSG), opposed the petition:Because Boracay Island was an unclassified land ofthe public domain. It formed part of the mass of lands classified aspublic forest, which was not available for disposition pursuant to Section 3(a)of Presidential Decree (PD) No. 705 or the Revised Forestry Code, asamended. The OSG maintained that respondents-claimants relianceon PD No. 1801 and PTA Circular No. 3-82 was misplaced. Theirright to judicial confirmation of title was governed by CA No. 141 and PD No.705. Since Boracay Island had not been classified asalienable and disposable, whatever possession they had cannot ripen intoownership. 

Issue/s:W/N Proclamation No. 1801 and PTA Circular No. 3-82converted Boracay into an agricultural land.

Held:No.

 

Privateclaimants cannot rely on Proclamation No. 1801 as basis for judicialconfirmation of imperfect title. The proclamation did not convert Boracayinto an agricultural land. ProclamationNo. 1801 or PTA Circular No. 3-82 did not convert the whole ofBoracay into an agricultural land. There is nothing in the law or theCircular which made Boracay Island an agricultural land. Thereference in Circular No. 3-82 to private lands and areas declared as alienableand disposable does not by itself classify the entire island asagricultural.

Therefore,Proclamation No. 1801 cannot be deemed the positive act needed toclassify Boracay Island as alienable and disposable land. IfPresident Marcos intended to classify the island as alienable and disposable orforest, or both, he

would have identified the specific limits of each, asPresident Arroyo did in Proclamation No. 1064. This was not done inProclamation No. 1801.

TheWhereas clauses of Proclamation No. 1801 also explain the rationale behind thedeclaration of Boracay Island, together with other islands, caves andpeninsulas in the Philippines, as a tourist zone and marine reserve to beadministered by the PTA to ensure the concentrated efforts of thepublic and private sectors in the development of the areas tourism potentialwith due regard for ecological balance in the marine environment. Simplyput, the proclamation is aimed at administering the islands for tourismand ecological purposes. It does not address the areas alienability.

Privateclaimants continued possession under Act No. 926 does not create a presumptionthat the land is alienable. Privateclaimants also contend that their continued possession of portionsof Boracay Island for the requisite period of ten (10) yearsunder Act No. 926 ipso facto converted the island into privateownership. Hence, they may apply for a title in their name.

Exceptfor lands already covered by existing titles, Boracay was an unclassified landof the public domain prior to Proclamation No. 1064. Such unclassifiedlands are considered public forest under PD No. 705. TheDENR and the National Mapping and Resource InformationAuthority certify that Boracay Island is an unclassifiedland of the public domain.

 

WHEREFORE,judgment is rendered as follows:

1.The petition for certiorari in G.R. No. 167707 is GRANTED andthe Court of Appeals Decision in CA-G.R. CV No. 71118 REVERSED AND SETASIDE.

2.The petition for certiorari in G.R. No. 173775 is DISMISSED forlack of merit.

SOORDERED.

 

Syllabus: Natural Resources; Public Lands; Regalian Doctrine; Words and Phrases;The Regalian Doctrine dictates that all lands of the public domain belong tothe State, that the State is the source of any asserted right to ownership ofland and charged with the conservation of such patrimony, a doctrineconsistently adopted under the 1935, 1973 and 1987 Constitution; Prior toProclamation No.1064 of May 22, 2006, Boracay Island had never been expresslyand administratively classfified under any of the grand divisions of land.Boracay was an unclassified land of the public Domain. --- The1935 Constitution classified lands of the public domain into agricultural,forest or timber. Meanwhile, the 1973 Constitution provided the followingdivisions: agricultural, industrial or commercial, residential, resettlement,mineral, timber or forest and grazing lands and such other classes as may beprovided by law, giving the government great leeway for classification. Thenthe 1987 Constitution reverted to the 1935 Constitution classification with oneaddition: national parks. Of these, only agricultural lands may be alienated.Prior to Proclamation No. 1064 of May 22, 2006, Boracay Island had never beenexpressly and administratively classified under any of these grand divisions.Boracay was an unclassified land of the public domain. The Regalian Doctrinedictates that all lands of the public domain belong to the State, That theState is the source of any asserted right to ownership of land and charged withthe conservation of such patrimony. The doctrine has been consistently adoptedunder the 1935, 1973, and 1987 Constitutions.

TheRegalian doctrine was first introduced in the Philippines through the Laws ofthe Indies and the Royal Cedulas, Which laid the foundation that "alllands that were not acquired from the Government, either by purchase or bygrant, belong to the public domain." -- Ourpresent land law traces its roots to the Regalian Doctrine. Upon the Spanishconquest of the Philippines, ownership of all lands, territories andpossessions in the Philippines passed to the Spanish Crown. The Regaliandoctrine was first introduced in the Philippines through the laws of the Indiesand the Royal Cedulas, which laid the foundation that "all lands that werenot acquired from the Government, either by purchase or by grant, belong to thepublic domain."

NOTES:

 Under the Regalian doctrine, all lands not otherwise appearing to be clearlywithin private ownership are presumed to belong to the State -- unless publicland is shown to have been reclassified as alienable or disposable to a privateperson by the State, it remains part of the inalienable public domain. (Republicvs. Jacob, 495 SCRA 529(2006))

 Underthe Regalian doctrine, all lands not otherwise appearing to be clearly withinprivate ownership are presumed to belong to the State -- applicants forconfirmation of imperfect title must therefore, prove the following: (a) thatthe lands of the public domain; and, (b) that they have been in open,continuous, exclusive, and notorious possession and occupation of the sameunder a bona fide claim of ownership either since time immemorial or since June12,1945 lies in the presumption that the land applied for pertains to theState, and that the occupants or possessor claim an interest thereon only byvirtue of their imperfect title as continuous open and notorious possession.(Republicvs. Candy Maker, Inc., 492 SCRA 272 (2006))

 Historyof Public Land Dispostition

TheRegalian principle traces its roots:

Uponthe Spanish conquest of the Philippines, ownership of all lands,territories and possessions in thePhilippines passed to the Spanish Crown.The Regalian doctrine was first introduced in the Philippines throughthe Laws of the Indies and the Royal Cedulas, which laid thefoundation that all lands that were not acquired from the Government, either bypurchase or by grant, belong to the public domain.

The Lawsof the Indies was followed by the Ley Hipotecaria orthe Mortgage Law of 1893. The Spanish Mortgage Law providedfor the systematic registration of titles and deeds as well as possessoryclaims.

TheRoyal Decree of 1894 or the Maura Law partly amended the SpanishMortgage Law and the Laws of the Indies. It established possessoryinformation as the method of legalizing possession of vacant Crown land, undercertain conditions which were set forth in said decree.

UnderSection 393 of the Maura Law, an informacion posesoria orpossessory information title, when duly inscribed in the Registry ofProperty, is converted into a title of ownership only after the lapse of twenty(20) years of uninterrupted possession which must be actual, public, andadverse, from the date of its inscription.

However, possessory informationtitle had to be perfected one year after the promulgation of the Maura Law, oruntil April 17, 1895. Otherwise, the lands would revert to the State.

Privateownership of land could only be founded on royal concessions which tookvarious forms, namely: (1) titulo real or royal grant;(2) concesion especial or special grant; (3) composicioncon el estado or adjustment title; (4) titulo de compra ortitle by purchase; and (5) informacion posesoria or possessoryinformation title.

AmericanRule:

The Philippine Bill of 1902: lands of the public domain in the Philippine Islands were classified into three (3) grand divisions, to wit: agricultural, mineral, and timber or forest lands. The act provided for, among others, the disposal of mineral lands by means of absolute grant (freehold system) and by lease (leasehold system). It also provided the definition by exclusion of agricultural public lands.

On February 1, 1903, the Philippine Legislature passed Act No. 496, otherwise known as the Land Registration Act. The act established a system of registration by which recorded title becomes absolute, indefeasible, and imprescriptible. This is known as the Torrens system.

On October 7, 1903, the Philippine Commission passed Act No. 926, which was the first Public Land Act. The Act introduced the homestead system and made provisions for judicial and administrative confirmation of imperfect titles and for the sale or lease of public lands. It permitted corporations regardless of the nationality of persons owning the controlling stock to lease or purchase lands of the public domain. Under the Act, open, continuous, exclusive, and notorious possession and occupation of agricultural lands for the next ten (10) years preceding July 26, 1904 was sufficient for judicial confirmation of imperfect title.

On November 29, 1919, Act No. 926 was superseded by Act No. 2874, otherwise known as the second Public Land Act. This new, more comprehensive law limited the exploitation of agricultural lands to Filipinos

and Americans and citizens of other countries which gave Filipinos the same privileges. For judicial confirmation of title, possession and occupation en concepto dueo since time immemorial, or since July 26, 1894, was required.

After the passage of the 1935 Constitution, CA No. 141 amended Act No. 2874 on December 1, 1936. To this day, CA No. 141, as amended, remains as the existing general law governing the classification and disposition of lands of the public domain other than timber and mineral lands, and privately owned lands which reverted to the State.

Section 48(b) of CA No. 141 retained the requirement under Act No. 2874 of possession and occupation of lands of the public domain since time immemorial or since July 26, 1894. However, this provision was superseded by Republic Act (RA) No. 1942, which provided for a simple thirty-year prescriptive period for judicial confirmation of imperfect title. The provision was last amended by PD No. 1073, which now provides for possession and occupation of the land applied for since June 12, 1945, or earlier.

The issuance of PD No. 892 on February 16, 1976 discontinued the use of Spanish titles as evidence in land registration proceedings. Under the decree, all holders of Spanish titles or grants should apply for registration of their lands under Act No. 496 within six (6) months from the effectivity of the decree on February 16, 1976. Thereafter, the recording of all unregistered lands shall be governed by Section 194 of the Revised Administrative Code, as amended by Act No. 3344.

On June 11, 1978, Act No. 496 was amended and updated by PD No. 1529, known as the Property Registration Decree. It was enacted to codify the various laws relative to registration of property. It governs registration of lands under the Torrens system as well as unregistered lands, including chattel mortgages.

11(Jess)

CENTRAL MINDANAO UNIVERSITY, G.R. No. 184869

Represented by Officer-In-Charge

Dr. Rodrigo L. Malunhao,

Petitioner

Vs

THE HONORABLE EXECUTIVE SECRETARY, THE HONORABLE SECRETARY OF THE DEPARTMENT OF ENVIRONMENT AND NATURAL RESOURCES, THE CHAIRPERSON AND COMMISSIONERS OF THE NATIONAL COMMISSION ON INDIGENOUS PEOPLES, and THE LEAD CONVENOR OF THE NATIONAL ANTI-POVERTY COMMISSION,

Respondents.

Facts

Central Mindanao University is a chartered educational institution owned and run by the State. In 1958, the Presidential Proclamation 476, reserving 3401 hectares of lands of the public domain in Musuan , Bukidnon, as school site for CMU. CMU obtained title in its name over 3,080 hectares of those lands under OCT. Meanwhile, the government distributed the remaining untitled lands to several tribes.

Forty five years later, President GMA issued PP 310 that takes 670 hectares from the CMUs registered lands for distribution belonging to tribes in Barangay Musuan, Maramag, Bukidnon

On April 3, 2003, however, CMU filed a petition for prohibition against respondents Executive Secretary, Secretary of the Department of Environment and Natural Resources, Chairperson and Commissioner of the National Commission on Indigenous Peoples (NCIP), and Lead Convenor of the National Anti-Poverty Commission (collectively, NCIP, et al) before the Regional Trial Court (RTC) of Malaybalay City (Branch 9), seeking to stop the implementation of Presidential Proclamation 310 and have it declared unconstitutional.

The NCIP, et al moved to dismiss the case on the ground of lack of jurisdiction of the Malaybalay RTC over the action, pointing out that since the act sought to be enjoined

relates to an official act of the Executive Department done in Manila, jurisdiction lies with the Manila RTC. The Malaybalay RTC denied the motion, however, and proceeded to hear CMUs application for preliminary injunction. Meanwhile, respondents NCIP, et al moved for partial reconsideration of the RTCs order denying their motion to dismiss.

On October 27, 2003, after hearing the preliminary injunction incident, the RTC issued a resolution granting NCIP, et als motion for partial reconsideration and dismissed CMUs action for lack of jurisdiction. Still, the RTC ruled that Presidential Proclamation 310 was constitutional, being a valid State act. The RTC said that the ultimate owner of the lands is the State and that CMU merely held the same in its behalf. CMU filed a motion for reconsideration of the resolution but the RTC denied the same on April 19, 2004.This prompted CMU to appeal the RTCs dismissal order to the Court of Appeals (CA) Mindanao Station.[2]

In a March 14, 2008 decision,[4] the CA dismissed CMUs appeal for lack of jurisdiction, ruling that CMUs recourse should have been a petition for review on certiorarifiled directly with this Court, because it raised pure questions lawbearing mainly on the constitutionality of Presidential Proclamation 310. The CA added that whether the trial court can decide the merits of the case based solely on the hearings of the motion to dismiss and the application for injunction is also a pure question of law.

Issue

Whether or not Presidential Proclamation 310 can distribute the land of CMU given by PP 476 in 1958

Held: No

It did not matter that it was President Arroyo who, in this case, attempted by proclamation to appropriate the lands for distribution to indigenous peoples and cultural communities. As already stated, the lands by their character have become inalienable from the moment President Garcia dedicated them for CMUs use in scientific and technological research in the field of agriculture. They have ceased to be alienable public lands.

Besides, when Congress enacted the Indigenous Peoples Rights Act (IPRA) or Republic Act 8371in 1997, it provided in Section 56 that property rights within the ancestral domains already existing and/or vested upon its effectivity shall be recognized and respected. In this case, ownership over the subject lands had been vested in CMU as early as 1958. Consequently, transferring the lands in 2003 to the indigenous peoples around the area is not in accord with the IPRA.

12(Diane)

HEIRSOF MARIO MALABANAN, (Represented by Sally A. Malabanan), Petitioners,vs. REPUBLIC OF THE PHILIPPINES, Respondent.

G.R.No. 179987 April 29, 2009

Thepetition, while unremarkable as to the facts, was accepted by the Court enbanc in order to provide definitive clarity to the applicability and scopeof original registration proceedings under Sections 14(1) and 14(2) of theProperty Registration Decree

(PDNo. 1529)

FACTS:

On20 February 1998, Mario Malabanan filed an application for land registrationcovering a parcel of land identified as Lot9864-A, Cad-452-D, Silang Cadastre,situated in Silang Cavite. Malabanan claimed that he had purchased the propertyfrom Eduardo Velazco, and that he and his predecessors-in-interest had been in open,notorious, and continuous adverse and peaceful possession of the land for morethan thirty (30) years. Malabanan and Aristedes Velazco, testified atthe hearing. Velazco testified that the property was originally belonged to a22 hectare property owned by his great-grandfather, Lino Velazco.

TheRepublic of the Philippines likewise did not present any evidence to controvertthe application.

Malabananpresented evidence during trial a Certification dated 11 June 2001, issued bythe CENRO-DENR, which stated that the subject property was “verified to bewithin the Alienable or Disposable land per Land Classification Map No.3013 established under Project No. 20-A and approved as such under FAO 4-1656 onMarch 15, 1982.”

On3 December 2002, the RTC rendered judgment in favor of Malabanan. The Republicappealed to the Court of Appeals. CA reversed the decision and dismissed theapplication of Malabanan. Malabanan died while the case was pending with theCA; it was his heirs who appealed the decision of the appellate court.

ISSUE:

Whetheror not Malabanan has acquired ownership over the subject property under Section48(b) of the Public Land Act.

HELD:

No.Evidence of petitioners is insufficient to establish that Malabanan hasacquired ownership over the subject property under Section 48(b) of the PublicLand Act. There is no substantive evidence to establish that Malabanan orpetitioners as his predecessors-in-interest have been in possession of theproperty since 12 June 1945 or earlier.

Neithercan petitioners properly invoke Section 14(2) as basis for registration. Whilethe subject property was declared as alienable or disposable in 1982, there isno competent evidence that is no longer intended for public use service or forthe development of the national evidence, conformably with Article 422 of theCivil Code. The classification of the subject property as alienable and disposableland of the public domain does not change its status as property of the publicdominion under Article 420(2) of the Civil Code. Thus, it is insusceptible toacquisition by prescription.

DOCTRINES:

CommonwealthAct No. 141 (Public Land Act) governed the classification and disposition oflands of the public domain. The President is authorized, from time to time, toclassify the lands of the public domain into alienable and disposable, timber,or mineral lands. Alienable and disposable lands of the public domain arefurther classified according to their uses into (a) agricultural; (b)residential, commercial, industrial, or for similar productive purposes; (c)educational, charitable, or other similar purposes; or (d) reservations fortown sites and for public and quasi-public uses.

Section11 of the Public Land Act acknowledges that public lands suitable foragricultural purposes may be disposed of “by confirmation of imperfect orincomplete titles” through “judicial legalization.”

Section48(b) of the Public Land Act, as amended by P.D. No. 1073, supplies the detailsand unmistakably grants that right, subject to the requisites stated therein:

Sec.48. The following described citizens of the Philippines, occupying lands of thepublic domain or claiming to own any such land or an interest therein, butwhose titles have not been perfected or completed, may apply to the Court ofFirst Instance of the province where the land is located for confirmation oftheir claims and the issuance of a certificate of title therefor, under theLand Registration Act, to wit:

(b)Those who by themselves or through their predecessors in interest have been in open,continuous, exclusive, and notorious possession and occupation of alienable anddisposable lands of the public domain, under a bona fide claim of acquisitionof ownership, since June 12, 1945, or earlier, immediately preceding the filingof the application for confirmation of title except when prevented by waror force majeure. These shall be conclusively presumed to have performed allthe conditions essential to a Government grant and shall be entitled to acertificate of title under the provisions of this chapter.

Twosignificant amendments were introduced by P.D. No. 1073. First, the term“agricultural lands” was changed to “alienable and disposable lands of thepublic domain.”

UnderSection 9 of the Public Land Act, “agricultural lands” are a mere subset of“lands of the public domain alienable or open to disposition.” Evidently,

alienableand disposable lands of the public domain are a larger class than only“agricultural lands.”

Second,the length of the requisite possession was changed from possession for “thirty(30) years immediately preceding the filing of the application” to possession“since June 12, 1945 or earlier.”

Itbears further observation that Section 48(b) of Com. Act No, 141 is virtuallythe same as Section 14(1) of the Property Registration Decree.

SECTION14. Who may apply.— The following persons may file in the proper Court of FirstInstance an application for registration of title to land, whether personallyor through their duly authorized representatives:

(1)those who by themselves or through their predecessors-in-interest have been inopen, continuous, exclusive and notorious possession and occupation ofalienable and disposable lands of the public domain under a bona fide claimof ownership since June 12, 1945, or earlier.

Notwithstandingthe passage of the Property Registration Decree and the inclusion of Section14(1) therein, the Public Land Act has remained in effect.

Bothlaws commonly refer to persons or their predecessors-in-interest who “have beenin open, continuous, exclusive and notorious possession and occupation ofalienable and disposable lands of the public domain under a bona fide claimof ownership since June 12, 1945, or earlier.”

Theopening clauses of Section 48 of the Public Land Act and Section 14 of theProperty Registration Decree warrant comparison:

Sec.48 [of the Public Land Act]. The following described citizens of thePhilippines, occupying lands of the public domain or claiming to own any suchland or an interest therein, but whose titles have not been perfected orcompleted, may apply to the Court of First Instance of the province wherethe land is located for confirmation of

their claims and the issuance of acertificate of title therefor, under the Land Registration Act, to wit:

xxx

Sec.14 [of the Property Registration Decree]. Who may apply.— The following personsmay file in the proper Court of First Instance an application forregistration of title to land, whether personally or through their dulyauthorized representatives:

Itis indeed the Public Land Act that primarily establishes the substantiveownership of the possessor who has been in possession of the propertysince 12 June 1945.

Section14(a) of the Property Registration Decree recognizesthe substantive right granted under Section 48(b) of the Public Land Act,as well provides the corresponding original registration procedure for thejudicial confirmation of an imperfect or incomplete title.

Thereis another limitation to the right granted under Section 48(b). Section 47 ofthe Public Land Act (amended by Rep. Act No. 9176 in 2002) limits theperiod within which one may exercise the right to seek registration underSection 48.

Section47. The persons specified in the next following section are hereby grantedtime, not to extend beyond December 31, 2020 within which to avail of thebenefits of this Chapter: Provided, That this period shall apply only wherethe area applied for does not exceed twelve (12) hectares: Provided, further,That the several periods of time designated by the President in accordance withSection Forty-Five of this Act shall apply also to the lands comprised in theprovisions of this Chapter, but this Section shall not be construed asprohibiting any said persons from acting under this Chapter at any time priorto the period fixed by the President.

Thesubstantive right granted under Section 48(b) may be availed of only until 31December 2020. The OSG has adopted the position that for one to acquire theright to seek registration of an alienable and disposable land of the publicdomain, it is not enough that the applicant and his/herpredecessors-in-interest be in possession under a bona fide claim ofownership since 12 June 1945; the alienable and disposable character of theproperty must have been declared also as of 12 June 1945.

Followingthe OSG’s approach, all lands certified as alienable and disposable after 12June 1945 cannot be registered either under Section 14(1) of the PropertyRegistration Decree or Section 48(b) of the Public Land Act as amended.

Discussedin Naguit. – “adopting the OSG’s view, that all lands of the publicdomain which were not declared alienable or disposable before June 12, 1945would not be susceptible to original registration, no matter the length ofunchallenged possession by the occupant. Such interpretation renders paragraph(1) of Section 14 virtually inoperative and even precludes the government fromgiving it effect even as it decides to reclassify public agricultural lands asalienable and disposable. The unreasonableness of the situation would even beaggravated considering that before June 12, 1945, the Philippines was not yeteven considered an independent state. “

“[T]hemore reasonable interpretation of Section 14(1) is that it merely requiresthe property sought to be registered as already alienable and disposable at thetime the application for registration of title is filed.”

Petitionersmake the salient observation that the contradictory passages from Herbieto areobiter dicta since the land registration proceedings therein is void abinitio in the first place due to lack of the requisite publication of thenotice of initial hearing.

Theapplication therein was ultimately granted, citing Section 14(2).

Theevidence submitted by petitioners therein did not establish any mode ofpossession on their part prior to 1948, thereby precluding the applicationof Section 14(1). It is not even apparent from the decision whether petitionerstherein had claimed entitlement to original registration following Section14(1), their position being that they had been in exclusive possession under abona fide claim of ownership for over fifty (50) years, but not before 12 June1945.

TheCourt in Naguit offered the following discussion concerning Section14(2)

Prescriptionis one of the modes of acquiring ownership under the Civil Code. There is aconsistent jurisprudential rule that properties classified as alienable publicland may be converted into private property by reason of open, continuous andexclusive possession of at least thirty (30) years.[[31]] With such conversion,such property may now fall within the contemplation of “private lands” underSection 14(2), and thus susceptible to registration by those who have acquiredownership through prescription. Thus, even if possession of the alienablepublic land commenced on a date later than June 12, 1945, and such possessionbeing been open, continuous and exclusive, then the possessor may have theright to register the land by virtue of Section 14(2) of the PropertyRegistration Decree.

Theobiter in Naguit cited the Civil Code provisions on prescription as thepossible basis for application for original registration under Section 14(2).Specifically, it is Article 1113 which provides legal foundation for the application.It reads:

Allthings which are within the commerce of men are susceptible of prescription,unless otherwise provided. Property of the State or any of its subdivisionsnot patrimonial in character shall not be the object of prescription.

Thecritical qualification under Article 1113 of the Civil Code is thus:“[p]roperty of the State or any of its subdivisions not patrimonial incharacter shall not be the object of prescription.” The identification whatconsists of patrimonial property is provided by Articles 420 and 421

Art.420. The following things are property of public dominion:

(1)Those intended for public use, such as roads, canals, rivers, torrents, portsand bridges constructed by the State, banks, shores, roadsteads, and others ofsimilar character;

(2)Those which belong to the State, without being for public use, and are intendedfor some public service or for the development of the national wealth.

Art.421. All other property of the State, which is not of thecharacter stated in the preceding article, is patrimonial property

Itis clear that property of public dominion, which generally includes propertybelonging to the State, cannot be the object of prescription

Landsof the public domain, whether declared alienable and disposable or not, areproperty of public dominion and thus insusceptible to acquisition by prescription.

Article422 of the Civil Code states that “[p]roperty of public dominion, when nolonger intended for public use or for public service, shall form part of thepatrimonial property of the State.”

Accordingly,there must be an express declaration by the State that the public dominionproperty is no longer intended for public service or the development of thenational wealth or that the property has been converted into patrimonial.

Shouldpublic domain lands become patrimonial because they are declared as such in aduly enacted law or duly promulgated proclamation that they are no longerintended for public service or for the development of the national wealth, wouldthe period of possession prior to the conversion of such public dominion intopatrimonial be reckoned in counting the prescriptive period in favor of thepossessors? - We rule in the negative.

Asthe application for registration under Section 14(2) falls wholly within theframework of prescription under the Civil Code, there is no way that possessionduring the time that the land was still classified as public dominion propertycan be counted to meet the requisites of acquisitive prescription and justifyregistration.

Section14(1) mandates registration on the basis of possession, while Section 14(2)entitles registration on the basis of prescription.Registration under Section 14(1) is extended under the aegis of the PropertyRegistration Decree and the Public Land Act while registration under Section14(2) is made available both by the Property Registration Decree and the CivilCode.

Registrationunder Section 48(b) of the Public Land Act as amended by Rep. Act No. 1472 isbased on thirty years of possession alone without regard to the Civil Code,while the registration under Section 14(2) of the Property

Registration Decreeis founded on extraordinary prescription under the Civil Code.

Whetherunder ordinary prescription or extraordinary prescription, the period ofpossession preceding the classification of public dominion lands as patrimonialcannot be counted for the purpose of computing prescription. But after theproperty has been become patrimonial, the period of prescription begins to runin favor of the possessor.

Oncethe possessor automatically becomes the owner of the converted patrimonialproperty, the ideal next step is the registration of the property under theTorrens system. It should be remembered that registration of property is not amode of acquisition of ownership, but merely a mode of confirmation of ownership.

 

(1)In connection with Section 14(1) of the Property Registration Decree, Section48(b) of the Public Land Act recognizes and confirms that “those who bythemselves or through their predecessors in interest have been in open,continuous, exclusive, and notorious possession and occupation of alienable anddisposable lands of the public domain, under a bona fide claim of acquisitionof ownership, since June 12, 1945” have acquired ownership of, and registrabletitle to, such lands based on the length and quality of their possession.

(a)Since Section 48(b) merely requires possession since 12 June 1945 and doesnot require that the lands should have been alienable and disposable during theentire period of possession, the possessor is entitled to secure judicialconfirmation of his title thereto as soon as it is declared alienable anddisposable, subject to the timeframe imposed by Section 47 of the Public LandAct.

(b)The right to register granted under Section 48(b) of the Public Land Actis further confirmed by Section 14(1) of the Property RegistrationDecree.

(2)In complying with Section 14(2) of the Property Registration Decree,consider that under the Civil Code, prescription is recognized as a mode ofacquiring ownership of patrimonial property. However, public domain landsbecome only patrimonial property not only with a declaration that these arealienable or disposable. There must also be an express government manifestationthat the property is already patrimonial or no longer retained for public serviceor the development of national wealth, under Article 422 of the Civil Code. Andonly when

the property has become patrimonial can the prescriptive period forthe acquisition of property of the public dominion begin to run.

(a)Patrimonial property is private property of the government. The person acquiresownership of patrimonial property by prescription under the Civil Code isentitled to secure registration thereof under Section 14(2) of theProperty Registration Decree.

(b)There are two kinds of prescription by which patrimonial property may beacquired, one ordinary and other extraordinary. Under ordinary acquisitiveprescription, a person acquires ownership of a patrimonial property throughpossession for at least ten (10) years, in good faith and with just title.Under extraordinary acquisitive prescription, a person’s uninterrupted adversepossession of patrimonial property for at least thirty (30) years, regardlessof good faith or just title, ripens into ownership.

 

13(April)

JOSE CHING AND CARIDAD CHING, petitioners,

vs.

Hon. ANTONIO Q. MALAYA, as Presiding Judgeof the CFI of Laguna, Branch IV, Hon. MAXIMIANO C. ASUNCION, as Presiding Judgeof the CFI of Laguna Branch II, and Spouses CESAR ALVARADO and ARACELIALVARADO, respondents.

G.R. No.L-56449          August 31, 1987

CRUZ, J.:

--------------------------------------------------------------------

(Thispetition for certiorari under Rule 65 of the Rules of Courtseeks a reversal of the decision of the respondent court nullifying thejudgment of the municipal court in a forcible entry case on the ground of lackof jurisdiction.)

FACTS: The petitioners had alleged in their complaint for ejectment thatthe private respondents had forced their way into the disputed premises withoutany right whatsoever and had refused to vacate the same despite repeateddemands. These demands were based on the petitioners’ case that they were theowners of the said property, having acquired it by virtue of a valid sale. Theprivate respondents, in their answer, had challenged the claimed sale, arguingthat the property belonged to them by right of inheritance. At any rate, theyhad argued, as the basic question was one of ownership and not of merepossession, the municipal court had no jurisdiction and should dismiss thecomplaint.

Themunicipal court, affirming its jurisdiction, proceeded totrial and thereafter rendered judgment ordering the private respondents tovacate the disputed property. It also required them to pay the petitioners backand current rentals at P 1,000.00 a month until actual surrender of thepremises, as well as a P 3,000.00 attorney’s fee plus the costs of the suit. Onappeal, this decision was set aside by the respondent judge, who held that themunicipal court had no competence to resolve the case as it involved a questionof ownership. 

The Supreme Court provided the pertinent provisions of R.A. No. 296 readas follows:

Sec. 88. Original jurisdiction incivil cases. – “Inall civil actions, including those mentioned in Rules fifty-nine and sixty-two(now Rule 57 and 60) of the Rules of Court, arising in his municipality orcity, and not exclusively cognizable by the Court of First Instance, thejustice of the peace (now municipal judge) and the judge of a municipal court(now city court) shall have exclusive original jurisdiction ...”.

Inforcible entry and detainer proceedings, the justice of the peace or judge ofthe municipal court shall have original jurisdiction, but the said justice orjudge may receive evidence upon the question of title therein, whatever may bethe value of the property, solely for the purpose of determining the characterand extent of possession and damages for detention.

 

ISSUE:

1)Whetheror not the case at bar falls in the exception.  

2)Whetheror not party may introduceevidence of ownership to prove character of possession and amount of damagesfor unjust deprivation thereof.

RULING:  

1) No.

Afterexamining the facts, the Court finds that it does not come under the exceptionto the rule.

Theproperty in question consists of a residential house and lot covered by TCT No.T-85126 and registered in the name of petitioner Jose Ching in the Registry ofDeeds of Laguna. The basis of the registration is a deedof sale executed in his favor by Felix Carpio, the former owner, who hadacquired it from Brigido Alvarado, Cesar Alvarado’s supposed father. Therecord does not show that such registration has been challenged since theissuance in 1978 of the said certificate of title, which in the absence ofevidence to the contrary should be presumed valid.

Thereis no encumbrance on the land, and there is no adverse claim or notice of lis pendis annotated in thecertificate. Such registration, it may be added, isbinding against the whole world unless annulled for cause in proper cases.

TheCourt also provided that fact alone could not divest the municipal court ofjurisdiction to continue trying the question of possession, more so since thequestion of ownership was appropriately being litigated in the annulment suit.Significantly, the deed of sale being challenged in that action was differentfrom the contract involved in the exception

2) Yes.

TheCourt ruled that the fact that the petitioners themselves adduced evidence ofownership over the property in question did not, as claimed, have the effect ofdivesting the municipal court of its jurisdiction. As permitted in the Section88 of R.A. No. 296: the plaintiff in an ejectment case may introduce suchevidence for the purpose of proving the character of his possession and theamount of damages he is claiming for unjust deprivation of such possession. Thepetitioners were only trying to prove their right to possession and damages byestablishing their right of ownership.

14(Daniel Eblahan)

Legardav. Saleeby

GRno. L-8936, Oct 2, 1915

Johnson,J.:

Facts:

         Consuelo Legarda and her husband owned a lot in Ermita adjoining the lot of N.MSaleeby separated by a stone wall located on her side. The wall was not a jointwall. On March 2 1906, Consuelo petitioned the Court of Land Registration toregister their lot and the court allowed them. In the original certificate andtitle issued to them, provided by the Torrens system, the wall was included. OnMarch 25, 1912 N.M Saleeby also petitioned the Court of Land Registration toregister his lot and was also allowed. The wall was also included in hisoriginal certificate and title. The spouses discovered this error andpetitioned the Court of Land Registration to adjust and correct it. The courtdenied them because they did not object to the error when the defendantregistered his lot.

Issue:

         Whether N.M. Saleeby owns the wall and the land occupying said wall

Held:

         No, the Court disagreed with the reason of the lower court that the plaintiffsdid not object to the registration of Saleeby. The Court argued that shouldthis be the case, the plaintiffs would always be on alert if another personregisters the property and should immediately oppose. The Court declared thatif the holder of the certificate cannot rest secure in the registered title,the purpose of the Torrens system is defeated and nothing is gained by theregistration. The Torrens system intended to quiet title, putting a stop to anyquestion of the legality of the title.

         In this case, the Court decided to award theplaintiffs the property because they were the once who acquired and registeredit first and complied with the requirements of the law. The defendant cannotraise the defense that he is an innocent purchaser, because the presumptionthat he has examined every instrument affecting the title is irrebutable. Thedefendant should have taken notice that the property was registered to theplaintiffs six years before he did. The title, once registered, is notice tothe whole world. All persons must take notice.

Significance:

The purpose of the Torrens system is to quiet the title of a land registered under said system. Once registered it is notice to the whole world and shall be conclusive upon and against all persons, including the Government and all the branches thereof, whether mentioned by name in the application, notice, or citation, or included in the general description "To all whom it may concern."

15(Melodia)

__________________________________________________________________________________________________

16(Mel)

DINAH C. CASTILLOvsANTONIO M. ESCUTIN

G.R. No. 171056               March 13, 2009

Facts:

Petitioner is a judgment creditor of Raquel K. Moratilla. Racquel, her mother, Urbana Kalaw and sister, Perla Moratilla, co-owned Lot 13713, 15,000 square-meter, covered by Tax Declaration No. 00449.

When the petitioner verified the property, she found out that the application of Summit Point Golf & Country Club, Inc. for conversion of several agricultural landholdings, including Lot 13713, to residential, commercial, and recreational uses was approved and the property was not covered by a certificate of title, whether judicial or patent, or subject to the issuance of a Certificate of Land Ownership Award or patent under the Comprehensive Agrarian Reform Program.

Petitioner then proceeded to levy on execution Lot 13713. Before the scheduled public auction sale, petitioner learned that Lot 13713 was inside the Summit Point Golf and Country Club Subdivision owned by Summit Point Realty and Development Corporation. She immediately went to the Makati City office of Summit Realty to meet with its Vice President, Orense. However, she claimed that Orense did not show her any document to prove ownership of Lot 13713 by Summit Realty.

Petitioner bought Raquel’s 1/3 pro-indiviso share in Lot and was then issued Tax Declaration No. 00942-A, indicating that she owned 5,000 square meters of Lot 13713, while Urbana and Perla owned the other 10,000 square meters.

When petitioner attempted to pay real estate taxes, she was shocked to that, without giving her notice, her Tax Declaration No. 00942-A was cancelled. Lot 13713 was said to be encompassed in and overlapping with the 105,648 square meter parcel of land known as Lot 1-B, both in the name of Francisco Catigbac. The reverse side of TCT No. 129642 bore three entries, reflecting the supposed sale of Lot 1-B to Summit Realty.

In the supposed Deed of Absolute Sale in favor of Summit Realty by Leonardo Yagin, as Catigbac’s attorney-in-fact, it did not express the desire of Summit Realty to purchase Lot 1-B or indicate its consent and conformity to the terms of the Deed. There were also missing information in the said Deed.

Petitioner asserted that Summit Realty was well-aware of Catigbac’s death, having acknowledged the same in LRC Case No. 00-0376, the Petition for Issuance of New Owner’s Duplicate of TCT No. 181 In Lieu of Lost One, filed by Summit Realty before the Regional Trial Court of Lipa City. During the ex parte presentation of evidence in the latter part of 2000, Orense testified on behalf of Summit Realty that Catigbac’s property used to form part of a bigger parcel of land, Lot 1 of Plan Psu-12014, measuring 132,975 square meters, covered by TCT No. 181 in the name of Catigbac; after Catigbac’s death, Lot 1 was informally subdivided into several parts among his heirs and/or successors-in-interest, some of whom again transferred their shares to other persons; Summit Realty separately bought subdivided parts of Lot 181 from their respective owners, with a consolidated area of 105,648 square meters, and identified as Lot 1-B after survey; despite the subdivision and transfer of ownership of Lot 1, TCT No. 181 covering the same was never cancelled; and the owner’s duplicate of TCT No. 181 was lost and the fact of such loss was annotated at the back of the original copy of TCT No. 181 with the Registry of Deeds. Subsequently, in an Order dated 3 January 2001, the RTC granted the Petition in LRC Case No. 00-0376 and directed the issuance of a new owner’s duplicate of TCT No. 181 in the name of Catigbac, under the same terms and condition as in its original form.

Petitioner cast doubt on the acts undertaken by Summit Realty in connection with Catigbac’s property, purportedly without legal personality and capacity. The Special Power of Attorney dated 6 February 1976 granted Yagin the right to sue on behalf of Catigbac, yet it was Summit Realty which instituted LRC Case No. 00-0376, and Yagin had no participation at all in said case. Likewise, it was not Yagin, but Orense, who, through a letter dated 27 June 2001, requested the cancellation of TCT No. 181 covering Lot 1 and the issuance of a new certificate of title for Lot 1-B. Hence, it was Orense’s request which resulted in the issuance of TCT No. 129642 in the name of Catigbac, later cancelled and replaced by TCT No. T-134609 in the name of Summit Realty.

Petitioner questioned why, despite the cancellation of TCT No. 129642 in the name of Catigbac and the issuance in its place of TCT No. T-134609 in the name of Summit Realty, it was the former cancelled title which was used as basis for canceling petitioner’s Tax Declaration No. 00942-A. Tax Declaration No. 00949-A was thus still issued in the name of Catigbac, instead of Summit Realty.

Summit Realty bought Lot 1-B measuring 105,648 square meters, specifically covered by TCT No. 129642, both in the name of Catigbac. As a result of such purchase, ownership of Lot 1-B was transferred from Catigbac to Summit Realty. Summit Realty had every reason to believe in good faith that said property was indeed owned by Catigbac on the basis of the latter’s certificate of title over the

same. Catigbac’s right as registered owner of Lot 1-B under TCT No. 181/No. 129642, was superior to petitioner’s, which was based on a mere tax declaration.

Issue:

Whether petitioner was indeed unlawfully deprived of her 5,000 square meter property.

Ruling:

As between Catigbac’s title, covered by a certificate of title, and petitioner’s title, evidenced only by a tax declaration, the former is evidently far superior and is, in the absence of any other certificate of title to the same property, conclusive and indefeasible as to Catigbac’s ownership of Lot 1-B. Catigbac’s certificate of title is binding upon the whole world, including respondent public officers and even petitioner herself. The Court ruled that tax declarations and corresponding tax receipts cannot be used to prove title to or ownership of a real property inasmuch as they are not conclusive evidence of the same. Petitioner acquired her title to the 5,000 square meter property from Raquel who, it is important to note, likewise only had a tax declaration to evidence her title. In addition, the Court of Appeals aptly observed that, "curiously, as to how and when petitioner’s alleged predecessor-in-interest, Raquel K. Moratilla and her supposed co-owners acquired portions of Lot 1 described as Lot 13713 stated in TD No. 00449, petitioner had so far remained utterly silent."

A title is different from a certificate of title. Title is generally defined as the lawful cause or ground of possessing that which is ours. It is that which is the foundation of ownership of property, real or personal. Title, therefore, may be defined briefly as that which constitutes a just cause of exclusive possession, or which is the foundation of ownership of property. Certificate of title, on the other hand, is a mere evidence of ownership; it is not the title to the land itself. Under the Torrens system, a certificate of title may be an Original Certificate of Title, which constitutes a true copy of the decree of registration; or a Transfer Certificate of Title, issued subsequent to the original registration.

___________________________________________________________________________________________________

 17(Rocky) 

Philippine National Bank vs.Court of Appeals, 153 SCRA 435

FACTS: Thesubject of the action is 30 parcels of land which they claim to be the conjugalproperty of the spouses Donata Montemayor and Clodualdo Vitug of which theyclaim a share of 2/11 of 1/2 thereof. They assailed the mortgage to the PNB andthe public auction of the properties as null and void. They invoked the case ofVitug vs. Montemayor, L-5297 decided by this Court on Oct. 20, 1953 which is anaction for partition and liquidation of the said 30 parcels of land wherein theproperties were found to be conjugal in nature.

ISSUE: Doesthe presumption of conjugality of properties acquired by the spouses duringcoverture provided for in Article 160 of the Civil Code apply to propertycovered by a Torrens certificate of title in the name of the widow?

HELD: The petitionis impressed with merit.

When the subject propertieswere mortgaged to the PNB they were registered in the name of DonataMontemayor, widow. Relying on the torrens certificate of title covering saidproperties the mortgage loan applications of Donata were granted by the PNB andthe mortgages were duly constituted and registered in the office of theRegister of Deeds. In processing the loan applications of Donata Montemayor,the PNB had the right to rely on what appears in the certificates of title andno more. On its face the properties are owned by Donata Montemayor, a widow.The PNB had no reason to doubt nor question the status of said registered ownerand her ownership thereof.

The well-known rule in thisjurisdiction is that a person dealing with a registered land has a right torely upon the face of the torrens certificate of title and to dispense with theneed of inquiring further, except when the party concerned has actual knowledgeof facts and circumstances that would impel a reasonably cautious man make suchinquiry.

The presumption applies toproperty acquired during the lifetime of the husband and wife. In this case, itappears on the face of the title that the properties were acquired by DonataMontemayor when she was already a widow. When the property is registered in thename of a spouse only and there is no showing as to when the property wasacquired by said spouse, this is an indication that the property belongsexclusively to said spouse. The PNB had a reason to rely on what appears on thecertificates of title of the properties mortgaged.

Indeed, if the PNB knew ofthe conjugal nature of said properties it would not have approved the mortgageapplications covering said properties of Donata Montemayor without requiringthe consent of all the other heirs or co-owners thereof. Moreover,

when saidproperties were sold at public auction, the PNB was a purchaser for value ingood faith. So its right thereto is beyond question.

As correctly held by thelower court. Pragmacio and Maximo Vitug as occupants and lessees of theproperty in question cannot now dispute the ownership of their mother over thesame who was their lessor.

Hilario vs. City of Manila,19 SCRA 931

FACTS: Dr.Jose Hilario was the registered owner of a large tract of land around 49hectares in area (Barrio Guinayang, San Mateo, Rizal). Upon his death thisproperty was inherited by his son, Jose Hilario, Jr., to whom a new certificateof title was issued. During the lifetime of plaintiff’s father, the Hilarioestate was bounded on the western side by the San Mateo River. To prevent itsentry into the land, a bamboo and lumber post dike or ditch was constructed onthe northwestern side. This was further fortified by a stonewall built on thenorthern side. For years, these safeguards served their purpose. However, in1937, a great and extraordinary flood occurred which inundated the entire placeincluding the neighboring barrios and municipalities. The River destroyed thedike on the northwest, left its original bed and meandered into the Hilarioestate, segregating from the rest thereof a lenticular piece of land. Thedisputed area is on the eastern side of this lenticular strip which now standsbetween the old riverbed site and the new course. In 1945, the US Army opened asand and gravel plant within the premises, and started scraping, excavating andextracting soil, gravel and sand from the nearby areas along the River. Theoperations eventually extended northward into the strip of land. Consequently,a claim for damages was filed with the US War Department by Luis Hidalgo, thethen administrator of Dr. Hilario’s estate. The US Army paid. In 1947, theplant was turned over to herein defendants-appellants and appellee who tookover its operations.

On 22 October 22, 1949,plaintiff filed his complaint for injunction and damages against the defendantsCity Engineer of Manila, District Engineer of Rizal, the Director of PublicWorks, and Engr. Busuego, the Engineer-in-charge of the plant. Subsequently,the Bureau of Mines and Atty. Maximo Calalang were respectively allowed to jointhe litigation as intervenors; as per issue of fees and penalties for materials(sand and gravel) extracted. On 14 March 1954, defendants filed a petition forinjunction against plaintiff and intervenor Calalang in the same case, allegingthat the latter have fenced off the disputed area in contravention of anagreement had between the latter and the Director of Public Works wherein thedefendants were allowed to continue their operations but subject to the finaloutcome of the pending suit. On 13 May 1954, plaintiff amended his complaintand impleaded as additional defendants the City of Manila, the ProvincialTreasurer of Rizal, and Engr. Eulogio Sese, the new Engineer-in-charge of theplant. Plaintiff also converted his claim to one purely for damages directedagainst the City of Manila and the Director of Public

Works, solidarily, in theamount of P1,000,000.00, as the cost of materials taken since 1949, as well asthose to be extracted therefrom until defendants stop their operations. On 21December 1956, the lower court rendered its decision, ordering the City ofManila and Director of Public Works to pay Hilario in solidum the sum ofP376,989.60 as cost of gravel and sand extracted from the plaintiff’s land,plus costs; and ordering the Provincial Treasurer of Rizal to reimburseintervenor Calalang of P36.80 representing gravel fees illegally collected.None of the parties litigants seemed satisfied with this decision and they allsought a reconsideration of the same. On August 30, 1957, the lower courtresolved the motions to reconsider with an order, holding that the 2/5 portionof the area in controversy to Hilario, and dismissing the case against theBureau of Public Works insofar as money claims are concerned without prejudiceto Hilario taking action against proper party in such claim. Hilario andCalalang filed a second motion for reconsideration, which the lower court denied.Hence, the appeal.

ISSUES: (a)Whether or not the northern two-fifths of the disputed area belongs toplaintiff Hilario. (b) When a river, leaving its old bed, changes its originalcourse and opens a new one through private property, would the new riverbankslining said course be of public ownership also?

HELD: (a) TheSupreme Court set aside the decision and orders appealed from, and enteredanother judgment to the effect that the City of Manila and the Director ofPublic Works, and his agent and employees, are absolved of liability fromextracting materials from subject property (of public domain); and the portionwithin the strip of land question declared not part of public domain andconfirmed as part of Hilario’s private property. No Costs.

(b) From 1945 to 1949, thewest bank of the River extended westward up to the “secondary bank” line; from1950 to 1952, this bank had moved, with the River, to the east, its lateralborders running along a line just 20 meters west of the camachile tree; andfrom 1953 to 1955, the extremities of the west bank further receded eastwardbeyond the camachile tree, until they lay just about 20 meters east of saidtree. Evidence shows that the River floods with annual regularity during therainy season. These floods can hardly be called “accidental”. The Colegio deSan Jose case is not exactly in point. What was mainly considered there wasArticle 74 of the Law of Waters relating to lakes, ponds and pools. In thepresent case, none of these is involved.

Defendants cannot be accusedof unjustly profiting at plaintiff’s expense. They were not responsible for theshifting of the river. It was due to natural causes for which no one can beblamed. Further, defendants were extracting from public property then, underproper authorization. The government, through the defendants, may have beenenriched by chance, but not unjustly.

Director of Lands vs. Courtof Appeals, 179 SCRA 522

FACTS: Theland in question is situated in Obando, Bulacan. It adjoins the Kailogan Riverand private respondent Valeriano have converted it into a fishpond. In theirapplication in 1976, private respondents claimed that they are the co-owners infee simple of the land partly through inheritance and partly by purchase andthat; it is not within any forest or military reservation. The Republic of thePhil., represented by the Dir of the Bureau of Forest Development, opposed theapplication on the principal ground that the land applied for is WITHINTHEUNCLASSIFIED REGION of Obando, Bulacan and that such area are denominated asFORESTLANDS-do not form part of the disposable and alienable portion of thepublic domain. The Trial Court ordered registration of the subject land infavor of the Valerianos. This was affirmed by the CA which said in part that´since the subject property is entirely devoted to fishpond purposes, it cannotbe categorized as part of forest lands.

ISSUE: Whetheror not the courts can reclassify the subject public land.

HELD: Courtscannot reclassify... it’s beyond their competence and jurisdiction. Theclassification of public lands is an exclusive prerogative of the Executive Departmentof the Government (Bureau of Forest Development) and not of the Courts. In the absenceof such classification, the land remains as unclassified land until it isreleased therefrom and rendered open to disposition. Since the subject propertyis still unclassified, whatever possession Applicants (Valeriano) may have had,and, however long, cannot ripen into private ownership. The conversion of thesubject property into a fishpond by Applicants does not automatically renderthe property as alienable and disposable. The recommendation of the DistrictForester for release of subject property from unclassified region is not theultimate word on the matter.

Ong vs. Republic, GR No.175746, 12 March 2008, 548 SCRA 160

FACTS: OnJuly 1, 1999, petitioner Charles L. Ong (petitioner) in his behalf and as dulyauthorized representative of his brothers, namely, Roberto, Alberto and Cesar,filed an Application for Registration of Title[4] over Lot 15911 (subject lot)situated in Barangay Anolid, Mangaldan, Pangasinan with an area of five hundredseventy four (574) square meters, more or less. They alleged that they are theco-owners of the subject lot; that the subject lot is their exclusive propertyhaving acquired the same by purchase from spouses Tony Bautista and AliciaVillamil on August 24, 1998; that the subject lot is presently unoccupied; andthat they and their predecessors-in-interest have been in open, continuous andpeaceful possession of the subject lot in the concept of owners for more thanthirty (30) years.

After due notice andpublication, only respondent Republic of the Philippines (respondent),represented by the Office of the Solicitor General, opposed the application forregistration of title. Respondent asserted that neither applicants nor theirpredecessors-in-interest have been in open, continuous, exclusive and notoriouspossession and occupation of the subject lot since June 12, 1945 or earlier asrequired by Section 48(b) of Commonwealth Act No. 141, as amended byPresidential Decree (P.D.) No. 1073; that applicants failed to adduce anymuniment of title to prove their claims; that the tax declaration appended tothe application does not appear genuine and merely shows pretended possessionof recent vintage; that the application was filed beyond the period allowedunder P.D. No. 892; and that the subject lot is part of the public domain whichcannot be the subject of private appropriation.

On January 16, 2002, thetrial court rendered a Decision in favor of petitioner and his brothers.

Aggrieved, respondentappealed to the Court of Appeals which rendered the assailed Decision.

In reversing the decision ofthe trial court, the Court of Appeals found that the subject lot is part of thealienable and disposable lands of the public domain. Thus, it was incumbentupon petitioner to prove that they possessed the subject lot in the nature andfor the duration required by law. However, petitioner failed to prove that heor his predecessors-in-interest have been in adverse possession of the subjectlot in the concept of owner since June 12, 1945 or earlier as mandated bySection 14(1) of P.D. 1529. It noted that the earliest tax declaration whichpetitioner presented is dated 1971. Consequently, petitioner could not fairlyclaim possession of the land prior to 1971. Neither was petitioner able toprove that he or his predecessors-in-interest actually occupied the subject lotprior to the filing of the application. Thus, the trial court erred in grantingthe application for registration of title over the subject lot.

ISSUE: (a)Whether or not petitioner, together with his brothers have registrable ownershipover the real property and (b) whether or not the findings and conclusions ofthe Court of Appeals that the subject real property is a public land iscorrect.

HELD: Thepetition lacks merit. The law provides that applicants for registration oftitle must prove that: (1) the subject land forms part of the disposable andalienable lands of the public domain, and (2) that they have been in open,continuous, exclusive and notorious possession and occupation of the same undera bona fide claim of ownership since June 12, 1945, or earlier. Theserequisites involve questions of fact which are not proper in a petition forreview on certiorari.

There is no dispute that thesubject lot is classified as alienable and disposable land of the publicdomain. The Report of the Bureau of Lands stated that the subject lot is

withinthe alienable and disposable zone. This finding is, likewise, embodied in theReport of the Department of Environment and Natural Resources CommunityEnvironment and Natural Resources Office (DENR-CENRO) and the blue print Copyof the plan covering the subject lot. However, petitioner failed to prove thathe or his predecessors-in-interest have been in open, continuous, exclusive andnotorious possession and occupation of the subject lot since June 12, 1945 orearlier.

Further, as correctlypointed by the Court of Appeals, possession alone is not sufficient to acquiretitle to alienable lands of the public domain because the law requirespossession and occupation.

Director of Lands vs. C.A,129 SCRA 689

FACTS: Petitionersseek a review of the Decision and Resolution of the Court of Appeals affirmingthe judgment of the Court of First Instance of Bulacan, decreeing registrationof a parcel of land in private respondents' favor. The land in question, issituated in Obando, Bulacan, and has an area of approximately 9.3 hectares. Itadjoins the Kailogan River and private respondents have converted it into afishpond.

In their application forregistration, private respondents claimed that they are the co-owners in feesimple of the land applied for partly through inheritance in 1918 and partly bypurchase on May 2, 1958; that it is not within any forest zone or militaryreservation; and that the same is assessed for taxation purposes in theirnames.

The Republic of thePhilippines, represented by the Director of the Bureau of Forest Developmentopposed the application on the principal ground that the land applied for iswithin the unclassified region of Obando, Bulacan, and that areas within theunclassified region are denominated as forest lands and do not form part of thedisposable and alienable portion of the pub

18 (Maribeth)

Bornales vs Intermediate Appellate Court, 166SCRA 519

FACTS:

In 1927, SixtoDumolong married to Isabel Marquez-Dumolong was awarded a parcel of landdenominated as Lot 1318 and covered by Original Certificate of Title No. 6161. Since1920 however, said spouses have been living separately and that

they had nochildren. Sixto then cohabited with one Placida Dumolong and they had children,one of which is Renito Dumolong.

In March 1978 aDeed of Extrajudicial Adjudication and Sale of Real Property was executed byRenito and Isabel as settlement of the conjugal estate of Sixto involving Lot1318 and its eventual sale. A supposed thumbmarkof Isabel appeared in the document. Thedeed was registered and pursuant thereto Transfer Certificate of Title No.T-15856 was issued in the name of the vendees. The subject lot was further sold to Spouses Bornales, the cultivators ofthe subject lot, and eventually they were able to secure a TCT in their names.    

On March 11,1980, Isabel filed an action for reconveyance and damages against SpousesBornales, Placida, Renito and all the other parties named in the ExtrajudicialAdjudication and Sale of Real Property alleging forgery thereof. The lower court rendered judgement in favorof Isabel and against all the defendants. Spouses Bornales were declared purchasers in bad faith. On appeal, the appellate court affirmed theappealed decision with modifications as it found that the subject land was theexclusive property of Sixto who had other illegitimate children surviving withRenito. It further denied the hereinpetitioners motion for reconsideration, hence, the petition.

ISSUE: Whether or not SpousesBornales may invoke the indefeasibilityof a certificate of title?

HELD: No. Having bought the land registered under the Torrenssystem from the vendor who procured title thereto by means of fraud, they cannotinvoke the indefeasibility of a certificate of title against Isabel to theextent of her interest over the lot.  TheTorrens system of land registration should not be used as a means to perpetratefraud against the rightful owner of real property.  Registration, to be effective, must be made ingood faith.  It is a settled rule thatthe defense of indefeasibility of a certificate of title does not extend to atransferee who takes it with notice of the flaws in his transferor’s title.

19 (Ailyn)

20 (Jen)

21 (Bambi) - Hindi ko mahanap ang Galloy vs. CA, 173 SCRA 26

22 (Nelson)

HACIENDA BIGAA, INC.,

Petitioner, 

     versus

 EPIFANIO V. CHAVEZ (deceased), substituted by SANTIAGO V. CHAVEZ,

Respondent. -- -

G.R. No. 174160

Facts:

                Hacienda Calatagan covered by TCT 722 with an area of 9,652.583 hectares was owned by Ayala y Cia and the Zobels, they expanded the lot to cover 2000 hectares consisting, among others of beach, foreshores and bay areas, and navigable waters(excess areas), making it appear that these excess areas are part of the hacienda. They sold the excess area to different third parties; one of them was the Hacienda Bigaan(herein petitioner). Notwithstanding the prior ruling of the court dated 1965 that renders Hacienda Calatagan as public land including the expanded area outside the land, they still sold it to another contending that the subject land is not included in the prior decision(antecedent cases of Dizon, Ayala y Cia, and De los Angeles).

Hacienda Bigaa also contended that the rulings in the antecedent cases on the nullity of its subdivision titles should not apply to the present case because the titles TCT Nos. 44695 and 56120 have not been specifically declared void by court order and must be given probative value. It likewise posits that Chavez failed to introduce evidence before the MTC that the land subject matter of the suit is the same land covered by the decision of the Supreme Court in the antecedent cases.

On June 5, 1996, petitioner Hacienda Bigaa, a buyer or transferee of the subject land, filed with the Municipal Trial Court (MTC) of Calatagan, Batangas a complaint[for ejectment (forcible entry) and damages with application for writ of preliminary injunction against respondent Epifanio V. Chavez (Chavez), docketed as Civil Case

No. 129. The complaint alleged that Chavez, by force, strategy and/or stealth, entered on April 29, 1996 the premises of Hacienda Bigaa's properties (subject land) by cutting through a section of the barbed wire fence surrounding the properties and destroying the lock of one of its gates, subsequently building a house on the property, and occupying the lots without the prior consent and against the will of Hacienda Bigaa.

Defendant (now respondent) Chavez alleged in his answer before the MTC of Calatagan that his mother, Zoila de Chavez (who died intestate on September 14, 1979) was a fishpond permittee/lessee under Fishpond Permit Nos. F-4572-0 and F-24735 issued by the Bureau of Fisheries on April 21, 1959 and June 3, 1966, respectively; that the areas covered by the permits are the same parcels of land which he presently occupies as Zoila's successor-in-interest and which Hacienda Bigaa also claims.

Issue:

1. Whether or not Chavez has a better claim over the subject land.2. Whether or not Hacienda Bigaa’s title carry no probative value.

Held:

1. NO. As framed above, the case before us inevitably brings to memory the antecedent decided cases touching on the ownership of the vast tract of land in Calatagan, Batangas, covered by Transfer Certificate of Title (TCT) No. 722 in the name/s of Ayala y Cia, Alfonso Zobel, Jacobo Zobel and Enrique Zobel and/or Hacienda Calatagan the predecessors-in-interest of petitioner Hacienda Bigaa. We ruled in the antecedent cases of Dizon, Ayala y Cia, and De los Angeles, that: (1) all expanded subdivision titles issued in the name of Ayala y Cia, the Zobels and/or Hacienda Calatagan covering areas beyond the true extent of TCT No. 722 are null and void because they cover areas belonging to the public domain; (2) Ayala y Cia and the Zobels of Hacienda Calatagan are mere usurpers of these public domain areas; and that (3) these areas must revert to the Republic.Significantly, we declared in De los Angeles that the Republic, as the rightful owner of the expanded areas portions of the public domain has the right to place its lessees and permittees (among them Zoila de Chavez) in possession of the fishpond lots whose ownership and possession were in issue in the case.

1. Yes. The Court rejected this contention in light of our holding in the Ayala y Cia and De los Angeles cases that apart from those expressly litigated and annulled, all other subdivision titles over the excess areas of Hacienda Calatagan must be nullified for covering unregisterable lands of the public

domain that must revert to the Republic.  To reiterate, lots and their titles derived from the Ayalas and the Zobels TCT No. 722 not shown to be within the original coverage of this title are conclusively public domain areas and their titles will be struck down as nullities. What could have saved Hacienda Bigaa, as successor-in-interest of the Ayalas and the Zobels, is competent evidence that the subdivision titles in its possession do not fall within the excess areas of TCT No. 722 that are null and void because they are lands of the public domain. Hacienda Bigaa however failed to discharge this burden.

Significance:

                The registration of lands of the public domain under Torrens System, by itself cannot convert public land into private land.

23 (JM)

 

 Director of Forestry vs. Munoz, 23 SCRA 1183 (L-24796, June 28, 1968)

 

FACTS:

 

PINAGCAMALIGAN INDO-AGRO DEVELOPMENTCORPORATION, INC (Piadeco) is a company engaged in logging. It was given a Certificate ofPrivate Woodland Registration sothat it can operate in a 72000 hectare.It also has a Titulo de Propriedad which it acquired in1894 under the Spanish regime.

In 1964, the NAWASA director orderedthe cancellation of Piadeco’s certificate because it encroached beyond what wasallowed in the certificate. It actually cut trees inthe Angat and Marikina watershed area which was prohibited. The lower courtruled in favor of Piadeco. Piadeco also had a settlement with Nawasa. Piadecosought to renew its certificate but it was deniedby the Asst. Director of Forestry. The latter ruled that the Spanish title isno longer recognized and should have never been used to applyfor a Certificate.

ISSUE:

Whether or not Piadeco can claimownership over the property.

HELD:

No. The Spanish title it acquired cannotbe used to register for another Certificate. There should be no question nowthat Forestry Administrative Order 12-2 has the forceand effect of law. It was promulgated pursuant to law. Section 1817, RevisedAdministrative Code, empowers the Bureau of Forestry, with the approval of the department head, to issue regulations deemed expedient or necessary to secure the protection and conservation of thepublic forests in such manner as to insure a continuedsupply of valuable timber andother forest products for the future, and regulating the use and occupancy of theforests and forest reserves, to the same end. It is an administrative regulation germane to the objects and purposes of the law. 

24 (Maris)

25 (Reg)

26 (Greg)

27 (Ed)

28 (LJ)

29 (Zax)

30 (Clathem)

31 (Lea)

32 (Rubie)

#19 Ailyn

Part 1

G.R. No. 77294     December 12, 1988

 

ANGELICA VIAJAR and CELSO VIAJAR,plaintiffs-appellants,

 

vs.

 

COURT OF APPEALS, LEONOR P. LADRIDO,LOURDES LADRIDO IGNACIO, EUGENIO P. LADRIDO and L P. LADRIDO,defendants-appellees.

FACTS

The spouses Ricardo Y. Ladrido andLeonor P. Ladrido were the owners of Lot No. 7511 of the Cadastral Survey ofPototan situated in barangay Cawayan, Pototan, Iloilo. Spouses Rosendo H. Teand Ana Te were also the registered owners of a parcel of land described intheir title as Lot No. 7340 of the Cadastral Survey of Pototan. On September 6,1973, Rosendo H. Te, with the conformity of Ana Te, sold this lot to AngelicaF. Viajar and Celso F. Viajar for P5,000. A Torrens title was later issued inthe names of Angelica F. Viajar and Celso F. Viajar. Later, Angelica F. Viajarhad Lot No. 7340 relocated and found out that the property was in thepossession of Ricardo Y. Ladrido. Consequently, she demanded its return butLadrido refused. On February 15, 1974, Angelica F. Viajar and Celso F. Viajarinstituted a civil action for recovery of possession and damages againstRicardo Y. Ladrido.  Summoned to plead,defendant Ladrido filed his answer with a counterclaim. The Viajars filed theirreply to the answer. Subsequently, the complaint was amended to implead RosendoH. Te as another defendant. The Viajars sought the annulment of the deed ofsale and the restitution of the purchase price with interest in the event thepossession of defendant Ladrido is sustained. Defendant Te filed his answer tothe amended complaint and he counter claimed for damages. The Viajars answeredthe counterclaim. During the pendency of the case, plaintiff Celso F. Viajarsold his rights over Lot No. 7340 to his mother and co-plaintiff, Angelica F.Viajar. For this reason, plaintiff Angelica F. Viajar now appears to be the soleregistered owner of this lot. On May 25, 1978, defendant Ladrido died. He wassubstituted in the civil action by his wife, Leonor P. Ladrido, and their children.

The facts admitted by the partiesduring the pre-trial show that the piece of real property which used to be LotNo. 7340 of the Cadastral Survey of Pototan was located in barangay GuibuanoganPototan, Iloilo; that it consisted of 20,089 square meters; that at the time ofthe cadastral survey in 1926, Lot No. 7511 and Lot No. 7340 were separated bythe Suague River; that the area of 11,819 square meters of what was Lot No.7340 has been in the possession of the defendants; that the area of 14,036square meters, which was formerly the river bed of the Suague River percadastral survey of 1926, has also been in the possession of the defendants;and that the plaintiffs have never been in actual physical possession of LotNo. 7340. On December 10, 1981, the trial court rendered its decision in favorof the defendants Ladrido. Not satisfied with the decision, the plaintiffsappealed to the Court of Appeals.  In thedecision

appealed from, the Court of Appeals held that the appeal of Viajar isnot impressed with merit.

ISSUE

            Whetherthe registration of the land is not a mode of acquiring ownership

RULING

            TheCourt affirmed the decision of the lower court.

The rulethat registration under the Torrens System does not protect the riparian owneragainst the diminution of the area of his registered land through gradualchanges in the course of an adjoining stream is well settled.

Thecontroversy in the present cases seems to be due to the erroneous conceptionthat Art. 366 of the Civil Code does not apply to Torrens registered land. Thatarticle provides that "any accretions which the banks of rivers maygradually receive from the effects of the current belong to the owners of theestates bordering thereon." Accretions of that character are naturalincidents to land bordering on running streams and are not affected by theregistration laws. It follows that registration does not protect the riparianowner against diminution of the area of his land through gradual changes in thecourse of the adjoining stream.

In C.N.Hodges vs. Garcia, 109 Phil. 133, The Court  ruled:

It clearlyappearing that the land in question has become part of defendant's estate as aresult of accretion, it follows that said land now belongs to him. The

factthat the accretion to his land used to pertain to plaintiffs estate, which iscovered by a Torrens Certificate of Title, cannot preclude him (defendant) frombeing the owner thereof. Registration does not protect the riparian owneragainst the diminution of the area of his land through gradual changes in the courseof the adjoining stream. Accretions which the banks of rivers may graduallyreceive from the effect of the current become the property of the owners of thebanks (Art. 366 of the Old Civil Code; Art. 457 of the New). Such accretionsare natural incidents to land bordering on running streams and the provisionsof the Civil Code in that respect are not affected by the Registration Act.

Part 2

Non-registrability of navigablerivers

G.R. No. L-31271   April 29, 1974

 

ROMEO MARTINEZ and LEONOR SUAREZ,spouses, petitioners-appellants,

 

vs.

 

HON. COURT OF APPEALS, SECRETARY andUNDERSECRETARY OF PUBLIC WORKS & COMMUNICATIONS, respondents-appellees.

 

FACTS

                       

The spouses Romeo Martinez andLeonor Suarez, now petitioners-appellees, are the registered owners of two (2)parcels of land located in Lubao, Pampanga. Both parcels of land are fishponds.The disputed property was originally owned by one Paulino Montemayor, whosecured a "titulo real" over it way back in 1883. After the death ofPaulino Montemayor the said property passed to his successors-in-interest,Maria Montemayor and Donata Montemayor, who in turn, sold it, as well as thefirst parcel, to a certain Potenciano Garcia. Because Potenciano Garcia wasprevented by the then municipal president of Lubao, Pedro Beltran, fromrestoring the dikes constructed on the contested property, the former, on June22, 1914, filed Civil Case No. 1407 with the Court of First Instance againstthe said Pedro Beltran to restrain the latter in his official capacity frommolesting him in the possession of said second parcel, and on even date,applied for a writ of preliminary injunction, which was issued against saidmunicipal president. The Court, by decision promulgated June 12, 1916, declaredpermanent the preliminary injunction, which, decision, on appeal, was affirmedby the Supreme Court on August 21, 1918. From June 22, 1914, the dikes aroundthe property in question remained closed until a portion thereof was againopened just before the outbreak of the Pacific War.

On April17, 1925. Potenciano Garcia applied for the registration of both parcels ofland in his name, and the Court of First Instance of Pampanga, sitting as landregistration court, granted the registration over and against the opposition ofthe Attorney-General and the Director of Forestry. Pursuant to the Court'sdecision, original certificate of title No. 14318, covering said parcels 1 and2 was issued to the spouses Potenciano Garcia and Lorenza Sioson. These parcelsof land were subsequently bought by Emilio Cruz de Dios in whose name transfercertificate of title No. 1421 was first issued on November 9, 1925. Thereafter,the ownership of these properties changed

hands until eventually they wereacquired by the herein appellee spouses who hold them by virtue of transfercertificate of title No. 15856.

To avoid any untoward incident,the disputants agreed to refer the matter to the Committee on Rivers andStreams, by then composed of the Honorable Pedro Tuason, at that time Secretaryof Justice, as chairman, and the Honorable Salvador Araneta and Vicente Orosa,Secretary of Agriculture and National Resources and Secretary of Public Worksand Communications, respectively, as members. This committee thereafterappointed a Sub-Committee to investigate the case and to conduct an ocularinspection of the contested property, and on March 11, 1954, said Sub-Committeesubmitted its report to the Committee on Rivers and Streams to the effect thatParcel No. 2 of transfer certificate of title No. 15856 was not a public riverbut a private fishpond owned by the herein spouses.

On July 7,1954, the Committee on Rivers and Streams rendered its decision restoring theownership to the exclusive possession of spouses Martinez

Themunicipal officials of Lubao, led by Acting Mayor Mariano Zagad, apparentlyrefused to recognize the above decision, because on September 1, 1954, thespouses Romeo Martinez and Leonor Suarez instituted Civil Case No. 751 beforethe Court of First Instance of Pampanga against said Mayor Zagad, praying thatthe latter be enjoined from molesting them in their possession of theirproperty and in the construction of the dikes therein. The writ of preliminaryinjunction applied for was issued against the respondent municipal Mayor, whoimmediately elevated the injunction suit for review to the Supreme Court, whichdismissed Mayor Zagad's petition on September 7, 1953. With this dismissalorder herein appellee spouses proceeded to construct the dikes in the disputedparcel of land.

Some four(4) years later, and while Civil Case No. 751 was still pending the HonorableFlorencio Moreno, then Secretary of Public Works and Communications, orderedanother investigation of the said parcel of land, directing the appelleesherein to remove the dikes they had constructed, on the strength of theauthority vested in him by Republic Act No. 2056, approved on June 13, 1958,entitled "An Act To Prohibit, Remove and/or Demolish the Construction ofDams. Dikes, Or Any Other Walls In Public Navigable Waters, Or Waterways and InCommunal Fishing Grounds, To Regulate Works in Such

Waters or Waterways And InCommunal Fishing Grounds, And To Provide Penalties For Its Violation, And ForOther Purposes. The said order which gave rise to the instant proceedings,embodied a threat that the dikes would be demolished should the hereinappellees fail to comply therewith within thirty (30) days.

The spouses Martinez replied tothe order by commencing on January 2, 1959 the present case, which was decidedin their favor by the lower Court. Asagainst this judgment respondent officials of the Department of Public Worksand Communications took the instant appeal.

ISSUE

            Whethernavigable rivers could be registered as private property

RULING

            TheCourt affirmed that the judgment of the Court of Appeals appealed from is inaccordance with law . The above-mentioned properties are parts of the publicdomain intended for public use, therefore they are outside the commerce of menand not subject to private appropriation.

In Ledesmav. Municipality of Iloilo, 49 Phil. 769, the Court held:

A simplepossession of a certificate of title under the Torrens system does notnecessarily make the possessor a true owner of all the property describedtherein. If a person obtains title under the Torrens system which includes bymistake or oversight, lands which cannot be registered under the Torrenssystem, he does not by virtue of said certificate alone become the owner of theland illegally included.

In Mercadov. Municipal President of Macabebe, 59 Phil. 592, it was also said:

It isuseless for the appellant now to allege that she has obtained certificate oftitle No. 329 in her favor because the said certificate does not confer uponher any right to the creek in question, inasmuch as the said creek, being ofthe public domain, is included among the various exceptions enumerated inSection 39 of Act 496 to which the said certificate is subject by expressprovision of the law.

The sameruling was laid down in Director of Lands v. Roman Catholic Bishop ofZamboanga, 61 Phil. 644, as regards public plaza.

In Dizon,et al. v. Rodriguez, et al., G.R. No. L-20300-01 and G.R. No. L-20355-56, April30, 1965, 20 SCRA 704, it was held that the incontestable and indefeasiblecharacter of a Torrens certificate of title does not operate when the landcovered thereby is not capable of registration.

It is now clearthat the authorities cited by the appellants as to the conclusiveness andincontestability of a Torrens certificate of title do not apply here. The LandRegistration Court has no jurisdiction over non-registerable properties, suchas public navigable rivers which are parts of the public domain, and cannotvalidly adjudge the registration of title in favor of a private applicant.Hence, the judgment of the Court of First Instance of Pampanga as regards theLot No. 2 of Certificate of Title No. 15856 in the name ofpetitioners-appellants may be attacked at any time, either directly orcollaterally, by the State which is not bound by any prescriptive periodprovided for by the Statute of Limitations (Article 1108, par. 4, new CivilCode). The right of reversion or reconveyance to the State of the publicproperties fraudulently registered and which are not capable of privateappropriation or private acquisition does not prescribe.

When it comes to registeredproperties, the jurisdiction of the Secretary of Public Works &Communications under Republic Act 2056 to order the removal or obstruction tonavigation along a public and navigable creek or

river included therein, hasbeen definitely settled. The evidence submitted before the trial court whichwas passed upon by the respondent Court of Appeals shows that Lot No. 2 (PlanPsu 992) of Transfer Certificate of Title No. 15856, is a river of the publicdomain.

The rulingthat a purchaser of a registered property cannot go beyond the record to makeinquiries as to the legality of the title of the registered owner, but may relyon the registry to determine if there is no lien or encumbrances over the same,cannot be availed of as against the law and the accepted principle that riversare parts of the public domain for public use and not capable of privateappropriation or acquisition by prescription.

Part 3

NATURE OF RECLAIMED LANDS

G.R. No. 133250 July 9,2002

FRANCISCO I. CHAVEZ, PETITIONER, VS. PUBLIC ESTATES AUTHORITY AND AMARICOASTAL BAY DEVELOPMENT CORPORATION, RESPONDENTS

FACTS

On November 20, 1973, the government, through theCommissioner of Public Highways, signed a contract with the Construction andDevelopment Corporation of the Philippines (CDCP) to reclaim certain foreshoreand

offshore areas of Manila Bay. On February 4, 1977, then President FerdinandE. Marcos issued Presidential Decree No. 1084 creating PEA - tasked “to reclaim land, including foreshoreand submerged areas,” and “to develop, improve, acquire, x x x lease and sellany and all kinds of lands.” On the same date, then President Marcos issuedPresidential Decree No. 1085 transferring to PEA the “lands reclaimed in theforeshore and offshore of the Manila Bay” under the Manila-Cavite Coastal Roadand Reclamation Project (MCCRRP).

OnJanuary 19, 1988, then President Corazon C. Aquino issued Special Patent No.3517, granting and transferring to PEA “the parcels of land so reclaimed underthe Manila-Cavite Coastal Road and Reclamation Project (MCCRRP) .Subsequently,on April 9, 1988, the Register of Deeds of the Municipality of Parañaque issuedTransfer Certificates of Title in the name of PEA, covering the three reclaimedislands known as the “Freedom Islands” located at the southern portion of theManila-Cavite Coastal Road, Parañaque City. On April 25, 1995, PEA entered intoa Joint Venture Agreement (JVA) with AMARI, a private corporation, to developthe Freedom Islands. PEA and AMARI entered into the JVA through negotiationwithout public bidding. On June 8, 1995, then President Fidel V. Ramos, throughthen Executive Secretary Ruben Torres, approved the JVA.

OnNovember 29, 1996, then Senate President Ernesto Maceda delivered a privilegespeech in the Senate and denounced the JVA as the “grandmother of all scams.”As a result, the Senate Committee on Government Corporations and PublicEnterprises, and the Committee on Accountability of Public Officers andInvestigations, conducted a joint investigation. Among the conclusions of theirreport are: (1) the reclaimed lands PEA seeks to transfer to AMARI under theJVA are lands of the public domain which the government has not classified asalienable lands and therefore PEA cannot alienate these lands; (2) thecertificates of title covering the Freedom Islands are thus void, and (3) theJVA itself is illegal.

OnDecember 5, 1997, then President Fidel V. Ramos issued PresidentialAdministrative Order No. 365 creating a Legal Task Force to conduct a study onthe legality of the JVA. The Legal Task Force upheld the legality of the JVA,contrary to the conclusions reached by the Senate Committees.

On April27, 1998, petitioner Frank I. Chavez (“Petitioner” for brevity) as a taxpayer,filed the instant Petition for Mandamus with Prayer for the Issuance of aWrit of Preliminary Injunction and Temporary Restraining Order. Petitionercontends the government stands to lose billions of pesos in the sale by PEA ofthe reclaimed lands to AMARI. Petitioner prays that PEA publicly disclose theterms of any renegotiation of the JVA, invoking Section 28, Article II, andSection 7, Article III, of the 1987 Constitution on the right of the people toinformation on matters of public concern. Petitioner assails the sale to

AMARIof lands of the public domain as a blatant violation of Section 3, Article XIIof the 1987 Constitution prohibiting the sale of alienable lands of the publicdomain to private corporations. In a Resolution dated March 23, 1999, the Courtgave due course to the petition and required the parties to file theirrespective memoranda.

On March30, 1999, PEA and AMARI signed the Amended Joint Venture Agreement (Amended JVA).On May 28, 1999, the Office of the President under the administration of thenPresident Joseph E. Estrada approved the Amended JVA.

Due tothe approval of the Amended JVA by the Office of the President, petitioner nowprays that on “constitutional and statutory grounds the renegotiated contractbe declared null and void.”

ISSUE

WHETHERTHE Lands reclaimed from foreshore andsubmerged areas also form part of the public domain and are also inalienable

WHETHERTHE STIPULATIONS IN THE AMENDED JOINT VENTURE AGREEMENT FOR THE TRANSFER TOAMARI OF CERTAIN LANDS, RECLAIMED AND STILL TO BE RECLAIMED, VIOLATE THE 1987CONSTITUTION

RULING

The 1987 Constitution, like the 1935 and 1973 Constitutions before it,has adopted the Regalian doctrine. The 1987 Constitution declares that allnatural resources are “owned by the State,” and except foralienable agricultural lands of the public domain, natural resources cannot bealienated.

UnderSection 2, Article XII of the 1987 Constitution, the foreshore and submergedareas of Manila Bay are part of the “lands of the public domain, waters x x xand other natural resources” and consequently “owned by the State.” As such,foreshore and submerged areas “shall not be alienated,” unless they areclassified as “agricultural lands” of the public domain. The mere reclamationof these areas does not convert these inalienable natural resources of theState into alienable or disposable lands of the public domain. There must be alaw or presidential proclamation officially classifying these reclaimed landsas alienable or disposable and open to disposition or concession. Moreover,these reclaimed lands cannot be classified as alienable or disposable if thelaw has reserved them for some public or quasi-public use.

Landsreclaimed by the government are sui generis, not available forsale to private parties unlike other alienable public lands. Reclaimed landsretain their inherent potential as areas for public use or public service.

Clearly,the Amended JVA violates glaringly Sections 2 and 3, Article XII of the 1987Constitution. Under Article 1409 of the Civil Code, contracts whose “object orpurpose is contrary to law,” or whose “object is outside the commerce of men,”are “inexistent and void from the beginning.” The Court must perform its dutyto defend and uphold the Constitution, and therefore declares the Amended JVAnull and void ab initio.

PART 4

VIII. PROCEEDINGS FORREGISTRATION OF LAND

A.       JUDICIAL REGISTRATION

h. Amendments. Need forpublication and notice if change are substantial

            i. DIRECTOR OF LANDS VS. IAC, 219 SCRA 399

G.R. No. 73246     March2, 1993

 

DIRECTOR OF LANDS AND DIRECTOR OF FORESTDEVELOPMENT, petitioners,

 

vs.

 

INTERMEDIATE APPELLATE COURT AND J. ANTONIOARANETA, respondents.

FACTS

The case is aboutthe registration of the Tambac Island in Lingayen Gulf situated in theMunicipality of Bani, Pangasinan. The initial application for registration wasfiled for Pacific Farms, Inc. under the provisions of the Land RegistrationAct, Act No. 496, as amended. The Republic of the Philippines, thru theDirector of Lands opposed the application alleging that the applicant, PacificFarms, Inc. does not possess a fee simple title to the land nor did itspredecessors possess the land for at least thirty (30) years immediatelypreceding the filing of application. The opposition likewise specificallyalleged that the applicant is a private corporation disqualified under the(1973) new Philippine Constitution from acquiring alienable lands of the publicdomain citing Section 11, Article 14. 2 The Director of Forest Development alsoentered its opposition alleging that the land is within the unclassified publicland and, hence, inalienable. Other private parties also filed theiroppositions, but were subsequently withdrawn.

In an amendedapplication, Pacific Farms, Inc. filed a manifestation-motion to change theapplicant from Pacific Farms, Inc. to J. Antonio Araneta. Despite the supposedamendment, there was no republication.Evidence presented by the applicantinclude the testimony of Placido Orlando, fishery guard of Pacific Farms, Inc.,who said he has known the disputed land since he attained the age of reason forsome forty (40) years now; that when he first came to know the property it wasthen owned by and in the possession of Paulino Castelo, Juan Ambrosio and JulioCastelo, and later on the whole island was bought by Atty. Vicente Castelo whoin turn sold it to J. Antonio Araneta.

While this case ispending, respondent filed an Omnibus Motion for Substitution of privaterespondent. Apparently, Antonio Araneta had assigned his rights to and interestin Tambac Island to Amancio R. Garcia who in turn assigned his rights and interest in the same property toJohnny A. Khonghun whose nationality was not alleged in the pleadings.

On October 4, 1979,the trial court rendered a decision adjudicating the subject property to J.Antonio Araneta. On appeal to the then Intermediate Appellate Court, thedecision of the lower court was affirmed on December 12, 1985.

ISSUE

Whether the lowercourt erred in adjudicating the land to the applicant under the provisions ofPresidential Decree No. 1529, otherwise known as the Property RegistrationDecree, despite absence of any specific invocation of this law in the originaland amended application.

RULING

The Court agree with petitioners that the amendment ofthe application from the name of Pacific Farms Inc., as applicant, to the nameof J. Antonio Araneta Inc., was a mere attempt to evade disqualification. The1973 even the 1987 Philippine Constitution prohibits private corporations orassociations from holding alienable lands of the public domain except by lease.Apparently realizing such prohibition, respondent amended its application toconform with the mandates of the law. However, the Court did not agree withpetitioners' position that the absence of republication of an amendedapplication for registration is a jurisdictional flaw. Amendments to theapplication may be due to change in parties or substantial change in theboundaries or increase in the area of the land applied for. Neither the LandRegistration Act, as amended, nor Presidential Decree No. 1529, otherwise knownas the Property Registration Decree, requires republication and registrationmay be allowed by the court at any stage of the proceeding upon just and reasonableterms. On the other hand, republication is required if the amendment is due tosubstantial change in the boundaries or increase in the area of the landapplied for.

PART 5

THE POWER TO CLASSIFY OR RECLASSIFY PUBLIC LANDSINTO ALIENABLE AND DISPOSABLE LANDS BELONG TO THE EXECUTIVE BRANCH OF THEGOVERNMENT

G.R. No. L-66866 June18, 1987

 

REPUBLIC OF THE PHILIPPINES, petitioner,

vs.

MINDA DE PORKAN, SADIN MARAUG, GORGONIO BERMUDEZ,LOLITA MACATINDOG, MEDORI DE PORKAN, JUAN ARANGALI, ANTONINA ESTARES, REGISTEROF DEEDS OF DAVAO DEL NORTE and the INTERMEDIATE APPELLATE COURT [Fourth CivilCases Division], respondents.

FACTS

The family of Sadin dePorkan, father of Medori and Macampon de Porkan, both native Muslims of La Paz,Carmen, Davao del Norte, had been in actual possession as owner since theSpanish colonial period of a tract of land planted with coconuts situated insaid municipality. During the Tagum Cadastral Survey of July 22, 1937, thistract of land, were respectively allocated to Medori and Macampon de Porkan,the predecessors-in-interest of the private respondents. For the succeedingyears, there had been changes in the ownership of the land within the de Porkanfamily which was approved by the government until a problem arose at the timeof ownership of Minda de Porkan and Lolita Macatindog.

On November 26, 1969, thePhilippine Fisheries Commissioner ordered the Regional Director at Davao Cityto investigate the conflict of fishpond applications between Viola C. Azurinand Moonyeen R. Beleno. On May 26, 1972,while the fishpond conflict case was pending investigation by the PhilippineFisheries Commission [Davao City], Viola C. Azurin filed with the Bureau ofLands a complaint for the correction,amendment or cancellation of Homestead Patent of Minda de Porkan and FreePatent of Lolita Macatindog situated at La Paz, Panabo [now Carmen], Davao delNorte, alleging, among others, that the patentees secured their patents andtitles through fraud, misrepresentation and illegal machinations.

Minda de Porkan andLolita Macatindog similarly claimed in their separate answers that they werethe ones who first protested, along with their Muslim relatives, the intrusionof the families of the Azurins, Rodriguezes, and Belenos

into their "ancestrallands, which from time immemorial had been occupied and cultivated by theirancestors and predecessors-in-interest.

After a joint hearing,the Court of First Instance of Davao del Norte, Branch [Tagum] rendered itsdecision 41 on November 18, 1980 dismissing the complaints for cancellation oftitles and upholding the validity of the patents/titles of Lolita Macatindogand Minda de Porkan, as well as the titles of their transferees co-defendantsJuan Arangali and Gorgonio Bermudez, who were adjudged to be innocentpurchasers for value and in good faith.

The Solicitor Generalappealed the decision of the lower court to the then Intermediate AppellateCourt, now Court of Appeals. On February 29, 1984, the Intermediate AppellateCourt, now Court of Appeals, affirmed the decision of the lower court.

ISSUE

            Whetherthe executive branch of government has the power to classify and reclassifypublic lands into alienable and disposable lands

RULING

It is significant to notethat the tract of public land then possessed, occupied, developed and plantedto coconuts by the family of Sadin de Porkan and his predecessors-in-interest,all native Muslims of La Paz, Carmen [formerly Panabo], Davao del Norte, which,by virtue of its being part of the unregistered lands, was included in theTagum Cadastral Survey of July 22, 1937 and formed part of the disposable oralienable agricultural lands of the public domain referred to under Section 6,par. [a] in relation to Section 9, par. [a] of the Public Land Act [C.A. 141,as amended]. The nature and character of said tract of public land, moreparticularly Lot No. 1099, as one found inside an "agriculturalzone", and that of Lot No. 1546, as one suitable for rice cultivation,which were categorically stated in the separate investigation reports in 1953of Vicente J. Villena, junior public land inspector of the Bureau

of Lands[Davao] is binding on the courts inasmuch as it is the exclusive prerogative ofthe Executive Department of the Government to classify public lands .

In the case of Mapa vs.Insular Government, 10 Phil. 175, this court said that the phrase 'agriculturallands' as used in Act No. 926 means those public lands acquired from Spainwhich are not timber or mineral. Whatever may have been the meaning of the term'forestry' under the Spanish law, the Act of Congress of July 1st, 1902,classifies the public lands in the Philippine Islands as timber, mineral oragricultural lands, and all public lands that are not timber or mineral landsare necessarily agricultural public lands, whether they are used as nipaswamps, manglares, fisheries or ordinary farm lands.

Since the disputed tractof public land is neither timber nor mineral lands, the same is alienable oropen to disposition as public agricultural lands, under Section 11, C.A. 141thru homestead settlement or free patent.

Where, as in the instantcase, the possession of a public land by Sadin de Porkan, father of Medori dePorkan, and their predecessors-in-interest who were native Muslims of la Paz,Panabo [now Carmen], Davao del Norte, dates back to the time of the Spanishcolonial period, such possession of the said tract of public land has attainedthe character and duration prescribed by law as the equivalent of an expressgrant from the Government. The mandate of the law itself is that the possessors"shall be conclusively presumed to have performed all the conditionsessential to a Government grant and shall be entitled to a certificate oftitle" and by legal fiction, the land ceases to be public and thus becomesprivate land.

Part 6

TAX DECLARATION

G.R. No. 177797, December 04, 2008

SPS. PEDRO TAN AND NENA ACERO TAN, PETITIONERS,

VS.

REPUBLIC OF THE PHILIPPINES, RESPONDENT

FACTS

The spouses Pedro Tan and Nena AceroTan were natural-born Filipino citizens, who became Australian citizens on 9February 1984. They seek to have thesubject property registered in their names. The subject property was declaredalienable and disposable on 31 December 1925, as established by a Certificationdated 14 August 2000 issued by the Department of Environment and NaturalResources (DENR), Community Environment and Natural Resources Office (CENRO),Cagayan de Oro City. Prior to the spouses Tan, the subject property was in thepossession of Lucio and Juanito Neri and their

respective spouses. Lucio and Juanito Neri had declared thesubject property for taxation purposes in their names.The spouses Tan acquiredthe subject property from Lucio and Juanito Neri and their spouses by virtue ofa duly notarized Deed of Sale of Unregistered Real Estate Property dated 26June 1970. The spouses Tan tookimmediate possession of the subject property on which they planted rubber,gemelina, and other fruit-bearing trees. They declared the subject property for taxation purposes in their names andpaid realty taxes thereon.

However, a certain Patermateo Casiño(Casiño) claimed a portion of the subject property, prompting the spouses Tanto file a Complaint for Quieting of Title against him before the RTC of Cagayande Oro City. On 29 August 1989, the RTC rendered a Decision favoring thespouses Tan and declaring their title to the subject property thus"quieted." Casiño appealed thesaid RTC Decision to the Court of Appeals . In a Resolution the appellate courtdismissed for lack of interest to prosecute. Casiño elevated his case to this Court via a Petition for Review onCertiorari. In a Resolution dated 13 March 1991  the Court denied Casiño's Petition for beinginsufficient in form and substance. Thesaid Resolution became final and executory on 3 June 1991. Refusing to give up,Casiño filed an Application for Free Patent on the subject property before theBureau of Lands. On 8 December 1999, Casiño's application was ordered cancelledby Officer Ruth G. Sabijon of DENR-CENRO, Cagayan de Oro City, upon the requestof herein petitioner Pedro Tan, the declared owner of the subject propertypursuant to the 29 August 1989 Decision of the RTC. In 2000, the spouses Tanfiled their Application for Registration of Title to the subject propertybefore the RTC of Cagayan de Oro City. The application of the spouses Taninvoked the provisions of Act No. 496 and/or Section 48 of Commonwealth Act No.141,as amended. In compliance with therequest of the Land Registration Authority (LRA) dated 29 August 2000, thespouses Tan filed on 5 October 2000 an Amended Application for Registration ofTitle to the subject property.

The Office of the Solicitor General(OSG) entered its appearance on behalf of the Republic, but failed to submit awritten opposition to the application of the spouses Tan.

When no opposition to theapplication of the spouses Tan was filed by the time of the initial hearing theRTC issued on 23 April 2001 an order of general default, except as against theRepublic. Thereafter, the spouses Tanwere allowed to present their evidence ex-parte.

After the establishment of thejurisdictional facts, the RTC heard the testimony of John B. Acero , nephew andlone witness of the spouses Tan. After Acero's testimony, the spouses Tanalready made a formal offer of evidence, which was admitted by the court a quo.On 9 May 2001, the RTC rendered a Decision granting the application of thespouses Tan. The Republic appealed the RTC Decision to the Court of Appeals.

On 28 February 2006, the Court ofAppeals rendered a Decision granting the appeal of the Republic, and reversingand setting aside the 9 May 2001 Decision of the RTC on the ground that thespouses Tan failed to comply with Section 48(b) of Commonwealth Act No. 141,otherwise known as the Public Land Act, as amended by Presidential Decree No.1073, which requires possession of the subject property to start on or prior to12 June 1945. Hence, the appellate court ordered the spouses Tan to return thesubject property to the Republic.

The spouses Tan filed a Motion forReconsideration of the foregoing Decision of the Court of Appeals. To refute the finding of the appellate courtthat they and their predecessors-in-interest did not possess the subjectproperty by 12 June 1945 or earlier, the spouses Tan attached to their Motion acopy of Tax Declaration No. 4627 covering the subject property issued in 1948in the name of their predecessor-in-interest, Lucio Neri. They called attention to the statement in TaxDeclaration No. 4627 that it cancelled Tax Declaration No. 2948. Unfortunately, no copy of Tax Declaration No.2948 was available even in the Office of the Archive of the Province of MisamisOriental. The spouses Tan asserted that judicial notice may be taken of thefact that land assessment is revised by the government every four years; andsince Tax Declaration No. 4627 was issued in the year 1948, it can bepresupposed that Tax Declaration No. 2948 was issued in the year 1944.

The Court of Appeals denied the Motion for Reconsiderationof the spouses Tan in a Resolution dated 12 April 2007.

Theearliest evidence of possession and occupation of the subject property can betraced back to a tax declaration issued in the name of theirpredecessors-in-interest only in 1952. However,the spouses Tan are now asking the kind

indulgence of this Court to take intoaccount Tax Declaration No. 4627 issued in 1948, which they had attached totheir Motion for Reconsideration before the Court of Appeals but which theappellate court refused to consider. Just as they had argued before the Court of Appeals, the spouses Tanpoint out that Tax Declaration No. 4627 was not newly issued but cancelled TaxDeclaration No. 2948; and should the Court take judicial notice of the factthat tax assessments are revised every four years, then Tax Declaration No.2948 covering the subject property was issued as early as 1944.

ISSUE

WHETHER TAXDECLARATIONS AND RECEIPTS ARE CONCLUSIVE EVIDENCE OF OWNERSHIP

RULING

Tax declarations and receipts are not conclusive evidence ofownership. At most, they constitute mere prima facie proofs of ownershipof the property for which taxes have been paid. In the absence of actual,public and adverse possession, the declaration of the land for tax purposesdoes not prove ownership.They may be good supporting or collaboratingevidence together with other acts of possession and ownership; but bythemselves, tax declarations are inadequate to establish possession of theproperty in the nature and for the period required by statute for acquiringimperfect or incomplete title to the land.

The spouses Tan purchased the subject property and came intopossession of the same only in 1970. To justify their application forregistration of title, they had to tack their possession of the subjectproperty to that of their predecessors-in-interest. While the spouses Tan undoubtedly possessedand occupied the subject property openly, continuously, exclusively andnotoriously, by immediately introducing improvements on the said property, inaddition to declaring the same and paying realty tax thereon; in contrast,there was a dearth of evidence that their predecessors-in-interest possessedand occupied the subject property in the same manner. The possession and occupation of the subjectproperty by the predecessors-in-interest of the spouses Tan were evidenced onlyby the tax declarations in the names of the former, the earliest of which, TaxDeclaration No. 4627, having

been issued only in 1948. No other evidence was presented by thespouses Tan to show specific acts of ownership exercised by theirpredecessors-in-interest over the subject property which may date back to 12June 1945 or earlier.

For failure of the Spouses Tan to satisfy the requirementsprescribed by Section 48(b) of the Public Land Act, as amended, this Court hasno other option but to deny their application for judicial confirmation andregistration of their title to the subject property.

Part 7

DECREE OF REGISTRATION

G.R. No.123346, December 14, 2007

MANOTOK REALTY, INC. AND MANOTOK ESTATE CORPORATION,PETITIONERS, VS. CLT REALTY DEVELOPMENT CORPORATION, RESPONDENT

FACTS

            On 10August 1992, CLT Realty Development Corporation (CLT) sought to recover fromManotok Realty, Inc. and Manotok Estate Corporation (Manotoks) the possessionof Lot 26 of the Maysilo Estate in an action filed before the Regional TrialCourt of Caloocan City. CLT’s claim was anchored on Transfer Certificate ofTitle (TCT) issued in its name by the Caloocan City Register of Deeds, whichtitle in turn was derived from Estelita Hipolito (Hipolito) by virtue of a Deedof Sale with Real Estate Mortgage dated 10 December 1988. Hipolito’s titleemanated from Jose Dimson’s (Dimson) TCT , a title issued pursuant to an orderof the Court of First Instance (CFI) of Caloocan City. Dimson’s title appearsto have been sourced from OCT No. 994.

For their part, the Manotoks challenged the validity of thetitle relied on by CLT, claiming that Dimson’s title, the proximate source ofCLT’s title, was irregularly issued and, hence, the same and subsequent titlesflowing therefrom are likewise void. The Manotoks asserted their ownership overLot 26 and claimed that they derived it from several awardees and/or vendees ofthe National Housing Authority. The Manotok title likewise traced as itsprimary source OCT No. 994 which, on 9 September 1918, was transferred toAlejandro Ruiz and Mariano Leuterio who had previously acquired the property on21 August 1918 by virtue of an “Escritura de Venta” executed by Don TomasArguelles and Don Enrique Llopis. On 3 March 1920, Ruiz and Leuterio sold theproperty to Francisco Gonzalez who held title thereto until 22 August 1938 whenthe property was transferred to Jose Leon Gonzalez, Consuelo Susana Gonzalez,Juana Francisca Gonzalez, Maria Clara Gonzalez, Francisco Felipe Gonzalez andConcepcion Maria Gonzalez under TCT No. 35486. The lot was then, per annotationdated 21 November 1946, subdivided into seven (7) parcels each in the name ofeach of the Gonzalezes.

The trial court, ruling for CLT, adopted the factualfindings and conclusions arrived at by the majority commissioners appointed toresolve the conflict of titles. It was established that the entire MaysiloEstate was registered under Act No. 496 by virtue of which OCT No. 994 wasissued by the Register of Deeds of Rizal;that Lot 26 was transferred to CLT byHipolito whose title was derived from the Dimson title and that on the basis ofthe technical descriptions of the property appearing in the Manotok titles, thelatter’s property indeed encroached on the property described in CLT’s title.

The Manotoks appealed to the Court of Appeals, whichaffirmed the decision of the trial court. Their motion for reconsiderationhaving been denied, they filed a petition for review with the Supreme Court,ascribing error to the appellate court in upholding the trial court’s decisionwhich decided the case on the basis of the majority commissioners’ report andoverlooked relevant facts in the minority commissioner’s report.

ISSUE

whether the titles issued in thename of CLT IS valid

RULING

With respect to G.R. No. 123346, the Court upheld thevalidity of the trial court’s adoption of the commissioners’ majority report aspart of the decision. The Court pointed out that the titles of respondents inall three cases were derived from OCT No. 994 of the Registry of Deeds ofCaloocan City registered on 19 April 1917. The Manotoks filed their respectivemotions for reconsideration. On 5 June 2006, the cases were elevated to theCourt en banc. In the Manotok petition, CLT hadoriginally filed a complaint for annulment of the titles in the name of theManotoks, alleging that it was the registered owner of Lot 26 of the MaysiloEstate. It is evident from all three titles CCLT’s, Hipolito’s andDimson’s—that the properties they purport to cover were “originally registeredon the 19th day April 1917 in the Registration Book of the Office of theRegister of Deeds of Rizal.” As earlier established, there is no such OCT No.994 originally registered on 19 April 1917. None of these three titles can beaccorded recognition simply because the original title commonly referred totherein never existed. To conclude otherwise would constitute deliberatedisregard of the truth. These titles could be affirmed only if it can be proventhat OCT No. 994 registered on 19 April 1917 had actually existed. CLT and theDimsons were given the opportunity to submit such proof before this Court, butthey did not. In fact, CLT has specifically manifested that the OCT No. 994they concede as true is also the one which the Office of Solicitor Generalsubmitted as true, and that is OCT No. 994 issued on 3 May 1917.Given

thisessential clarification, there is no sense in affirming the 2005 Decision whichsustained the complaints for annulment of title and/or recovery of possessionfiled by CLT and the Dimson when their causes of action are both founded on aninexistent mother title.

From these premises, the Court is able to make the followingbinding conclusions. First, there is only one OCT No. 994. As it appears on therecord, that mother title was received for transcription by the Register ofDeeds on 3 May 1917, and that should be the date which should be reckoned asthe date of registration of the title. It may also be acknowledged, as appearson the title, that OCT No. 994 resulted from the issuance of the decree ofregistration on 17 April 1917, although such date cannot be considered as thedate of the title or the date when the title took effect. Second,any title thattraces its source to OCT No. 994 dated 17 April 1917 is void, for such mothertitle is inexistent. This error alone is, in fact, sufficient to invalidate theDimson and CLT claims over the subject property if singular reliance is placedby them on the dates appearing on their respective titles.

The land becomes a registered landonly upon the transcription of the decree in the original registration book bythe register of deeds, the date and time of such transcription being set forthin the process and certified to at the foot of each entry or certificate oftitle. The issuance of the original and owner’s duplicate certificates arebasic for the valid existence of the title. Issuance of additional copies arepermissive and their non-existence does not affect the status of title. Acertificate of title is deemed as regularly issued with the issuance of theoriginal copy and owner’s duplicate

land of the private respondents.On July 29, 2002, the COSLAP issued a writ of execution of its decision, ordering the DENR Sec. to implement the August 3, 1998 decision as affirmed by SC. DENR Sec. ordered the DENR Director of Region 12 toconduct a review and investigation of FLGLA No. 542. The latter found violations by petitioner of the terms of the FLGLA. On August 15, 2002, Sec. Alvarez cancelled FLGLA No. 542. Petitioner’s MR was denied.On November 26, 2002, Community Environment and Natural Resources Officer (CENRO) Andrew B. Patricio Jr. sent a letter to petitioner, advising him to vacate and remove all improvements in the area within 10 days from receipt of the letter.On November 27, 2002, CENRO Patricio issued an Installation Order, which directed the immediate installation and occupation of the area by the private respondents indigenous communities.Upon appeal, the CA dismissed petitioner’s appeal, holding that issue was already decided by SC in GR No. G.R. No. 145838. Hence, this instant petition via Rule 45.Issue: w/n

petitioner has residual rights over subject property until the expiration of FLGLA 542 (Dec�

PartII

1.Almirol v. Register of Deeds

22SCRA 1152   March 20, 1968

FACTS:

               On June 28, 1961,TeodoroAlmirol purchased from Arcenio Abalo a parcel of land covered byoriginalcertificate of title P-1237 in the name of Arcenio Abalo and hisdeceased wife,Nicolasa M. Abalo. Sometime in May 1962 Almirol went to theoffice of the Registerof Deeds to register the deed of sale and to secure inhis name a transfercertificate of title. Registration was refused by theRegister of Deeds becauseaccording to the Registrar, the land was a conjugalproperty and therefore, bothspouses must sign the Deed of Sale. Since, as inthis case, the wife has alreadydied when the sale was made, the survivinghusband cannot dispose of the wholeproperty.

Issue:Whether or not the Registrar has the power to determine thevalidity of adocument sought to be registered.

Held:

               No, whether adocument is validor invalid, the Register of deeds is not granted theauthority to determine;this function belongs properly to a court of competentjurisdiction. Althoughthe reasons relied upon by the Registrar evince asincere desire on his part tomaintain inviolate the law on succession andtransmission of rights over realproperties, these do not constitute legalgrounds for his refusal to registerthe deed. Whether the document is invalid,frivolous or intended to harass, itis not the duty of a Register of Deeds todecide, but a court of competentjurisdiction. (Gabriel vs. Register of Deedsof Rizal, et al., L-17956, Sept.30, 1953).

               Hence,the refusal of theregistration of document was invalid.

Balbin vs. Register of DeedsG.R. L-20611 May 8, 1969Makalintal, J.Facts:On November 15, 1961 petitioners presented to the register of deeds of Ilocos Sur a duplicate copy of the registered owner's certificate of title (OCT No. 548) and an instrument entitled "Deed of Donation inter-vivos," with the request that the same

be annotated on the title. Under the terms of the instrument sought to be annotated one Cornelio Balbin, registered owner of the parcel of land described in OCT No. 548, appears to have donated inter-vivos an undivided two-thirds (²/³) portion thereof in favor of petitioners. The entire area of the land is 11.2225 hectares.The register of deeds denied the requested annotation for being "legally defective or otherwise not sufficient in law." It appears that previously annotated in the memorandum of encumbrances on the certificate are three separate sales of undivided portions of the land earlier executed by Cornelio Balbin in favor of three different buyers. The final part of the annotations referring to the abovementioned sales contains an additional memorandum stating that "three co-owner's duplicate certificates of title No. 548 have been issued (by the register of deeds of Ilocos Sur) in the name of Florentino Gabayan, Roberto Bravo and Juana Gabayan upon verbal request of Mr. Andres Cabeldo, Notary Public of Caoayan, I. Sur, for and in the name of the vendees, this 5th day of January, 1956 at Vigan, I. Sur." Mainly because these three other co-owner's copies of the certificate of title No. 548 had not been presented by petitioners, the Register of Deeds refused to make the requested annotation.Issue: Whether the refusal of the Register of Deeds to make the annotation is proper.Held: Yes. Section 55 of Act 496, which provides that "the production of the owner's duplicate certificate of title whenever any voluntary instrument is presented for registration shall be conclusive authority from the registered owner to the register of deeds to make a memorandum of registration in accordance with such instrument," obviously assumes that there is only one duplicate copy of the title in question, namely, that of the registered owner himself, such that its production whenever a voluntary instrument is presented constitutes sufficient authority from him for the register of deeds to make the corresponding memorandum of registration. There being several copies of the same title in existence, it is easy to see how their integrity may be adversely affected if an encumbrance, or an outright conveyance, is annotated on one copy and not on the others. The law itself refers to every copy authorized to be issued as a duplicate of the original, which means that both must contain identical entries of the transactions, particularly voluntary ones, affecting the land covered by the title. If this were not so, if different copies were permitted to carry differing annotations, the whole system of Torrens registration would cease to be reliable.

3(Gina)

MELITONGALLARDO and TERESA VILLANUEVA

vs.

HONORABLEINTERMEDIATE APPELLATE COURT

G.R. No.L-67742 October 29, 1987

PARAS,J.:

Facts:

Thesubject matter of this controversy involves a parcel of land situated inCavinti, Laguna consisting of 81,300 square meters, more or less, initiallycovered by an original Certificate of Title No. 2262, issued on April 2, 1924owned and registered in the name of the late Pedro Villanueva pursuant toDecree No. 150562 issued in L.R.C. Cadastral Record No. 136, Cad. Case No. 1.

OnAugust 10, 1937, petitioner claimed that the aforestated land was sold to themin a private document, an unnotarized deed of sale written in Tagalog that wasallegedly signed by the late Pedro Villanueva conveying and transfering theproperty in question in favor of the petitioners.

Subsequently,the Original Certificate of Title was cancelled on the basis of the privatedocument of sale and a new certificate of title was issued in the name of thepetitioners covered by Transfer Certificate of Title No. RT- 6293 (No. 23350)on January 4, 1944.

Duringthe Second World War, the records as well as the Office of the Register ofDeeds of Laguna, where the original of their new transfer certificate of titlewas kept, were completely burned. Accordingly, by virtue of an Affidavit ofReconstitution dated December 2, 1958 and upon presentation of the Owner'sDuplicate Certificate of Title, the title was administratively reconstitutedand the Register of Deeds of Laguna issued Transfer Certificate of Title No.RT-6293 (No. 23350) in the name of the petitioners.

However,when private respondent Marta Villanueva vda. de Agana refused to sign anAffidavit of Quit-claim, petitioners instituted court suit against the privaterespondent and her husband, Dr. Marcelo S. Agana, Sr. by filing a complaint forQuieting of Title and Damages with the Court of First Instance of Laguna

onFebruary 3, 1977, demanding that their title over the questioned land befortified by a declaration of ownership in their favor and avoiding theaf/recited Deed of Conveyance and Release of Claim.

Accordingly,private respondents in their answer countered that the Deed of Sale in Tagalogand petitioners' title over the land be declared void ab initio, among other demands.

The trialcourt found that said private document was null and void and that it was signedby somebody else not Pedro Villanueva. Such findings of fact besides beingbased on the records, were sustained by the Court of Appeals The IntermediateAppellate Court, on May 22, 1984, affirmed in toto the decision of the trialcourt.

Issue:

whether or not there was a validreconstitution of Transfer Certificate of Title No. RT-6293 (No. 23350) issuedin the names of petitioners.

Held:

No. True,as argued by appellants, a private conveyance of registered property is validas between the parties. However, the only right the vendee of registeredproperty in a private document is to compel through court processes the vendorto execute a deed of conveyance sufficient in law for purposes of registration.Plaintiffs-appellants' reliance on Article 1356 of the Civil Code isunfortunate. The general rule enunciated in said Art. 1356 is that contractsare obligatory, in whatever form they may have been entered, provided all theessential requisites for their validity are present. The next sentence providesthe exception, requiring a contract to be in some form when the law so requiresfor validity or enforceability. Said law is Section 127 of Act 496 whichrequires, among other things, that the conveyance be executed "before thejudge of a court of record or clerk of a court of record or a notary public ora justice of the peace, who shall certify such acknowledgment substantially inform next hereinafter stated."

Such lawwas violated in this case. The action of the Register of Deeds of Laguna inallowing the registration of the private deed of sale was unauthorized and didnot lend a bit of validity to the defective private document of sale.

Uponconsideration of the facts and circumstances surrounding the execution of theassailed document, the trial court found that said private document was nulland void and that it was signed by somebody else not Pedro Villanueva. Suchfindings of fact besides being based on the records, were sustained by theCourt of Appeals.

In thecase at bar, it will be noted that what transpired was an administrativereconstitution, essentially ex-parte and without notice, thereby lendingcredence to the claim that private respondent Marta Agana was unaware of suchreconstitution and possession until she discovered the same in the Office ofthe Register of Deeds in 1976. As such it cannot be claimed that she slept onher right as from that time on, it is

undeniablethat she filed her adverse claim on the said lot.

4(Zhon)

Cruz vs. DENR Sec., G.R. No. 135385, Dec. 6, 2000

Facts:

This is a petition assailing the constitutionality of the following provisions of Republic Act No. 8371 (R.A. 8371), otherwise known as the Indigenous Peoples Rights Act of 1997 (IPRA), on the ground that they amount to an unlawful deprivation of the States ownership over lands of the public domain as well as minerals and other natural resources therein, in violation of the regalian doctrine embodied in Section 2, Article XII of the Constitution:

(1) Section 3(a) which defines the extent and coverage of ancestral domains, and Section 3(b) which, in turn, defines ancestral lands;

(2) Section 5, in relation to section 3(a), which provides that ancestral domains including inalienable public lands, bodies of water, mineral and other resources found within ancestral domains are private but community property of the indigenous peoples;

(3) Section 6 in relation to section 3(a) and 3(b) which defines the composition of ancestral domains andancestral lands;

(4) Section 7 which recognizes and enumerates the rights of the indigenous peoples over the ancestral domains;

(5) Section 8 which recognizes and enumerates the rights of the indigenous peoples over the ancestral lands;

(6) Section 57 which provides for priority rights of the indigenous peoples in the harvesting, extraction,development or exploration of minerals and other natural resources within the areas claimed to be their ancestral domains, and the right to enter into agreements with nonindigenous peoples for the development and utilization of natural resources therein for a period not exceeding 25 years, renewable for not more than 25 years; and

(7) Section 58 which gives the indigenous peoples the responsibility to maintain, develop, protect and conserve the ancestral domains and portions thereof which are found to be necessary for critical watersheds, mangroves, wildlife sanctuaries, wilderness, protected areas, forest cover or reforestation.Petitioners also content that, by providing for an all-encompassing definition of ancestral domains and ancestral lands which might even include private lands found within said areas, Sections 3(a) and 3(b) violate the rights of private landowners.

In addition, petitioners question the provisions of the IPRA defining the powers and jurisdiction of the NCIP and making customary law applicable to the settlement of disputes involving ancestral domains and ancestral lands on the ground that these provisions violate the due process clause of the Constitution.

Finally, petitioners assail the validity of Rule VII, Part II, Section 1 of the NCIP Administrative Order No. 1, series of 1998, which provides that the administrative relationship of the NCIP to the Office of the President is characterized as a lateral but autonomous relationship for purposes of policy and program coordination. They contend that said Rule infringes upon thePresidents power of control over executive departments under Section 17,Article VII of the Constitution.

Issue: w/n the assailed provisions are constitutional

Held:

Seven (7) voted to dismiss the petition. Justice Kapunan filed anopinion, which the Chief Justice and Justices Bellosillo, Quisumbing, andSantiago join, sustaining the validity of the challenged provisions of R.A.8371. Justice Puno also filed a separate opinion sustaining all challengedprovisions of the law with the exception of Section 1, Part II, Rule III of NCIPAdministrative Order No. 1, series of 1998, the Rules and RegulationsImplementing the IPRA, and Section 57 of the IPRA which he contendsshould be interpreted as dealing with the large-scale exploitation of naturalresources and should be read in conjunction with Section 2, Article XII of the1987 Constitution. On the other hand, Justice Mendoza voted to dismiss thepetition solely on the ground that it does not raise a justiciable controversyand petitioners do not have standing to question the constitutionality of R.A.8371.

Seven (7) other members of the Court voted to grant the petition.Justice Panganiban filed a separate opinion expressing the view thatSections 3 (a)(b), 5, 6, 7 (a)(b), 8, and related provisions of R.A. 8371 areunconstitutional. He reserves judgment on the constitutionality of Sections58, 59, 65, and 66 of the law, which he believes must await the filing ofspecific cases by those whose rights may have been violated by the IPRA.Justice Vitug also filed a separate opinion expressing the view that Sections3(a), 7, and 57 of R.A. 8371 are unconstitutional. Justices Melo, Pardo,Buena, Gonzaga-Reyes, and De Leon join in the separate opinions of JusticesPanganiban and Vitug.As the votes were equally divided (7 to 7) and the necessary majoritywas not obtained, the case was redeliberated upon. However, afterredeliberation, the voting remained the same.Accordingly, pursuant to Rule56, Section 7 of the Rules of Civil Procedure, the petition is DISMISSED.

5(May Ann)

6(Rhea B.)

CANETE V. GENUINO ICE CO. INC.,GR No. 154080 January 22, 2008Facts:Petitioners filed a complaint and an amended complaint for cancellation of title to property covered by several TCTs for being spurious, fictitious and issued under mysterious circumstances considering that the holders thereof and their predecessors-in-interest were never in actual, adverse, and physical possession of the property rendering them ineligible to acquire title under the Friar Lands Act. They also seek to annul OCT No. 614 from which the foregoing TCTs originated of were derived.

The amended complaint alleged that the plaintiffs and their predecessors-in-interest are among those in actual, adverse, peaceful, and continuous possession in the concept of owner of unregistered parcels of land in Sitio Mabilog, Quezon City. And that the real property in question is a portion of the friar land known as the “Piedad Estate”, which is intended for distribution among the bona fide occupants thereof pursuant to the Friar Lands Act.Respondent, sought the dismissal of the case on the ground that it fails to state a cause of action because petitioners are not real parties-in-interest, that no relief may be granted as a matter of law, and that petitioners failed to exhaust all administrative remedies. The motion to dismiss was denied. The respondents filed a petition for certiorari to the CA which granted the petition and dismissed the amended complain of the petitioners.Issue:Whether the CA acted with grave abuse of discretion in granting the certiorari and dismissing the complaintRuling:No. The complaint and amended complaint failed to state the “ultimate fact” which are essential facts constituting the plaintiff’s cause of action. The plaintiffs gave only an incomplete narration of facts unsupported by documentary or other exhibits, and the allegations are mere conclusions of law also, the allegations of fraud are not specific and were not substantiated.The initial claim that OCT 164 of which all the other subject titles are derivatives is null and void has been proven wrong as held in previous cases (Pinlac). It has been found that OCT 614 did legally exist and was previously issued in the name of the Philippine Government in 1910. An Ad Hoc Committee of the then Ministry of Natural Resources specifically tasked to investigate the historical background of the Piedad Estate, found that as early as prior to the Second World War, all lots in the Piedad Estate had already been disposed of. The Piedad Estate has been placed under the Torrens system which means that all lots therein are titled. Also, as held in the Balicudiong case one who acquired title under the Friar Land Act, as well as his successors-in-interest, may not claim successional rights to purchase by reason of occupation from time immemorial unless it is shown that their predecessors-in-interest were actual settlers and occupants at the time said land were acquired by the government. Also, the plaintiffs did not pray to be declared owners of the subject property-despite their alleged adverse possession-but only to be adjudged as the bona fide occupants thereof, conceding to the State’s ownership of the property. Being so, they are not real parties in interest for the purpose of maintaining a suit for cancellation of the subject titles. Their interest is mere expectancy based on the probability that the government would give them preference as buyers or lessees of the subject lands. On real-parties in interest may file for the cancellation of title of property and not one whose interest is based on mere expectancy.

7(Angel)

8.

G.R. No. 66807. January 26, 1989

Republic v. Alagad

Facts:

On October 11, 1951, defendants filed an application for registration of their title over a parcel of land situated at Linga, Pila, Laguna, which was divided into two parcels, namely, Lot 1 and Lot 2.

The Republic opposed the application on the stereo-typed ground that applicants and their predecessors have not been in possession of the land openly, continuously, publicly and adversely under a bona fide claim of ownership since July 26, 1894 and the land has not ceased to be a part of the public domain.

On January 16, 1956, by virtue of a final judgment supplemented by orders, the Alagads were declared owners of Lot 1 and the remaining portion, or Lot 2, was declared public land. According to the trial court, the aforementioned parcel of land is a portion of the public domain belonging to the Republic of the Philippines, and hence, available disposition and registration.

The Republic filed a petition for “annulment of title and reversion, insofar as the 1.42 hectare northwestern portion on end of Lot 1 is concerned, contending that such is foreshore land, and had since time immemorial, been foreshore land reached and covered by the waters of the Laguna de Bay.

Issue:

Whether properties in dispute are capable of private appropriation?

Held:

According to the trial court, the aforementioned parcel of land is a portion of the public domain belonging to the Republic of the Philippines, and hence, available for disposition and registration. As we have pointed out, the Government holds otherwise, and that as foreshore land, it is not registerable.

The law provides that property of the public dominion refers to things held by the State by regalian right. They are things res publicae in nature and hence, incapable

of private appropriation. Thus, under the present Constitution, with the exception of agricultural lands, all other natural resources shall not be alienated.

Thus, assuming that the properties are foreshore in nature, the Republic has legitimate reason to demand reconveyance.

9(Mike Jayson)

10(Lou)

RomeoMARTINEZ and Leonor Suarez v. COURT of APPEALS, SECRETARY andUNDERSECRETARY OF PUBLIC WORKS & COMMUNICATIONS

G.R.No. L-31271 (April 29, 1974)

 

FACTS:The spouses Romeo Martinez and Leonor Suarez, now petitioners-appellees, arethe registered owners of two (2) parcels of land located in Lubao, Pampanga,covered by transfer certificate of title No. 15856 of the Register of Deeds ofthe said province. Both parcels of land are fishponds. The property involved inthe instant case is the second parcel mentioned in the above-named transfercertificate of title.

Toavoid any untoward incident, the disputants agreed to refer the matter to theCommittee on Rivers and Streams. This committee thereafter appointed aSub-Committee to investigate the case and to conduct an ocular inspection ofthe contested property, and on March 11, 1954, said Sub-Committee submitted itsreport to the Committee on Rivers and Streams to the effect that Parcel No. 2of transfer certificate of title No. 15856 was not

apublic river but a private fishpond owned by the herein spouses.

Themunicipal officials of Lubao, led by Acting Mayor Mariano Zagad, apparentlyrefused to recognize the above decision. The writ of preliminary injunctionapplied for was issued against the respondent municipal Mayor, who

immediatelyelevated the injunction suit for review to the Supreme Court, which dismissedMayor Zagad's petition on September 7, 1953. With this dismissal order hereinappellee spouses proceeded to construct the dikes in the disputed parcel ofland.

Somefour (4) years later, and while Civil Case No. 751 was still pending theHonorable Florencio Moreno, then Secretary of Public Works and Communications,ordered another investigation of the said parcel of land, directing theappellees herein to remove the dikes they had constructed, on the strength ofthe authority vested in him by Republic Act No. 2056, approved on June 13,1958, entitled "An Act To Prohibit, Remove and/or Demolish theConstruction of Dams. Dikes, Or Any Other Walls In Public Navigable Waters, OrWaterways and In Communal Fishing Grounds, To Regulate Works in Such Waters orWaterways And In Communal Fishing Grounds, And To Provide Penalties For ItsViolation, And For Other Purposes.  

Thesaid order which gave rise to the instant proceedings, embodied a threat thatthe dikes would be demolished should the herein appellees fail to complytherewith within thirty (30) days.

Thespouses Martinez replied to the order by commencing on January 2, 1959 thepresent case, which was decided in their favor by the lower Court in a decisiondated August 10, 1959.

Asagainst this judgment respondent officials of the Department of Public Worksand Communications took the instant appeal.

ISSUE:W/N the Court may adjudge title over non-registrable properties such as publicnavigable river.

RULING:No.

TheLand Registration Court has no jurisdiction over non-registerable properties,such as public navigable rivers which are parts of the public domain, andcannot validly adjudge the registration of title in favor of a private applicant.

InArticle 339 of the old Civil Code which reads as follows:

Propertyof public ownership is:

1.That destined to the public use, such as roads, canals, rivers, torrents,ports, and bridges constructed by the State, and banks shores, roadsteads, andthat of a similar character. (Par. 1)

Theabove-mentioned properties are parts of the public domain intended for publicuse, are outside the commerce of men and, therefore, not subject to privateappropriation. ( 3 Manresa, 6th ed. 101-104.)

In Ledesmav. Municipality of Iloilo, 49 Phil. 769, this Court held:

Asimple possession of a certificate of title under the Torrens system does notnecessarily make the possessor a true owner of all the property describedtherein. If a person obtains title under the Torrens system which includes bymistake or oversight, lands which cannot be registered under the Torrenssystem, he does not by virtue of said certificate alone become the owner of theland illegally included.

Hence,the judgment of the Court of First Instance of Pampanga as regards the Lot No.2 of Certificate of Title No. 15856 in the name of petitioners-appellants maybe attacked at any time, either directly or collaterally, by the State which isnot bound by any prescriptive period provided for by the Statute of Limitations(Article 1108, par. 4, new Civil Code).

Beforepurchasing a parcel of land, it cannot be contended that the appellants whowere the vendees did not know exactly the condition of the land that they werebuying and the obstacles or restrictions thereon that may be put up by thegovernment in connection with their project of converting Lot No. 2 in questioninto a fishpond. Nevertheless, they willfully and voluntarily assumed the risksattendant to the sale of said lot. One who buys something with knowledge

ofdefect or lack of title in his vendor cannot claim that he acquired it in goodfaith (Leung Lee v. Strong Machinery Co., et al., 37 Phil. 664).

Theruling that a purchaser of a registered property cannot go beyond the record tomake inquiries as to the legality of the title of the registered owner, but mayrely on the registry to determine if there is no lien or encumbrances over thesame, cannot be availed of as against the law and the accepted principle thatrivers are parts of the public domain for public use and not capable of privateappropriation or acquisition by prescription.

Significance:

 

LandRegistration; Court may not adjudge title over non-registerable land.-- The Land Registration Court has no jurisdiction over non-registerableproperties, such as public navigable rivers which are parts of the publicdomain, and cannot validly adjudge the registration of title in favor of aprivate applicant. Hence, the judgment of the Court of First Instance ofPampanga as regards the Lot No. 2 of Certificate of Title No. 15856 in the nameof petitioners, may be attacked at any time, either directly or collaterally,by the state which is not bound by any prescriptive period provided for by theStatute of Limitations.

11(Jess)

GODOFREDO NAVERA, petitioner,  vs. HON. PERFECTO QUICHO ETC., ET AL., respondents.

Facts

On January 24, 1961, the municipality of Ligao filed with the Court of First Instance of Albay a petition under Section 112 of Act No. 496, as amended, for the correction of Transfer Certificate of Title No. T-9304 issued in the name of Godofredo Navera,

covering Lot No. 2793-A, on the ground that a portion of 123 sq. m. was erroneously included in said title during the cadastral survey of Ligao.

Navera filed a motion to dismiss based on the ground that the relief which petitioner seeks to obtain cannot be granted under Section 112 of Act 496 because the same would involve the opening of the original decree of registration. He contends that, under said section, the court can only authorize an alteration which may not impair the rights recorded in the decree, or one which will not prejudice such rights, or one which is consented to by all parties concerned, or can authorize the correction of any error or mistake which would not involve the reopening of the original decree of registration. Here the petition will have such effect, for it will involve the correction of the technical description of the land covered by the certificate of title in question, segregating therefrom the portion alleged to have been erroneously included, which eventually will cause the amendment of the original decree of registration. This cannot be done at this stage after the lapse of 23 years from the issuance of the certificate of title.

After hearing both parties, the court a quo issued an order denying the motion to dismiss and requiring Navera to answer the petition within the reglementary period. After his motion for reconsideration was denied, Navera filed the present petition for certiorari disputing the jurisdiction of the court a quo.

Issue

Whether or not it is under Section 112 of Act No 496

Held: No

The theory entertained by the court a quo that if the portion to be segregated was really erroneously included in the title issued to petitioner because it is part of the Natera street which belongs to the municipality of Ligao that portion may be excluded under Section 112 of Act 496 because under the law1 any public highway, even if not noted on a title, is deemed excluded therefrom as a legal lien or encumbrance, is in our opinion correct. This is upon the principle that a person who obtains a title which includes by mistake a land which cannot legally be registered does not by virtue of such inclusion become the owner of the land erroneously included therein.2 But this theory only holds true if there is no dispute that the portion to be excluded is really part of a public highway. This principle only applies if there is unanimity as to the issue of fact involved.1äwphï1.ñët

Here said unanimity is lacking. The claim of the municipality that an error has been committed in the survey of the lot recorded in respondent's name by including a portion of the Natera street is not agreed to by petitioner. In fact, he claims that is a question of fact that needs to be proven because it is controversial. There being dissension as to an important question of fact, the petition cannot be granted under Section 112 of Act No. 496.

We are of the opinion that the lower court did not err in finding that it lacks jurisdiction to entertain the present petition for the simple reason that it involves as controversial issue which takes this case out of the scope of Section 112 of Act No. 496. While this section, among other things, authorizes a person in interest to ask the court for any erasure, alteration, or amendment of a certificate of title "upon the ground that registered interests of any description, whether vested, contingent, expectant, or inchoate, have terminated and ceased", and apparently the petition comes under its scope, such relief can only be granted if there is unanimity among the parties, or there is no adverse claim or serious objection on the part of any party in interest; otherwise the case becomes controversial and should be threshed out in an ordinary case or in the case where the incident properly belongs.

WHEREFORE, petition is granted. The order of respondent court dated March 8, 1961, as well as its order dated March 25, 1961, are hereby set aside. No costs.

12(Diane)

 

G.R. No. 92013 July 25, 1990

 

FACTS: The Roppongi property was acquired from the Japanese government through Reparations Contract No. 300 dated June 27, 1958. A proposal was presented to President Corazon C. Aquino to make the property the subject of a lease agreement with a Japanese firm. On July 25, 1987, the President issued Executive Order No. 296 entitling non-Filipino citizens or entities to avail of separations' capital goods and services in the event of sale, lease or disposition. Vice-President Laurel states that the Roppongi property is classified as one of public dominion, and not of private ownership under Article 420 of the Civil Code. The respondents rely upon the rule of lex situs which is used in determining the applicable law regarding the acquisition, transfer and devolution of the title to a property.

ISSUE: Whether or not the Roppongi property can be alienated by the Philippine Government.

HELD: No. There can be no doubt that the property is of public dominion. The property is classified under Art 420 of the Civil Code as property belonging to the

State and intended for some public service. The fact that it has not been used for actual Embassy service does not automatically convert it to patrimonial property. Such conversion happens only if property is withdrawn from public use, through an abandonment of the intention to use the  Roppongi property for public service and to make it patrimonial property. Abandonment must be a certain and positive act based on correct legal premises.

The  Roppongi  property  was  acquired  together  with  the  other  properties through reparation agreements. They  were  assigned  to  the  government sector and that the Roppongi property was  specifically  designated  under the agreement to house the Philippine embassy. 

It  is  of  public  dominion  unless  it  is  convincingly  shown  that  the  property has become patrimonial. 

As property of public dominion, the Roppongi lot is outside the commerce of man. It cannot be alienated.    Its  ownership  is  a  special  collective ownership for general use and payment, in application to the satisfaction of collective needs, and resides in the social group. The purpose is not  to serve the State as the juridical person but the citizens; it is  intended for the common and public welfare and cannot be the object of appropriation. 

13(April)

14(Daniel Eblahan)

Navarav. Quicho

GRno. L-18339, June 29 1962

BautistaAneglo, J.:

Facts:

              Godofredo Navera owned a lot inLigao, Albay. On January 24, 1961, the municipality of Ligao petitioned theCourt of First Instance of Albay to correct the Transfer Certificate of Titleof Navera because a portion of his lot was erroneously included in the title.Said portion was a part of a street and the encroachment allegedly deprived thestreet of an area amounting to 123 sq. meters. Navera moved to dismiss thepetition because the correction would involve, not the correction of the title,but the opening of the original title and return of said portion to themunicipality. The lower court dismissed his motion and granted the petition,ruling that he cannot be the owner of the portion of the lot which was part ofthe street since public highways and streets cannot be registered and isexempted as a legal lien or encumbrance.

Issue:

             Whether or not the petition tocorrect the title of Navera should be allowed.

Held:

             No. The Court ruled that even thoughthe lower court was correct in its theory that the portion of the lot wasreally a part of the street and is excluded and cannot be registered and ownedby the petitioner, this theory applies only if there is unanimity among theparties that the land is in fact part of the street and there is no adverseclaim or any serious objection from any party in interest.

Inthis case, the Court ruled that there is no unanimity since there is anobjection coming from the petitioner, there is a controversy and it should beresolved in an ordinary case, not through the petition to correct petitioner’stitle.

15(Melodia)

________________________________________________________________________________________________

16(Mel)

 

REPUBLIC V. AYALA y SIA

14 SCRA 259

 

Facts:

 

In an amended complaint filed against Ayala Y Cia et al., the plaintiff sought the annulment of titles allegedly obtained by the defendant over portions of the territorial waters of the public domain. The defendant company caused the survey and preparation of a composite plan of Hacienda Calatagan, increasing its area from 9,652.583 hectares (as evidenced by TCT No. 722) to 12,000 hectares, by taking or including therein lands of public dominion. Thus, plaintiff also prayed for recovery of possession of such areas in excess of those covered by TCT No. 722, and for which fishpond permits were already issued in favor of bona fide applicants. Miguel Tolentino and 22 others alleged holders of fishpond permits issued by the Bureau of Fisheries over the areas supposedly outside the boundaries of Hacienda Calatagan, were allowed to intervene in the case and make demand for recovery of possession of said areas, and claim for damages for the deprivation of possession thereof allegedly by the illegal acts of defendants.

The defendants contended that the excess was insignificant in nature and attributable to the inaccuracy of the magnetic survey that was used in the preparation of the plan upon which OCT No. 20 was based.

After trial, the court rendered judgment annulling TCT No. T-9550 issued to defendants Dizons covering Lots 360, 362, 363 and 182, as well as other subdivision titles issued to Ayala y Cia. and/or Hacienda de Calatagan over the areas outside its private property covered by TCT No. 722. This ruling was based upon the finding that the disputed areas form part of the navigable water, or are portions of the sea, beach and foreshores of the bay.

 

Issue:

Whether the areas in dispute are territorial waters of the public domain.

Ruling:

The decision of the lower court appealed from is hereby affirmed.

The areas in dispute (those covered by permits issued by the Bureau of Fisheries) were found to be portions of the foreshore, beach, or of the navigable water itself. And, it is an elementary principle of law that said areas not being capable of registration, their inclusion in a certificate of title does not convert the same into properties of private ownership or confer title on the registrant.

In the present case, as the lots covered by TCT No. T-9550 issued in the names of defendants Dizons (and which were purchased by the latter from defendants Ayala y Cia., and/or Alfonso Zobel) were found to be portions of the foreshore or of the territorial waters, the lower court committed no error in rendering judgment against said defendants and ordering the reversion of said properties to the public dominion.

___________________________

17(Rocky)

18(Maribeth)

Republicvs Sioson, 9 SCRA533

FACTS:

OnNovember 6,1951, Spouses Sioson filed with the CFI an application forregistration of fourparcels of land situated in barrio San Roque, Paombong,Bulacan, claiming thatthey are the owners in fee simple. TheDirector of

Landsopposed the application for registration over Lot 4 based onseveral reasons;one which is that Lot 4 is a part of the public domain and assuch belong to theRepublic of the Philippines. After hearing, the court adjudicated and orderedthe registration of LotNos. 1, 2 and 3 in favor of the Spouses. No adjudicationas to Lot 4 hence the appeal with the Court of Appeals.

Onappeal, thelower court’s decision was modified as the Sol. Gen., in behalf ofthe Directorof Lands, recommended the registration of Lot 4 in the name oftheSpouses. By virtue of the modifiedjudgment, the lower court ordered theissuance of the decree on Lot 4.

OnApril 25, 1956,the Sol. Gen. in behalf of the Republic of the Philippines filedwith the lowercourt a petition for review of the decree of registration andcancellation oftitle over Lot 4 alleging actual and extrinsic fraud by theSpouses, byintentional and deliberate concealment of facts and connivance withthe landinspector. In the Spouse’s opposition tothe petition, they allegedamong other things that Lot 4 was and still is intruth and in fact an accretionto a titled parcel of land. The lower court, without hearing andpresentation ofevidence entered an order denying the petition, hence an appealby the Republicof the Philippines.

ISSUE: Whetheror not Lot4 being a part of the bed of a navigable river may be registered inthe name ofthe Spouses?

HELD: No.Navigable rivers cannot be appropriated and registeredunder the LandRegistration Act. If itis true that Lot 4 is or forms part of the bed of anavigable steam, creek orriver, the decree and title to it in the name of theSpouses would not givethem any right or title to it.

19(Ailyn)

20(Jen)

SECRETARY OF DENR VS YAP, GR NO. 173775, 8 OCTOBER 2008

FACTS: On November 10, 1978, then President Marcos issued Proc. No. 1801declaring Boracay Island, among other islands, caves and peninsulas in the

Philippines, as tourist zones and marine reserves under the administration of the Philippine Tourism Authority (PTA).  President Marcos later approved the issuance of PTA Circular 3-82 dated September 3, 1982, to implement Proclamation No. 1801.

Claiming that Proclamation No. 1801 and PTA Circular No 3-82 precluded them from filing an application for judicial confirmation of imperfect title or survey of land for titling purposes, respondents-claimants  Mayor Jose S. Yap, Jr., Libertad Talapian, Mila Y. Sumndad, and Aniceto Yap filed a petition for declaratory relief with the RTC in Kalibo, Aklan.

In their petition, respondents-claimants alleged that Proc. No. 1801 and PTA Circular No. 3-82 raised doubts on their right to secure titles over their occupied lands.  They declared that they themselves, or through their predecessors-in-interest, had been in open, continuous, exclusive, and notorious possession and occupation in Boracay since June 12, 1945, or earlier since time immemorial.  They declared their lands for tax purposes and paid realty taxes on them. Respondents-claimants posited that Proclamation No. 1801 and its implementing Circular did not place Boracay beyond the commerce of man.  Since the Island was classified as a tourist zone, it was susceptible of private ownership.  Under Section 48(b) of the Public Land Act, they had the right to have the lots registered in their names through judicial confirmation of imperfect titles.

The Republic, through the OSG, opposed the petition for declaratory relief.  The OSG countered that Boracay Island was an unclassified land of the public domain.  It formed part of the mass of lands classified as “public forest,” which was not available for disposition pursuant to Section 3(a) of  the Revised Forestry Code, as amended. The OSG maintained that respondents-claimants’ reliance on PD No. 1801 and PTA Circular No. 3-82 was misplaced.  Their right to judicial confirmation of title was governed by Public Land Act and Revised Forestry Code, as amended. Since Boracay Island had not been classified as alienable and disposable, whatever possession they had cannot ripen into ownership.

On July 14, 1999, the RTC rendered a decision in favor of respondents-claimants, declaring that, “PD 1810 and PTA Circular No. 3-82 Revised Forestry Code, as amended.

The OSG moved for reconsideration but its motion was denied. The Republic then appealed to the CA. On In 2004, the appellate court affirmed in toto the RTC decision. Again, the OSG sought reconsideration but it was similarly denied. Hence, the present petition under Rule 45.

On May 22, 2006, during the pendency the petition in the trial court, President Gloria Macapagal-Arroyo issued Proclamation No. 1064 classifying Boracay Island partly reserved forest land (protection purposes) and partly agricultural land (alienable and disposable).

On August 10, 2006, petitioners-claimants Sacay,and other landowners in Boracay filed with this Court an original petition for prohibition, mandamus, and nullification of Proclamation No. 1064. They allege  that the Proclamation infringed on their “prior vested rights” over portions of Boracay.  They have been in continued possession of their respective lots in Boracay since time immemorial.

On November 21, 2006, this Court ordered the consolidation of the two petitions.

Hence, this case.

ISSUE: Whether private claimants have a right to secure titles over their occupied portions in Boracay.

HELD:  petitions DENIED. The CA decision is reversed.

Except for lands already covered by existing titles, Boracay was an unclassified land of the public domain prior to Proclamation No. 1064.  Such unclassified lands are considered public forest under PD No. 705.

PD No. 705 issued by President Marcos categorized all unclassified lands of  the public domain as public forest.  Section 3(a) of PD No. 705 defines a public forest as “a mass of lands of the public domain which has not been the subject of the present

system of classification for the determination of which lands are needed for forest purpose and which are not.”  Applying PD No. 705, all unclassified lands, including those in Boracay Island, are ipso factoconsidered public forests.  PD No. 705, however, respects titles already existing prior to its effectivity.

The 1935 Constitution classified lands of the public domain into agricultural, forest or timber, such classification modified by the 1973 Constitution. The 1987 Constitution reverted to the 1935 Constitution classification with one addition: national parks. Of these, only agricultural lands may be alienated.Prior to Proclamation No. 1064 of May 22, 2006, Boracay Island had never been expressly and administratively classified under any of these grand divisions.  Boracay was an unclassified land of the public domain.

A positive act declaring land as alienable and disposable is required.  In keeping with the presumption of State ownership, the Court has time and again emphasized that there must be a positive act of the government, such as a presidential proclamation or an executive order; an administrative action; investigation reports of Bureau of Lands investigators; and a legislative act or a statute. The applicant may also secure a certification from the government that the land claimed to have been possessed for the required number of years is alienable and disposable. The burden of proof in overcoming such presumption is on the person applying for registration (or claiming ownership), who must prove that the land subject of the application is alienable or disposable.

In the case at bar, no such proclamation, executive order, administrative action, report, statute, or certification was presented to the Court.  The records are bereft of evidence showing that, prior to 2006, the portions of Boracay occupied by private claimants were subject of a government proclamation that the land is alienable and disposable. Matters of land classification or reclassification cannot be assumed. They call for proof.

Proc. No. 1801 cannot be deemed the positive act needed to classify Boracay Island as alienable and disposable land.  If President Marcos intended to classify the island as alienable and disposable or forest, or both, he would have identified the specific limits of each, as President Arroyo did in Proclamation No. 1064.  This was not done in Proclamation No. 1801.

CENONMATEO, petitioner-appellant, vs. HON. FLORENCIO MORENO, in his capacity asSECRETARY OF PUBLIC WORKS AND COMMUNICATIONS, defendant-appellee. G.R. No.L-21024           July 28, 1969FACTS: Sometimein 1959 a number of residents of Guiguinto, Bulacan, sent a letter-complaint tothe Highway District Engineer of that province asking that the Sapang Cabay, apublic navigable stream, which had been blocked by means of dikes and dams andconverted into fishponds, be ordered reopened and restored to its originalcondition. The letter was referred to the Secretary of Public Works andCommunications, who caused an investigation to be conducted pursuant toRepublic Act No. 2056. Acting on the report which the investigator submitted tohim, the Secretary rendered his decision on August 10, 1959, finding that theSapang Cabay was a public navigable stream and ordering Cenon Mateo, the hereinpetitioner-appellant, who had in the meantime acquired the property insidewhich the said creek is situated, to remove the dikes and dams thereinconstructed within thirty days from notice; otherwise they would be removed athis expense. Mateo moved to reconsider but was turned down, whereupon he filedthe basic petition to restrain the respondent Secretary from enforcing hisdecision. The petition, as already stated, was dismissed by the Court a quo.The certification of the appeal to us was upon motion of both parties in viewof the constitutional question involved.ISSUE: whether or not RepublicAct No. 2056 is unconstitutional because it unduly delegates judicial power tothe Secretary and unlawfully deprives the appellant and others similarlysituated of their property without due process of lawHELD: Theconstitutionality of the aforesaid statute has been upheld by this Court inLovina vs. Moreno, G.R. No. L-17821, November 29, 1963, shortly before thepresent appeal was submitted for decision. That case held, furthermore, thatthe absence of any mention of a navigable stream within a property covered by acertificate of title does not preclude a subsequent investigation anddetermination of its existence nor make it private property of the titleholder; that the findings of fact made by the Secretary of Public Works andCommunications should be respected in the absence of illegality, error of law,fraud or imposition, as long as such findings are supported by substantialevidence; and that the ownership of a navigable stream or of the bed thereof isnot subject to acquisitive prescription.

22(Nelson)

Republicvs CA 132 SCRA 514

Facts:

           Respondents sought to register the land adjacent to their fishpond; categorizedas lot 1, lot 2 and lot 3(later on lot 3 was withdrawn). They are theregistered owner of parcel of lot(Transfer Certificate of Title No. 89709)bordering on the Bocaue and Meycauyan rivers. Lot 1 and lot 2 are

accretions tothe land owned by the respondents and so the trial court ordered theregistration of land in favor of them. The petitioner submits that there is noaccretion to speak of under Article 457 of the New Civil Code because whatactually happened is that the private respondents simply transferred theirdikes further down the river bed of the Meycauayan River, and thus, if there isany accretion to speak of, it is man-made and artificial and not the result ofthe gradual and imperceptible sedimentation by the waters of the river.

Issue:

           Whether or not man-made alluvial deposit are registerable.

HELD:

NO.The lower court cannot validly order the registration of Lots 1 & 2 in thenames of the private respondents. These lots were portions of the bed of theMeycauayan river and are therefore classified as property of the public domainunder Article 420 paragraph 1 and Article 502, paragraph 1 of the Civil Code ofthe Philippines. They are not open to registration under the Land RegistrationAct. The adjudication of the lands in question as private property in the namesof the private respondents is null and void.

WHEREFORE,the instant petition is GRANTED. The decision appealed from is hereby REVERSEDand SET ASIDE. The private respondents are ordered to move back the dikes oftheir fishponds to their original location and return the disputed property tothe river to which it belongs.

23(JM)

MANECLANGVS. INTERMEDIATE APPELLATE COURT (IAC)

161SCRA 469

FACTS

          The Municipal Council of Bugallion,Pangasinan passed aResolutions Nos. 38 and 95 authorizing public bidding forthe lease of allmunicipal ferries and fisheries, including the fishpond.Alfredo Maza theawardees in the public bidding.

    Adriano Maneclang et al filed a complaintfor quitting of titleover

acertain fishpond belonging to them situated in Bario Salomague,Bugallion,Pangasinan, and the annulment of Resolution No. 38 and 95 of theMunicipalCouncil of Bugallion.

          The trial court dismissed the complaint upona findingthat the body of water traversing the titled properties of petitionersis acreek constituting a tributary of the Agno River; therefore public innatureand not subject to private appropriation.

    Due to lack of interest of respondentsAlfredo Maza in commentingthe complaint. The parties entered into amicablesettlement by Submitting to theCourt a Compromise Agreement praying thatjudgment be rendered recognizing theownership of petitioners over the land thebody of water found within theirtitled properties.

ISSUE:

Whetherthe creek or the body of watertraversing the titled properties of Maneclang etal is susceptible to privateappropriation.

HELD:

CREEKis defined as a recess, or armextending from a river from a river andparticipating in the ebb and flow ofthe sea, is a property belonging to thepublic domain which is not susceptibleto private appropriation and acquisitiveprescription, and as a public water itcannot be registered under the TorrensSystem in the name of any individual.

 

24(Maris)

25(Reg)

26 

G.R. No. L-30263-5           October 3, 1987

REPUBLIC OF THE PHILIPPINES, Represented by the DIRECTOR OF LANDS, petitioners, vs. THE HON. ANDRES REYES, Presiding Judge, Branch VI, Court of First Instance of Rizal, GODOFREDO R. EUSEBIO, URBANO C. LARA, GIL VENEZUELA, RODOLFO CENIDOZA, RAMON OROSA, AND JOSEFINA OROSA (Spouses), THE PHILIPPINE COMMERCIAL AND INDUSTRIAL BANK and THE REGISTER OF DEEDS OF RIZAL, respondents.

 

FACTS:

Sometime in 1965, Eusebio and Lara filed with the Bureau of Lands their Free Patent Applications for the parcels of land designated as Free Patent Application Nos. 7-207 and 7-208 for Lot no. 1 and Lot no. 2 respectively situated in Napindan, Taguig, Rizal. Their application was approved and the patents were registered in the Register of Deeds. It was later discovered by the Anti-Graft and Corruption Board of the Bureau of Lands that the lands were under water and form part of the Laguna de Bay – no signs of cultivation or of any improvement thereon.

On March 16 and 22, 1960, both Eusebio and Lara executed separate affidavits admitting noncompliance with the requirements of the Public Land Act and expressly agreed to have their patents and certificates of title cancelled.

The plaintiffs filed a case for the cancellation of the patents in the CFI Rizal. Eusebio and Lara failed to file an answer and was declared in default. The CFI Rizal ordered the cancellation of the patents. When advised to surrender their owner’s duplicate copy of Original Certificates, both claimed to have long surrendered the same to Atty. Javier of the Investigation Section of the Bureau of Lands.

Five years later, on June 3, 1967, Eusebio and Lara (plaintiff here) filed for the annulment of the decision on the same cases alleging that the CFI Rizal, 7th District, Branch II had not acquired jurisdiction over their persons and their decision was procured through fraud.

Despite records showing that the Director of Lands had not been properly served, the defendants were declared in default. CFI Rizal, 7th District, Branch IV, declared the earlier decision null and void.

The Director of Lands, claiming that the Branch IV had not acquired jurisdiction over his person, filed a Motion to Admit Petition to Reopen Proceedings with Additional Parties. These additional persons were alleged to have bought the subject lands from Eusebio and Lara. Venezuela, Cenidoza, and Orosa acquired Transfer Certificates of Title from Eusebio and Lara for P10,000. Venezuela and Cenidoza transferred their rights to Orosa. Spouses Orosa had executed a deed of mortgage in favor of Philippine Commercial Industrial Bank.

The petition was then denied after a motion for reconsideration filed by the intervenors.

 

ISSUE:

Whether Orosa, Venezuela and Cenidoza would be considered as buyers in good faith.

HELD:

It is well settled that any title issued on non-disposable lots even in the hands of an alleged innocent purchaser of value, shall be cancelled. In the case at bar, the free patents and certificates of title issued to Eusebio and Lara cover areas which form parts of Laguna de Bay. These are neither agricultural nor disposable. Subject patents and titles were erroneously issued due to misrepresentations and false reports and must therefore be cancelled. Any false statement in an application for public land shall ipso facto produce the cancellation of the title granted. This rule applies even after the issuance of the certificate of title. A certificate of title cannot be used as a shield to perpetuate fraud, and the doctrine of indefeasibility of Torrens title does not apply to free patent secured through fraud. Likewise, the mere possession of land does not itself divest the land of its public character.

Void free patents and certificates of title do not divest the state of its ownership of the land nor operate to change the public character of the land to private.

27(Ed)

28(LJ)

29(Zax)

30(Clathem)

Republic vs Sps. Maximo, 135 SCRA 156 Facts: This case is about the validity of the registration of 885 hectares of public forestal land located in Mulanay, Quezon.In Land Registration Case No. 81-G of the Court of First Instance at Gumaca, Quezon, Judge Vicente del Rosario on March 21, 1961 rendered a decision, ordering the registration of said land, Lot 1, allegedly located at Barrio Cambuga (Anonang), Mulanay, in the names of the spouses Prudencio Maxino and Tarciana Morales, less 200 hectares which should be registered in the names of the Heirs of Lorenzo Consolacion (72, Record on Appeal). The decision became final and executory. A decree and an original certificate of title were issued.Issue: Whether or not the Inclusion of a forest land in a title nullifies the title.Held: Yes. It is incontestable that Lot 1, the 885-hectare area registered by the Maxinos is within the public forest, not alienable and disposable nor susceptible of private appropriation. Thus, Its inclusion in the public forest certified by Director of Forestry shall annul the title of Maxino spouses.

31(Lea)

G.R. No. 181502, February 2, 2010DIAZ vs. REPUBLIC OF THE PHILIPPINES

FACTS: Petitioner’s late mother, Flora Garcia (Garcia) , filed an application for registration of a vast tract of land located in Laur, Nueva Ecija and Palayan City. She alleged that she possessed the land as owner and worked, developed and harvested the agricultural products and benefits of the same continuously, publicly and adversely for more or less 26 years.The Republic of the Philippines, represented by

the Office of the Solicitor General, opposed the application because the land in question was within the Fort Magsaysay Military Reservation (FMMR), established by virtue of Proclamation No. 237. Thus, it was inalienable as it formed part of the public domain. Significantly, SC already ruled in Director of Lands v. Reyes that the property subject of Garcia’s application was inalienable as it formed part of military reservation.The CFI ruled in favor of Garcia. The CA reversed CFI’s decision. The CA found that Reyes case was applicable to petitioner’s case because it involved the same property.During the pendency of the case in the CA, Garcia passed away and was substituted by her heirs, one of whom was petitioner Florencia Diaz.The parties ultimately entered into a compromise agreement with the Republic withdrawing its claim on the more or less 4,689 hectares supposedly outside the FMMR. For her part, petitioner withdrew her application for the portion of the property inside the military reservation. They filed a motion for approval of the amicable settlement in the CA and CA approved.However, acting on a letter written by a certain Atty. Restituto S. Lazaro, the OSG filed a motion for reconsideration and informed the appellate court that the tract of land subject of the amicable settlement was still within the military reservation. CA annulled the compromise agreement.

ISSUE: Whether petitioner can acquire the subject land that is part of military reservation.

RULING:By way of a background, SC recognized in Reyes that the property where the military reservation is situated in forest lands are not registrable under CA 141. Section 49 [b] of CA No. 141, as amended, applies exclusively to public agricultural land. Forest lands or area covered with forest are excluded. It is well-settled that forest land is incapable of registration; and its inclusion in a title, whether such title be one issued using the Spanish sovereignty or under the present Torrens system of registration, nullifies the title.However, it is true that forest lands may be registered when they have been reclassified as alienable by the President in a clear and categorical manner (upon the recommendation of the proper department head who has the authority to classify the lands of the public domain into alienable or disposable, timber and mineral lands) coupled with possession by the claimant as well as that of her predecessors-in-interest.Unfortunately for petitioner, she was not able to produce such evidence. Accordingly, her occupation thereof, and that of her predecessors-in-interest, could not have ripened into ownership of the subject land. This is because prior to the conversion of forest land as alienable land, any occupation or possession thereof cannot be counted in reckoning compliance with the thirty-year possession requirement under Commonwealth Act 141 (CA 141) or the Public Land Act.Therefore, even if possession was for more than 30 years, it could never ripen to ownership.But even assuming that the land in question was alienable land before it was established as a military reservation, there was nevertheless still a dearth of evidence with respect to its occupation by petitioner and her predecessors-in-interest for more than 30 years.

PartIII

1.DIRECTOR OF FORESTRY vs.VILLAREAL

G.R.No. L-32266 February 27, 1989

CRUZ, J.:

 

FACTS:

               A land consistingof 178,113square meters of mangrove swamps is located in the municipality ofSapian,Capiz. Ruperto Villareal applied for its registration alleging that heand hispredecessors-in-interest had been in possession of the land for morethan 40years. He was opposed by several persons, including the petitioner onbehalf ofthe Republic of the Philippine on the ground that mangrove swampswhich formpart of the public forests are cannot be disposed.

               In the case ofMontano v.Insular Government (1909), mangrove swamps or manglares are mudflats,alternately washed and exposed by the tide, in which grows variouskindredplants which will not live except when watered by the sea, extendingtheir rootsdeep into the mud and casting their seeds, which also germinatethere. Thus,mangrove swamps were considered agricultural lands and so susceptibleof privateownership. However, the Philippine Legislature subsequentlycategoricallydeclared that mangrove swamps form part of the public forests ofthis countrywhich are inalienable.

ISSUE:Whether or not mangrove swamps are comprised within the publicforests oragricultural lands.

HELD:

               Mangrove swampsor manglares shouldbe understood as comprised within thepublic forests of the Philippines asdefined in the aforecited Section 1820 ofthe Administrative Code of 1917. Thelegislature having so determined, thecourts have

no authority to ignore ormodify its decision, and in effect vetoit, in the exercise of the courts’discretion.

               Assuch, the Supreme Courtruled that Villareal was not able to establish hisright to the registration ofthe subject land in his name because mangroveswamps are inalienable.

2. Tongson vs. Director of ForestryG.R. No. L-34463 September 27, 1977Fernando, J.Facts:The testimonial evidence shows that as early as the year 1905 the parcel of land which later became Lot 855 of the cadastral survey of Pilar, was under the exclusive possession of Francisco Boria who cut trees therefrom and converted them into firewood. He also established a salt factory and that he sold the firewood and the salt without having been disturbed by anybody. After the death of Francisco. Boria, his son Arturo Borja took possession of the land, up to the year 1910. On May 1, 1917 Antero Borja sold the land to Deogracias Gayacao. On January 4, 1940, Deogracias Gayacao sold five parcels of land to Santiago M. Bermejo and one of the parcels known as parcel No. 4 is cadastral Lot No. 855. During his lifetime, Santiago M. Bermejo possessed said parcel of land, cut trees for the firewood purposes and also had a salt factory. Upon the death of Santiago M. Bermejo in 1951, his children took possession of this parcel of land and when Macario Bermejo was appointed judicial administrator by the Court of First Instance of Capiz, At present Lot 855 is a completed and producing fishpond. When Macario Bermejo took possession of the land in 1953 he converted it into a fishpond and started to construct fishpond dikes. However, due to lack of funds, the construction of the fishpond was not completed. On May 30, 1956, Macario Bermejo, in his capacity as administrator of the estate of the late Santiago M. Bermejo, leased the land to Leopoldo L. Somes with the approval of the Court of First Instance of Capiz. At present Leopoldo L. Somes is in actual possession of Lot 855 by virtue of said lease contract. Issue: whether or not a parcel of land, in the possession of the predecessors- in-interest and the oppositor Bermejo as far back as 1905, asserted to have originally been mangrove swamps, thereafter converted into a fishpond, may still be considered as part of the timber domain which is not disposable.Held:No. The opposition rests mainly upon the proposition that in the land covered by the application there are mangrove lands, but the court thinks this opposition of the Director of Forestry is untenable, in as much as it has been definitely decided that mangrove lands are not forests lands in the sense in which this phrase is used in the Act of Congress. It could be said, therefore, that even on the assumptions that the parcel of land in question could be characterized as mangrove swamps, the conclusion reached by the lower court is not without support in the applicable authorities. "Mangrove swamps where only trees of mangrove species grow, where the trees are small and sparse fit only for

firewood purposes and the trees growing are not of commercial value as lumber, do not convert the land into public land. Such lands are not forest in character. They do not form part of the public domain."

3(Gina)

REPUBLICOF THE PHILIPPINES  vs. AMANDA LAT VDA. DE CASTILLO

G.R. No.L-69002 June 30, 1988

Facts:

Sometimein 1951, the late Modesto Castillo applied for the registration of two parcelsof land, Lots 1 and 2, located in Banadero, Tanauan, Batangas, described inPlan Psu-119166, with a total area of 39,755 square meters. In a decision datedAugust 31, 1951, the said Modesto Castillo, married to Amanda Lat, was declaredthe true and absolute owner of the land with the improvements thereon, for whichOriginal Certificate of Title No. 0-665 was, issued to him by the Register ofDeeds at Batangas, Batangas, on February 7, 1952.

TheRepublic of the Philippines filed Civil Case No. 2044 with the lower court forthe annulment of the certificates of title issued to defendasors of ModestoCastillo, and for the reversion of the lands covered thereby (Lots 1 and 2,Psu-119166) to the State. It was alleged that said lands had always formed partof the Taal Lake, washed and inundated by the waters thereof, and being ofpublic ownership, it could not be the subject of registration as privateproperty.

On theother hand, private respondents maintain that Lots 1 and 2 have always been inthe possession of the Castillo family for more than 76 years and that their possessionwas public, peaceful, continuous, and adverse against the whole world and thatsaid lots were not titled during the cadastral survey of Tanauan, because theywere still under water as a result of the eruption of Taal Volcano on May 5,1911 and that the inundation of the land in question by the waters of Taal Lakewas merely accidental and does not affect private respondents' ownership

andpossession thereof pursuant to Article 778 of the Law of Waters. They finallyinsisted that this issue of facts had been squarely raised at the hearing ofthe land registration case and, therefore, res judicata.

Otherwisestated, it has been satisfactorily established as found by the trial court,that the properties in question were the shorelands of Taal Lake during thecadastral survey of 1923. The trial court decided the case in favor of thegovernment but the decision was reversed on appeal by the Court of Appeals.

Issue:

Whetheror not the lakeshore lands are part of the public domain.

Held:

Yes Lakeshoreland or lands adjacent to the lake, like the lands in question must bedifferentiated from foreshore land or that part of the land adjacent to the seawhich is alternately covered and left dry by the ordinary flow of the tides.

Suchdistinction draws importance from the fact that accretions on the bank of alake, like Laguna de Bay, belong to the owners of the estate to which they havebeen added  while accretion on a sea bankstill belongs to the public domain, and is not available for private ownershipuntil formally declared by the government to be no longer needed for public use.

But saiddistinction will not help private respondents because there is no accretionshown to exist in the case at bar. On the contrary, it was established that theoccupants of the lots who were engaged in duck raising filled up the area withshells and sand to make it habitable.

Thedefense of long possession is likewise not available in this case because, asalready ruled by this Court, mere possession of land does not by itselfautomatically divest the land of its public character.

4(Zhon)

Republic vs. Alagad, et al.

169 SCRA 455, G.R. No. L-66807, January 26, 1989

Facts:

On or about October 11, 1951, defendants filed an application for registration of their title over a parcel of land situated at Linga, Pila, Laguna, with an area of 8.1263 hectares, which was amended after the land was divided into two parcels, namely, Lot 1 with an area of 5.2476 hectares and Lot 2 with an area of 2.8421 hectares.

The Republic opposed the application on the ground that applicants and their predecessors have not been in possession of the land openly, continuously, publicly and adversely under a bona fide claim of ownership since July 26, 1894 and the land has not ceased to be a part of the public domain.

Defendants were declared owners of Lot 1 and the remaining portion, or Lot 2, was declared public land. Decree No. N-51479 was entered and Original Certificate of Title No. 0- 40 1, dated October 18, 1956, was issued in the names of defendants.

In August, 1966, Civil Case No. 52 of the Municipal Court of Pila, Laguna, was filed by defendants to evict the barrio folk occupying portions of Lot 1. On August 8, 1968, judgment was rendered in the eviction case ordering the defendants therein to return possession of the premises to herein defendants, as plaintiffs therein. The defendants therein did not appeal.

Republic filed a petition for annulment of title and reversion, claiming that the subject land is a foreshore land, belongs to the public domain, and concomitantly, cannot be appropriated by respondents. On October 6, 1970, as prayed for in the complaint, a writ of preliminary injunction was issued enjoining the Provincial Sheriff of Laguna or his deputies from enforcing the writ of execution issued in Civil Case No. 52, and the defendants from selling, mortgaging, disposing or otherwise entering into any transaction affecting the area.

This case was set for pre-trial on July 6, 1971. Despite notice of the pre-trial, Atty. Alejandro A. Ponferada, Special Attorney, Bureau of Lands, representing plaintiff Republic, did not appear. On July 16, 1971, the trial court dismissed the complaint.

In sustaining the trial court, the Court of Appeals held that under the Rules of Court, dismissal was proper upon failure of the Republic to appear for pre-trial. It likewise ruled that the judgment, dated January 16, 1956, in the said LRC No. 189 has long become final.

Issue:

1. w/n the Gov’t be bound by, or estopped from, the mistakes or negligent acts of its official or agents2. w/n the subject land is a foreshore land [define foreshore land]Held:1. It is well-established that the State cannot be bound by, or estopped from, the mistakes or negligent acts of its official or agents, 7 much more, non-suited as a result thereof.

2. A foreshore land is a strip of land that lies between the high and low water marks and that is alternatively wet and dry according to the flow of the tide. If the submergence, however, of the land is due to precipitation, it does not become foreshore, despite its proximity to the waters.Since this issue is a question of fact, SC remanded the case to the trial court to determine whether or not the property subject of controversy is foreshore.

Held:

1. It is well-established that the State cannot be bound by, or estopped from, the mistakes or negligent acts of its official or agents, 7 much more, non-suited as a result thereof.

2. A foreshore land is a strip of land that lies between the high and low water marks and that is alternatively wet and dry according to the flow of the tide. If the submergence, however, of the land is due to precipitation, it does not become foreshore, despite its proximity to the waters.

Since this issue is a question of fact, SC remanded the case to the trial court to determine whether or not the property subject of controversy is foreshore.

5(May Ann)

6(Rhea B.)

DIRECTOR OF LAND MANAGEMENT VS CA205 SCRA 486Facts:Teodoro Abistado filed a petition for original registration of his title over 648 square meters of land under Presidential Decree (P.D.) No. 1529. The land registration court in its decision dated June 13, 1989 dismissed the petition “for want of jurisdiction”, in compliance with the mandatory provision requiring publication of the notice of initial hearing in a newspaper of general circulation. The case was elevated to respondent Court of Appeals which, set aside the decision of the trial court and ordered the registration of the title in the name of Teodoro Abistado. The Court of Appeals ruled that it was merely procedural and that the failure to cause such publication did not deprive the trial court of its authority to grant the application. The Director of Lands represented by the Solicitor General thus elevated this recourse to the Supreme Court.Issue:Whether or not the Director of Lands is correct that newspaper publication of the notice of initial hearing in an original land registration case is mandatory.Ruling:YES. Petition was granted.The pertinent part of Section 23 of Presidential Decree No. 1529 requires publication of the notice of initial hearing. It should be noted further that land registration is a proceeding in rem. Being in rem, such proceeding requires constructive seizure of the land as against all persons, including the state, who have rights to or interests in the property. An in rem proceeding is validated essentially through publication. This being so, the process must strictly be complied with.The Supreme Court has no authority to dispense with such mandatory requirement. The law is unambiguous and its rationale clear. Time and again, this Court has declared that where the law speaks in clear and categorical language, there is no room for interpretation, vacillation or equivocation; thereis room only for application. There is no alternative. Thus, the application for land registration filed by private respondents must be dismissed without prejudice to reapplication in the future, after all the legal requisites shall have been duly complied with.

7(Angel)

8. G.R. No. L-69002 June 30, 1988

Republic vs. Vda. de Castillo, 163 SCRA 286

Facts:

In 1951, Modesto Castillo applied for the registration of two parcels of land, Lots 1 and 2, located in Tanauan, Batangas. On August 31, 1951 Modesto, married to Amanda Lat, was declared the true and absolute owner of the land, for which Original Certificate of Title was issued to him by the Register of Deeds. After

Modesto’s death, Amanda Lat Vda. de Castillo, et al., executed a deed of partition and assumption of mortgage in favor of Florencio L. Castillo, et al.

The Republic of the Philippines filed a case in the lower court for the annulment of the certificates of title issued to defendantx, as heirs/successors of Modesto Castillo, and for the reversion of the lands covered thereby to the State. Petitioner contends that Lots 1 and 2 had always formed part of the Taal Lake, washed and inundated by the waters thereof. Consequently, the same were not subject to registration, being outside the commerce of men; and that since the lots in litigation are of public domain, the registration court (of 1951) did not have jurisdiction to adjudicate said lands as private property.

On the other hand, private respondents maintain that Lots 1 and 2 have always been in the possession of the Castillo family for more than 76 years and that their possession was public, peaceful, continuous, and adverse against the whole world and that said lots were not titled during the cadastral survey of Tanauan, because they were still under water as a result of the eruption of Taal Volcano on May 5, 1911 and that the inundation of the land in question by the waters of Taal Lake was merely accidental and does not affect private respondents' ownership and possession thereof pursuant to Article 778 of the Law of Waters.

After trial the Court of First Instance ruled in favor of the Republic of the Philippines.

The Court of Appeals subsequently reversed the decision and dismissed the complaint.

Issue:

Are the properties in dispute capable of being registered?

Held:

No. Shores are properties of the public domain intended for public and, therefore, not registrable. Thus, it has long been settled that portions of the foreshore or of the territorial waters and beaches cannot be registered. Their inclusion in a certificate of title does not convert the same into properties of private ownership or confer title upon the registrant.

 It has been satisfactorily established as found by the trial court, that the properties in question were the shore lands of Taal Lake during the cadastral survey of 1923.

Lakeshore land or lands adjacent to the lake, like the lands in question must be differentiated from foreshore land or that part of the land adjacent to the sea which is alternately covered and left dry by the ordinary flow of the tides.

Such distinction draws importance from the fact that accretions on the bank of a lake, like Laguna de Bay, belong to the owners of the estate to which they have been added  while accretion on a sea bank still belongs to the public domain, and is not available for private ownership until formally declared by the government to be no longer needed for public use.

The defense of long possession is likewise not available in this case because, as already ruled by this Court, mere possession of land does not by itself automatically divest the land of its public character.

Cantoja vs Lim

Facts:   Late Roberto Cantoja Sr.filed with the office of the DENR, GenSan City, an application for a ForshoreLease Contract over an area situated in Makar, GenSan. Cantoja was arawded theFLA (Foreshore Lease Agreement) on Novemver 1990. Lim protest questioning thesaid FLA to Cantoja based on his allegation that Cantoja committed fraud andmisrepresentation in declaring in his application that the subject foreshorearea adjoined hi (cantojas) property. To prove, he presented his TransferCertificate Title which adjoins the foreshore area subject of the lease.

                Regional ExecutiveDirector Momongan of DENR Davao assigned the case to Investigator Marohomsalicfor further investigation which found that Cantoja was in acyual possession ofthe foreshore area which utilized as dock-board of the Cantojas FishingBusiness. On Feb 1996, Geodetic Engineer Soria, in compliance with the Oct 1995 Order of the City and NaturalResources Office, Submitted his report stating that there was bo overlapping ofthe lot of Cantoja and the Foreshore area. Director Momongan dismissed theprotest.

                Meanwhile, on Oct1997 DENR instituted Civil case for annulment of Patent coth issued in the Nameof Jacinto Acharon, as well as Lim’s TCT. The suit was anchored on the findingsand recommendations of Maromhomsalic that the area in question is partly foreshoreand partly river bed of the Makar GenSan and therefore, inalienable.

                On May 2, 2000,DENR Sec. Cerilles cancelled the FLA previously granted to Cantoja. Cantojafiled again a motion for Recon. Sec. Cerilles issued special order for thecreation of a team to conduct investigatiob and ocular inspection. Withoutwaiting, however,for the result of the investigation of said team, Sec.Cerilles set aside its 2 May 2000 in favour of Cantoja. On appeal, the Officeof the president rendered herein assailed decision affirming the 17 Oct 2000Order of the DENR Sec.

                Lim appealed to CAwhich reinstated the May 2 2000 decision of the DENR Sec, which cancelled andrescinded the FLC covering the foreshore area.

Issue: Whether the CA erred in cancelling the Foreshore Lease Contractgranted ro Cantoja covering the foreshore area.

Ruling: No. It is undisputed thatCantoja is the not the Registered owner of the land adjacent to the foreshorearea leased to them. Cantoja committed fraud when he misrepresented himself asthe riparian of littoral owner in his application for the foreshore lease.Under stipulation no. 15 of the FLA, any fraud or misrepresentation committedby the applicant is a ground for cancellation or rescission of the FLA.

10(Lou)

BALAIS-MABANAGv. REGISTER OF DEEDS OF QUEZON City, CONCEPCION D. ALCARAZ, and RAMONAPATRICIA ALCARAZ

G.R.No. 153142 (29 March 2010)

BERSAMIN, J.:

 

FACTS:  On January 19, 1985, Coronels (Romulo A. Coronel, Alarico A.Coronel, Annette A. Coronel, Annabelle C. Gonzales, Floraida C. Tupper, andCielito A. Coronel) executed a document entitled receipt of down payment,stipulating that they received from respondent Ramona Patricia Alcaraz(Ramona), through Ramona’s mother, respondent Concepcion D. Alcaraz(Concepcion), the sum of P50,000.00 as

downpayment on the total purchaseprice of P1,240,000.00 for their "inherited house and lot, covered byTCT No. 119627 of the Registry of Deeds of Quezon City."

OnFebruary 18, 1985, the Coronels sold the property covered by TCT No. 327043 tothe petitioner for the higher price of P1,580,000.00 after the latterdelivered an initial sum of P300,000.00. For this reason, the Coronelsrescinded their contract with Ramona by depositing her downpaymentof P50,000.00 in the bank in trust for Ramona Patricia Alcaraz.

TheCoronels executed a deed of absolute sale in favor of the petitioner. On June5, 1985, TCT No. 351582 was issued in the name of the petitioner.

OnMarch 1, 1989, the RTC rendered its decision ordering defendant to execute infavor of plaintiffs(Concepcion and Ramona) a deed of absolute sale and nopronouncement as to costs. Upon denial of the motion for reconsideration, theCoronels and the petitioner interposed an appeal to the CA, which promulgated ajudgment on December 16, 1991, fully upholding the decision of the RTC. Thus,the petitioner and the Coronels appealed the CA judgment to this Court (G.R.No. 103577), which affirmed the CA on October 7, 1996. Thereafter, thedecision of the RTC became final and executory.

 

Uponfailure of the petitioner and the Coronels to comply with the writ ofexecution, the RTC approved the respondents’ motion for appointment of suitableperson to execute deed, etc., and ordered on April 8, 1998 the Branch Clerk ofthe RTC, Branch 83, Quezon City, to execute the deed of absolute sale in favorof Ramona in lieu of the defendants (i.e., the petitioner and the Coronels).

Thepetitioner and the Coronels filed in the CA a petition for certiorari assailingthe RTC’s orders of October 1, 1997 and March 10, 1998, but the CA dismissedthe petition on July 30, 1998.

OnSeptember 2, 1998, the RTC held in abeyance the respondents’ motion reiteratingprevious motion to resolve respondents’ motion, whereby the respondents soughtan order to direct the petitioner to surrender her TCT No.

331582, and theRegistrar of Deeds of Quezon City to cancel the petitioner’s copy of said TCTfor her failure to comply with the earlier order for her to surrender the TCTto the Registrar of Deeds pending resolution by the CA of the petitioner’s motionfor reconsideration.

Ultimately,on September 30, 1998, the CA denied the petitioner’s motion forreconsideration.

ISSUE:W/N the petitioner lacked the personality to file the suit.

HELD:YES.

TheCourt stated that under Section 7, Batas Pambansa Blg.185, the Solicitor General or his representative shall institute escheatproceedings against its violators. Although the law does not categoricallystate that only the Government, through the Solicitor General, may attack thetitle of an alien transferee of land, it is nonetheless correct to hold thatonly the Government, through the Solicitor General, has the personality to filea case challenging the capacity of a person to acquire or to own land based onnon-citizenship. This limitation is based on the fact that the violation iscommitted against the State, not against any individual; and that in the eventthat the transferee is adjudged to be not a Filipino citizen, the affectedproperty reverts to the State, not to the previous owner or any otherindividual.

Herein,even assuming that Ramona was legally disqualified from owning the subjectproperty, the decision that voids or annuls their right of ownership over thesubject land will not inure to the benefit of the petitioner. Instead, thesubject property will be escheated in favor of the State in accordance withBatas Pambansa Blg. 185.

WHEREFORE,the petition for review on certiorari is denied, and the decision datedDecember 5, 2000 promulgated in C.A.-G.R. SP No. 55576 is affirmed.

Coststo be paid by the petitioner.

SOORDERED.

Significance:Only the Government through the Solicitor General, has the personality tofile a case challenging the capacity of a person to acquire or to own landbased on non-citizenship.

11(Jess)

12(Diane)

OFFICE OF THE CITY MAYOR OF PARAÑAQUE CITY v. MARIO D.EBIO AND HIS CHILDREN/HEIRS

G.R. No. 178411 June 23, 2010

FACTS: Respondents claim to be absolute owners of a parcel of land in Parañaque City covered by Tax in the name of respondent Mario D. Ebio. Said land was an accretion of Cut-cut creek. Respondents assert that the original occupant and possessor land was their great grandfather, Jose Vitalez, which was given to his son, Pedro Valdez, in 1930. From then on, Pedro continuously and exclusively occupied and possessed the said lot. In 1966, after executing an affidavit declaring possession and occupancy. He also paid taxes for the land.

Mario Ebio married Pedro’s daughter, Zenaida. Ebio secured building permits from the Parañaque municipal office for the construction of their house within the land. On March 30, 1999, the Office of the Sangguniang Barangay of Vitalez passed a resolution seeking assistance from the City Government of Parañaque for the construction of an access road along Cut-cut Creek. The proposed road will run from

Urma Drive to the main road of Vitalez Compound traversing the lot occupied by the respondents. Respondents immediately opposed and the project was suspended.

On March 28, 2005, the City Administrator sent a letter to the respondents ordering them to vacate the area within the next thirty (30) days, or be physically evicted from the said property. Respondents sent a reply, asserting their claim over the subject property and expressing intent for further dialogue.

ISSUE: Whether or not the State may build on the land in question.

HELD: No. It is an uncontested fact that the subject land was formed from the alluvial deposits that have gradually settled along the banks of Cut-cut creek. This being the case, the law that governs ownership over the accreted portion is Article 84 of the Spanish Law of Waters of 1866, which remains in effect, in relation to Article 457 of the Civil Code.

It is therefore explicit from the foregoing provisions that alluvial deposits along the banks of a creek do not form part of the public domain as the alluvial property automatically belongs to the owner of the estate to which it may have been added. The only restriction provided for by law is that the owner of the adjoining property must register the same under the Torrens system; otherwise, the alluvial property may be subject to acquisition through prescription by third persons.

In contrast, properties of public dominion cannot be acquired by prescription. No matter how long the possession of the properties has been, there can be no prescription against the State regarding property of public domain. Even a city or municipality cannot acquire them by prescription as against the State.

Hence, while it is true that a creek is a property of public dominion, the land which is formed by the gradual and imperceptible accumulation of sediments along its banks does not form part of the public domain by clear provision of law.

NOTES:

 ART. 84. Accretions deposited gradually upon lands contiguous to creeks, streams, rivers, and lakes, by accessions or sediments from the waters thereof, belong to the owners of such lands.

Art. 457. To the owners of lands adjoining the banks of rivers belong the accretion which they gradually receive from the effects of the current of the waters.

13(April)

14(Daniel Eblahan)

Directorof Lands v. Rivas

GRno. L-61539, February 14, 1986

Aquino,C.J.:

Facts:

             On March 14, 1873, the Alcalde Mayorand the Court of First Instance of Tuguegarao granted Domingo Bunagan apossessory information title called “informacion posesoria”, for a tract ofland “destined for grazing their cattle”. On November 3, 1885, he also obtaineda gratuitous adjustment title called “composicion gratuita” for the parcel ofland in Nottab, Enrile, Cagayan. After Bunagan’s death, the land was sold todifferent people. In another case the court ruled that the whole land was soldto Cagayan Valley Agricultural Corporation. In this case, a remainder of theland was transferred to respondent Rivas by his brother and the land waseventually sold to the other respondents. The trial court declared the landpublic land and dismissed both their claims. The appellate court reversed theruling of the lower court and granted the respondents’ application. TheDirector of Lands appealed, contending that the land is part of a forestreservation.

Issue:

             Whether or not the land in dispute isinalienable land

Held:

             Yes. The Court ruled to dismiss theapplications of the respondents because the land is inalienable public grazingland, part of a forest reserve under Presidential Proclamation 129 and cannotbe registered. Grazing lands and timber lands are not

alienable under the 1935and 1973 Constitutions and cannot be the subject of private ownership.

             In this case, the land in questionwas issued to the original claimant in 1873 was described in the “informacionpossessoria” as “una estancia de ganado al terreno” or grazing land. RespondentRivas leased it in 1962 as “pasture land”. In the 1960 and 1968 taxdeclarations of respondent Rivas described the land as for “pastureexclusively.” In the sale to respondent Pascua’s father describing the land as“a parcel of pasture land.” The land has always been described and treated aspasture or grazing land.

15(Melodia)

_______________________________________________________________________________

16(Mel)

Republic of the Philippines, petitioner

vs

Intermediate Appellate Court, Esteban Mendoza and Leon Pasahol

155 SCRA 412   November 5, 1987

FACTS:

                On December 8, 1968, a petition was filed by Esteban Mendoza and Leon Pasahol with the then Court of First Instance of Bataan with alleging ownership of a parcel of land which they have purchased from its original owners and thereafter, they have actual possession of the said land tacked on to their predecessors-in-interest for a period exceeding 30 years.

                The CFI of Bataan ruled on the case, favoring the private respondent citing that “after the sale had zealously cultivated the property and religiously paid the taxes thereon for a good numbers of years”, the CFI of Bataan ruled that the private

respondent’s possession of the land was in good faith and that the private respondent should not as a consequence, be held accountable for the lapse of their predecessor to file a cadastral claim to the property. The private defendant may tack their period of possession with that of their vendors totaling to more than thirty (30) years.

                When the Solicitor General appealed it to the Intermediate Court pointing out Section 1 of Republic Act 391, as amended by Republic Act 2061, wherein the Solicitor General reiterated that, reopening cadastral proceedings is allowable only with respect to such of said parcels of land as have not been alienated, reserved, leased, granted, or otherwise provisionally or permanently disposed of by the Government. Applying said provision to the lot in question, it is claimed that the registration is not possible as said land is actually already forest land and/or part of military reservation. In support of the contention of the Solicitor General, it also cited the report of Forest Guard Cresencio Abuzman to the District Forester which states that “the area involved is a portion of former Military Reservation turned over to the Philippine Government, hence, not disposable under any circumstances.

                The Intermediate Appellate Court rendered its decision affirming the decision of the CFI and disagreed with the contention of the Solicitor General, stating that under the Proposed Land Classification Project  No. 4-C of Mariveles Bulacan, the contested lot has already been delineated and classified by Bureau of Forestry as alienable and disposable public land and was approved by the Director of Lands as disposable in character.

ISSUE:

                Whether or not a mere proposal to classify a Military Reservation as alienable and disposable public land is allowed.

HELD:

                No, the Supreme did not sustained the ruling of the Intermediate Appellate Court that the land in dispute is no longer a part of Military reservation on the basis of a mere proposal to classify it as a alienable and disposable public land that is needed according to the Supreme Court is a formal act declaring forest land reservation for the purpose of classifying it as a alienable and disposable public land need the approval of the President of the Philippines as required by Public Land Act (C.A. No. 141) and R.A. 1275.

__________________________________________________________________

17(Rocky)

18(Maribeth)

Miller vs. DirectorofLands, 12 SCRA 292

 

FACTS:

OnJune 18, 1956, Miller and Espinosa applied for registration in theCFI ofMasbate covering a parcel of land located in Tigbao, Milagros, Masbate.Afternotice and publication, initial hearing was held. The Director of Landsand Bureauof Public Highways filed written oppositions, while 35 individualsappeared andexpressed verbal oppositions.

Applicants’presentation of evidence ensued; private oppositors weregiven 5 days to filewritten opposition for which 28 filed written butunverified opposition. OnAugust 20, 1958 applicants finished adducing evidenceand rested their case.

OnAugust 27, 1958 the private oppositors presented their firstwitness. After hiscross-examination, counsel for applicants called the Court'sattention to thelack of verification in the opposition filed by the privateoppositors and movedto dismiss the same.

Thus,the private oppositors offered to verify their opposition.After parties hadfiled memoranda, the court issued an order dismissing theunverified opposition.Motion for reconsideration was denied. The privateoppositors appealed from bothorders alleging among others that lack of, ordefect in the verification of apleading may be waived by the adverse party'sfailure to make a proper andtimely objection thereto. Where a party proceedswith the case as though hisadversary's pleading were verified, he waives thelack of verification of suchpleading.

 

ISSUE: Whetheror not the belatedobjection to the unverified opposition to the applicationshall be considered awaiver to such defect?

HELD: Yes.The court heldthat the written appearance with opposition presentedbypetitioners was a valid one, and sufficient to give them legal

standingincourt and would entitle them to notice, as a matter of right. The lowercourterred in choosing to ignore the written appearance with opposition, whichwas asubstantial compliance with the law, that requires a formal answer. Theprivate oppositors should be allowed, asthey had requested, to verify theiropposition because, in any event, thesupposed defect is deemed waived.Theorders appealed from are set aside and the case is remanded to the court aquo for furtherproceedings.  

19(Ailyn)

20(Jen)

REP. VS. CA, 73 SCRA 146

FACTS: On January 22, 1921, Eugenio de Jesus, the father of respondent Alejandro de Jesus, applied with the Bureau of Lands for Sales Patent (Sales Application No. 5436) of a 33-hectare situated in barrio Libaron, Municipality of Davao (now Davao City).  The property applied for was a portion of what was then known as Lot 522 of the Davao Cadastre.

On January 23, 1934, the Bureau of Lands, through its Davao District Land Officer, accepted sealed bids for the purchase of the subject land. One Irineo Jose bidded for P20.00 per hectare, while a certain Dr. Josc Ebro submitted a bid of P100.50 per hectare The Director of Lands, however, annulled the auction sale for the reason that the sales applicant, Eugenio de Jesus, failed to participate in the bidding for non-service of notice on him of the scheduled bidding.

In lieu of that sale, another bidding was held on October 4, 1934. Sales applicant Eugenio de Jesus was the lone bidder. He equalled the bid previously submitted by Dr. Jose Ebro and made a deposit of P221.00 representing 10% of the price of the land at P100.50 per hectare.

On November 23, 1934, the Director of Lands issued to Eugenio de Jesus an Order of Award.

On August 28, 1936, the Director of Lands ordered an amendment of the Sales Application of Eugenio de Jesus stating that "a portion of the land covered by Sales Application No. 5436 (E-3231) of Eugenio de Jesus is needed by the Philippine Army for military camp site purposes, the said application is amended so as to exclude therefrom portion "A" as shown in the sketch on the back thereof, and as thus amended, it will continue to be given due course." The area excluded was Identified as Lot 1176-B-2, the very land in question, consisting of 12.8081 hectares.

On September 7, 1936, President Manuel L. Quezon issued Proclaimation No. 85 withdrawing Lot No. 1176-B-2 from sale and settlement and reserving the same for military purposes, under the administration of the Chief of Staff, Philippine Army.

On November 29, 1939, Eugenio de Jesus paid P660.45 covering the 8th and 10th installment for 20.6400 hectares, the remaining area after his Sales Application was amended. This payment did not include the military camp site (Lot No. 1176-B-2) as the same had already been excluded from the Sales Application at the time the payment was made.  

On August 11, 1956, President Ramon Magsaysay revoked Proclamation No. 85 and declared the disputed Lot 1176-B-2 open to disposition under the provisions of the Public land Act for resettlement of the squatters in the Piapi Beach, Davao City.  In the following October 9, President Magsaysay revoked this Proclamation No. 328 and reserved the same Lot No. 1176-B-2 for medical center site purposes under the administration of the Director of Hospital. 

Whereupon, on December 6, 1969, petitioner Mindanao Medical Center applied for the Torrens registration of the 12.8081-hectare Lot 1176-B-2 with the Court of First Instance of Davao. The Medical Center claimed "fee simple" title to the land on the strength of proclamation No. 350 reserving the area for medical center site purposes.

Respondent Alejandro de Jesus, the son and successor-in-interest of sale applicant Eugenio de Jesus, opposed the registration oil the ground that his father, Eugenio de Jesus, had aquired a vested right on the subject lot by virtue of the Order of Award issued to him by the Director of Lands.

A certain Arsenio Suazo likewise filed his opposition to the registration on the claim that the 2-hectare portion on the northeastern part of Lot 1176-B-2 belongs to him.

On September 2, 1966 the CFI of Davao awarded the lot to Mindanao Medical Center, Bureau of Medical Services, Department of Health.

The two oppositors, Alejandro de Jesus and Arsenio Suazo appealed the case to the respondent Court of Appeals.

CA modified the judgment denying the claim of Arsenio Suazo and affirmed the appeal of Alejandre Y. de Jesus.

On July 5, 1974, petitioner Mindanao Medical Center moved for reconsideration, maintaining ownership over the entire area of 12.8081 hectares, but the Appellate Court in a Special Division of Five denied the motion on June 17, 19750.

Hence, this case.

ISSUE: Whether or not petitioner Mindanao Medical Center has registerable title over a full 12.8081-hectare land by virtue of an executive proclamation in 1956 reserving the area for medical center site purposes.

HELD: Petitioner Mindanao Medical Center has registerable title over the whole contested area of 12.8081 hectares, designated Lot No. 1176-B-2, and not only on a portion thereof occupied by the Medical Center, its nervous disease pavilion and their reasonable appurtenances. Proclamation No. 350, dated October 9, 1956, of President Magsaysay legally effected a land grant to the Mindanao Medical Center,

Bureau of Medical Services, Department of Health, of the whole lot, validity sufficient for initial registration under the Land Registration Act. Such land grant is constitutive of a "fee simple" tile or absolute title in favor of petitioner Mindanao Medical Center. Thus, Section 122 of the Act, which governs the registration of grants or patents involving public lands, provides that "Whenever public lands in the Philippine Islands belonging to the Government of the Philippines are alienated, granted, or conveyed to persons or to public or private corporations, the same shall be brought forthwith under the operation of this Act [Land Registration Act, Act 496] and shall become registered lands." 9 It would be completely absurd to rule that, on the basis of Proclamation No. 350, the Medical Center has registerable title on the portion occupied by it, its nervous disease pavilion and the reasonable appurtenances, and not on the full extent of the reservation, when the proclamation explicitly reserved the entire Lot 1176-B-2 of 12.8081 hectares to the Center.

REPUBLICOF THE PHILIPPINES, THE SUPERINTENDENT OF THE PHILIPPINE MILITARY ACADEMY,petitioners, vsHON. PIO R. MARCOS, JUDGE, Court of First Instance of BaguioCity, KOSEN PIRASO, SAMAY PIRASO, COTILENG PIRASO, PETER PARAN and MARTINAPIRASO, DAISY PACNOS, SPOUSES ALBINO REYES and ISABEL SANTAMARIA, and ARTUROTONGSON, respondents.

FACTS: It is by statute provided thatall persons "claiming title to parcels of land that have been the objectof cadastral proceedings" in actual possession of the same at the time ofthe survey but unable for some justifiable reason to file their claim in theproper court during the time, limit established by law, "in case suchparcels of land on account of their failure to file such claims, have been, orare about to be declared land of the public domain by virtue of judicialproceedings" instituted within the forty-year period next preceding June20, 1953, the time of the approval of this particular enactment, are granted"the right within five years" from said date to petition for areopening of the judicial proceedings but "only with respect to such ofsaid parcels of land as have not been alienated, reserved, leased, granted, orotherwise provisionally or permanently disposed of by the Government, ...."The jurisdiction of respondent Judge Pio R. Marcos to act in accordancewith Republic Act No. 931 in connection with the petition for a reopening filedby respondent Kosen Piraso, joined by his kinsmen, likewise respondents, allsurnamed Piraso, is assailed in this certiorari and prohibition proceeding,included in which are the other respondents, Daisy Pacnos and the spousesAlbino Reyes and Isabel Santamaria, petitioners being the Republic of thePhilippines and the Superintendent of the Philippine Military Academy. 

ISSUE:Whether or not , respondent Judge is devoid of jurisdiction to pass uponthe claim of private respondents invoking the benefits of Republic Act No. 931. 

HELD:Republic Act No. 931 speaks in a manner far from ambiguous. It is quiteexplicit and categorical. Only persons "claiming title to parcels of landthat have

been the object of cadastral proceedings" are granted the rightto petition for a reopening thereof if the other conditions named therein aresuccessfully met. It cannot admit of doubt, therefore, that if the parcels ofland were not the object of cadastral proceedings, then this statute finds noapplication. Considering that as far back as October 10, 1910, the thenPresident of the United States, William H. Taft, issued an executive orderreserving for naval purposes the lots now disputed, they could not have beenthe object of the cadastral proceeding involving the Baguio townsitereservation, decided only on November 13, 1922.What is even more conclusive asto the absence of any right on the part of the private respondents to seek areopening under Republic Act No. 931 is our ruling in Government v. Court ofFirst Instance of Pampanga, a 1926 decision. We there explicitly held:"The defendant's contention that the respondent court, in a cadastralcase, has jurisdiction to order the registration of portions of a legallyestablished military reservation cannot be sustained. The establishment ofmilitary reservations is governed by Act No. 627 of the Philippine Commissionand Section 1 of that Act provides that 'All lands or buildings, or anyinterest therein, within the Philippine Islands lying within the boundaries ofthe areas now or hereafter set apart and declared to be military reservationsshall be forthwith brought under the operations of the Land Registration Act,... .' "The conclusion is therefore inescapable that, as contended bypetitioners, respondent Judge is devoid of jurisdiction to pass upon the claimof private respondents invoking the benefits of Republic Act No. 931.

22(Nelson) hindi ko makita ung case ni marcos

23(JM)

REPUBLICVS.  JUDGE SINFOROSOFAÑGONIL

133SCRA 517

 

FACTS:After more than half a century fromthe 1922 decision declaring BaguioTownsiteReservation as public domain, Modesta Paris, Lagya Paris, SamuelBaliwan, PabloRamos, Jr., Josephine Abanag, Menita T. Victor, EmilianoBautista and OdiDianson filed with the Court of First Instance of Baguioapplications for theregistration of lots (with considerable areas) inside theBaguio TownsiteReservation. Alternatively, they allege that in case the lotsare

notregisterable under Act No. 496, then section 48(b) and (c) of thePublic LandLaw should be applied because they and their predecessors have beeninpossession of the lots for more than thirty years. The Director of lands ontheother hand contented that thedisposition of the said lots should be made by theDirector of Lands underChapter 11 of the Public Land Law regarding TownsiteReservations.

 

ISSUE:Whetheror not the applicants may file for the registration of lands covered bytheBaguio Townsite Registration.

 

HELD:No. Theapplicants do not base their applications under Act No. 496 on anypurchase orgrant from the State nor on possession since time immemorial. Thatis why ActNo. 496 cannot apply to them. The period of more than fifty yearscompletelybars the applicants from securing relief due to the alleged lack ofpersonalnotice to their predecessors. The law helps the vigilant but not thosewhosleep on their rights. “For time is a means of destroying obligations and actions,becausetime runs against the slothful and contemners of their own rights.”

24(Maris)

25(Reg)

26

G.R. No. 199310               February 19, 2014

REPUBLIC OF THE PHILIPPINES, Petitioner, vs. REMMAN ENTERPRISES, INC., represented by RONNIE P. INOCENCIO, Respondent.

 

FACTS:

December 3, 2001, Remman Enterprises, Inc. filed an application with the RTC for judicial confirmation of title over two parcels of land situated in Barangay Napindan, Taguig, MM. 10 days later, the RTC issued the Order finding REI’s application for registration sufficient in form and substance and set it for initial hearing on Feb. 21, 2002 – reset to May 30. Notice of Initial Hearing was published in the Official Gazette, People’s Balita, and likewise posted in a conspicuous place on the subject lots and bulletin board of the City hall of Taguig. Only Laguna Lake Development Authority appeared as oppositor, thus an order of general default was issued.

REI presented witness testimonies that they and its predecessors-in interest have been in open, continuous, exclusive and notorious possession of the said parcels of land long before June 12, 1945. The original possessor of the land, Veronica Jaime, cultivated and planted different kinds of crops in the said lots, through her caretakers and hired farmers.

The following documents were presented by REI: Deed of Absollute Sale dated August 28, 1989; survey plans of subject properties; technical description of subject properties; Geodetic Engineer’s Certificate; tax declarations for 2002; certification issued by Corazon Calamno, Senior Forest Management Specialist of the DENR attesting the lots form part of the alienable and disposable lands of the public domain.

RTC ruled on May 16, 2007 granting REI’s application for registration of title. This was affirmed by the CA.

ISSUE:

Whether the CA erred in affirming the RTC Decision dated May 16, 2007, which granted the application for registration filed by the respondent.

HELD:

Petition is meritorious.

Section 14(1) of PD 1529 provides that: Sec. 14. Who may apply. The following persons may file in the proper CFI an application for registration of title to land, whether personally or through their duly authorized representatives: (1) Those who by themselves or through their predecessors-in interest have been in open, continuous, exclusive and notorious possession and occupation of alienable and

disposable lands of the public domain under a bona fide claim of ownership since June 12, 1945, or earlier.

Section 14 of PD 1529 refers to the judicial confirmation of imperfect or incomplete titles to public land acquired under Section 48(b) of Commonwealth Act no. 141, or the Public Land Act as amended. Under Section 14(1), applicants for registration of title must sufficiently establish:

First, that the subject land forms part of the disposable and alienable lands of the public domain;

Second, that the applicant and his predecessors-in interest have been in open, continuous, exclusive and notorious (OCEN) possession and occupation of the same; and

Third, that it is under a bona fide claim of ownership since June 12, 1945, or earlier.

The first requirement was not satisfied in this case. To prove that the subject property forms part of the alienable and disposable lands of the public domain, the respondent presented two certifications, issued by Calamno attesting that the subject lots form part of the alienable and disposable lands of the public domain. This is insufficient. The Court clarified in Republic v. T.A.N. Properties, Inc. that in addition to the certification issued by the proper government agency that a parcel of land is alienable and disposable, applicants for land registration must prove that the DENR Secretary had approved the land classification and released the land of public domain as alienable and disposable. They must present a copy of the original classification approved by the DENR Secretary and certified as true copy by the legal custodian of the records. PENRO or CENRO certifications are not enough. Roche only presented the survey map and technical description of the land, bearing no info on the land’s classification.

For the second requirement, proof of specific acts of ownership must be presented to substantiate the claim of OCEN possession and occupation of the subject land. It consists of manifestation of acts of dominion over it of such a nature as a party would actually exercise over his own property. Mere casual cultivation of portions of the land does not constitute possession under claim of ownership.  

27(Ed)

28(LJ)

29(Zax)

30(Clathem)

Del Rosario-Igbiten V. Republic, GR No. 158449, 22 October 2004Facts: On 08 January 1998, petitioners filed with the trial court an application for registration of land under Presidential Decree (PD) No. 1529, otherwise known as the Property Registration Decree. The application covered a parcel of land in Silang, Cavite. Petitioners alleged that they acquired the Subject Property by purchase, and that they, by themselves and through their predecessors-in-interest, had been in actual, continuous, uninterrupted, open, public, and adverse possession of the Subject Property in the concept of owner for more than 30 years. Teodoro Calanog came into possession of the Subject Property in 1968. In the same year, the Subject Property was transferred to spouses Alfredo Tonido and Agatona Calanog. On 21 November 1995, the Tonido family sold the Subject Property to petitioners, as evidenced by a Deed of Absolute Sale. On 15 August 2000, the trial court rendered a decision approving petitioners application for registration of the Subject Property.Issue: whether or not the the decision approving petitioners application for registration of the subject property is valid.Held: No. petitioners have failed to comply with the period of possession and occupation of the Subject Property, as required by both the Property Registration Decree and the Public Land Act. It bears noting that, proceedings under the Property Registration Decree and the Public Land Act are the same in that both are against the whole world, both take the nature of judicial proceedings, and the decree of registration issued for both is conclusive and final. They differ mainly in that under the Property Registration Decree, there already exists a title which the court only needs to confirm. On the other hand, under the Public Land Act, there exists a presumption that the land applied for still pertains to the State, and that the occupants and possessors can only claim an interest in the land by virtue of their imperfect title or continuous, open, and notorious possession thereof. Nonetheless, in the end, the two laws arrive at the same goal, namely, a Torrens title, which aims at complete extinguishment, once and for all, of rights adverse to the record title.

31(Lea)

G.R. No. L-57461 September 11, 1987DIRECTOR OF LANDS vs. MERALCO FACTS:Manila Electric Company filed an amended application for registration of a parcel of land located in Taguig, Metro Manila on December 4, 1979. On August 17, 1976, applicant acquired the land applied for registration by purchase from Ricardo Natividad who in turn acquired the same from his father Gregorio Natividad as evidenced by a Deed of Original Absolute Sale executed on December 28, 1970 Applicant's predecessors-in-interest have possessed the property under the concept of an owner for more than 30 years. The property was declared for taxation purposes under the name of the applicant and the taxes due thereon have been paid.On May 29, 1981 respondent Judge rendered a decision ordering the registration of the property in the name of the private respondent. The Director of Lands interposed this petition.ISSUE: Whether a corporation may apply for registration of title to land. RULING: Yes.In the Acme decision (The Director of Lands v. Intermediate Appellate Court and Acme Plywood & Veneer Co., Inc., etc., No. L-73002 (December 29, 1986), 146 SCRA 509), this Court upheld the doctrine that open, exclusive and undisputed possession of alienable public land for the period prescribed by law creates the legal fiction whereby the land, upon completion of the requisite period ipso jure and without the need of judicial or other sanction, ceases to be public land and becomes private property.In the case at bar, if the land was already private at the time Meralco bought it from Natividad, then the prohibition in the 1973 Constitution against corporations holding alienable lands of the public domain except by lease (1973 Const., Art. XIV, See. 11) does not apply.Petitioner, however, contends that a corporation is not among those that may apply for confirmation of title under Section 48 of Commonwealth Act No. 141, the Public Land Act.As ruled in the Acme case, the fact that the confirmation proceedings were instituted by a corporation is simply another accidental circumstance, "productive of a defect hardly more than procedural and in nowise affecting the substance and merits of the right of ownership sought to be confirmed in said proceedings." Considering that it is not disputed that the Natividads could have had their title confirmed, only a rigid subservience to the letter of the law would deny private respondent the right to register its property which was validly acquired.

PartIV

1.SUSIv. RAZON

G.R.No.L-24066        December 9, 1925

 VILLA-REAL,J.:

FACTS:

               OnDecember 18,1880, Pinlacsold the land in question, then a fish pond, to Garciaand Mendoza. After havingbeen in possession thereof for about eight years,Garcia and Mendoza sold theland to Valentin Susi for the sum of P12. Thepossession and occupation of theland in question by different owners has beenopen, continuous, adverse andpublic, without any interruption, except duringthe revolution, or disturbance.However, on September 13, 1913, Angela Razoncommenced an action in the Court ofFirst Instance of Pampanga to obtain thesaid land but the court dismissed thecomplaint. Having failed in her attemptto obtain possession of the land inquestion through the court, Angela Razonapplied to the Director of Lands forthe purchase thereof. After making theproper administrative investigation, theDirector of Lands sold the land toAngela Razon. By virtue of said grant, Razonapplied for the proper certificateof title and the register of deeds ofPampanga issued thereafter issued it.

ISSUE:Whether or not Angela Razon mayapply for the registration of said land in herfavor.

HELD:

               No because she Susi isthelawful owner of the land. It clearly appears from the evidence thatValentinSusi has been in possession of the land in question openly, continuously,adversely,publicly, personally, and through his predecessors, since the year1880, thatis, for about forty-five years. Angela Razon is not allowed to applyfor thegrant in her favor since Valentin Susi had already acquired, byoperation oflaw, not only a right to a grant, but a grant of the Government.Consequently,the act of the Director of Deeds in selling the land over whichhe had no anytitle or control to Angela Razon was void and of no effect. Thus,Angela Razondid not thereby acquire any right.

2(Charleen)

3(Gina)

Priolovs Priolo, GR No. 37698, 71 Of. Gaz. 48, December 1, 1975

*casecannot be found. If you find it, please tell me so I can digest it for theclass. Thank you very much. :)

4(Zhon)

Republic vs. CA and Alejandro de Jesus

G.R. No. L-40912, September 30, 1976

Facts:

On January 22, 1921, Eugenio de Jesus, the father of respondent Alejandro de Jesus, applied with the Bureau of Lands for Sales Patent (Sales Application No. 5436) of a 33-hectare situated in barrio Libaron, Municipality of Davao (now Davao City). On November 23, 1934, the Director of Lands issued to Eugenio de Jesus an Order of Award. However, after the conduct of survey by the Bureau of Lands, the area was reduced to 20.6400 hectares.

On August 28, 1936, the Director of Lands ordered an amendment of the Sales Application of Eugenio de Jesus stating that "a portion of the land covered by Sales Application No. 5436 of Eugenio de Jesus is needed by the Philippine Army for military camp site purposes. The area excluded was identified as Lot 1176-B-2, the very land in question, consisting of 12.8081 hectares.

On September 7, 1936, President Manuel L. Quezon issued Proclaimation No. 85 withdrawing Lot No. 1176-B-2 from sale and settlement and reserving the same for military purposes, under the administration of the Chief of Staff, Philippine Army.

On May 15, 1948, then Director of Lands Jose P. Dans ordered the issuance of patent to Eugenio de Jesus. On the same date, then Secretary of Agriculture and Natural Resources Mariano Garchitorena granted the Sales Patent to Eugenio de Jesus.

On August 11, 1956, President Ramon Magsaysay issued Proclamation No, 328, revoking Proclamation No. 85 and declaring the disputed Lot 1176-B-2 open to disposition under the provisions of the Public land Act for resettlement of the squatters in the Piapi Beach, Davao City.

On October 9, 1956, President Magsaysay issued Proclamation No. 350, revoking this Proclamation No. 328 and reserving the same Lot No. 1176-B-2 for medical center site purposes under the administration of the Director of Hospital.

On December 6, 1969, petitioner Mindanao Medical Center applied for the Torrens registration of Lot 1176-B-2 with the CFI of Davao. Respondent Alejandro de Jesus, the son and successor-in-interest of sale applicant Eugenio de Jesus, opposed the registration.

On September 2, 1966, the CFI of Davao rendered judgment directing the registration of the title to Lot No. 1176-B-2 of in the name of the.

Upon appeal, CA held that Lot 1176-B-2 must be registered in the name of Alejandro de Jesus, but he is hereby ordered to relinquish to the Mindanao Medical Center that portion of Lot 1176-B-2 which is occupied by the medical center and nervous disease pavilion and their reasonable appurtenances.

Hence, this petition.

Issue: whether or not petitioner Mindanao Medical Center has registerabletitle over a full 12.8081-hectare land by virtue of an executiveproclamation in 1956 reserving the area for medical center sitepurposes

Held:

Petitioner has registerable title over the whole contested area of12.8081 hectares, designated Lot No. 1176-B-2, and not only on a portionthereof occupied by the Medical Center, its nervous disease pavilion andtheir reasonable appurtenances. Proclamation No. 350 issued by PresidentMagsaysay, which legally effected a land grant of the whole lot to theMindanao Medical Center, is sufficient for initial registration under the LandRegistration Act [Act 496]. Section 122 of the Act, provides that "Wheneverpublic lands in the Philippine Islands belonging to the Government of thePhilippines are alienated, granted, or conveyed to persons or to public orprivate corporations, the same shall be brought forthwith under theoperation of this Act and shall become registered lands.”

Further, section 64 (e) of the Revised Administrative Code empowersthe president "(t)o reserve from sale or other disposition and for specificpublic uses for service, any land belonging to the private domain of theGovernment of the Philippines, the use of which is not otherwise directed bylaw. The land reserved "shall be used for the specific purposes directed bysuch executive order until otherwise provided by law."

Similarly, Section 83 of the Public Land Act (CA 141) authorizes thePresident to "designate by proclamation any tract or tracts of land of thepublic domain as reservations for the use of the commonwealth of thePhilippines or of any of its branches, or of the inhabitants thereof, ... or forquasi-public uses or purposes when the public interest requires it, includingreservations for ... other improvements for the public benefit.”

5(May Ann)

6(Rhea B.)

LARAGAN VS CA

G.R. No. L-47644 August 21, 1987

Facts:

On 14 October 1968, the herein petitioners filed an application with the CFI of Isabela for the registration of their title over a parcel of land with an area of 221,667 sq. m., more or less, situated in the Barrio of Sto. Tomas, Ilagan, Isabela. The applicants alleged that they acquired said parcel of land by way of an absolute deed of sale from the spouses Anastacio and Lucrecia Sibbaluca and that they have been in possession thereof for more than 34 years. The Land Registration Commission issued a notice of initial hearing. On 7 July 1969, the Solicitor General filed a written opposition, on behalf of the Director of Lands, alleging that the applicants and their predecessor-in-interest do not have sufficient title to the parcel of land sought to be registered. He prayed that the land be declared public land. On 2 August 1969, Teodoro Leano, Tomas Leano, Vicente Leano, Francisco Leano, and Consolacion Leano filed their opposition to the application claiming that they are the owners, pro indiviso, of the southern part of the land applied for, with an area of 16 hectares of their deceased parents and which has been in their possession for more than 30 years.

The trial court rendered judgment confirming the title of the applicants over the parcel of land applied for and ordering its registration in the names of the applicants. The oppositors appealed to the CA. On 9 November 1977, the appellate court affirmed the judgment of the trial court, but excluded the southern portion of the land applied for, the appellate court declaring such excluded portion to be public land, and part of the public domain, in view of

the failure of the applicants and oppositors to prove registrable title over the same. The petitioners filed a motion for reconsideration of the decision but their motion

was denied.

Issue:

Whether the appellate court acted without or in excess of jurisdiction in declaring the parcel of land in question as public land.

.

Held:

The argument is untenable. While it may be true that the Director of Lands did not appeal from the decision of the trial court, his failure to so appeal did not make the decision of the trial court final and executory, in view of the appeal interposed by the other oppositors, Teodoro Leano, Tomas Leano, Francisco Leano, and Consolacion Leano, who also seek the confirmation of their imperfect title over the land in question. Neither did such failure of the Director of Lands to appeal foreclose the appellate court from declaring the land in question to be public land, since the oppositors and the herein petitioners are both seeking the registration of their title pursuant to the provisions of Section 48 (b) of the Public Land Law where the presumption always is that the land pertains to the state, and the occupants and possessors claim an interest in the same, by virtue of their imperfect title or continuous, open, exclusive and notorious possession and occupation under a bona fide claim of ownership for the required number of years.

Besides, it is an established rule that an applicant for registration is not necessarily entitled to have the land registered in his name simply because no one appears to oppose his title and to oppose the registration of the land. He must show, even in the absence of opposition, to the satisfaction of the court, that he is the absolute owner, in fee simple. Courts are not justified in registering property under the Torrens system, simply because there is no opposition offered. Courts may, even in the absence of any opposition, deny registration of the land under the Torrens system, if the facts presented do not show that the petitioner is the owner, in fee simple, of the land which he seeks to register.

The petition is denied for lack of merit.

7(Angel)

8(Janine)

9 Kidpalos vs Baguio Gold MiningCompany 14 SCRA 913

 

Facts:    Petitioners sued the Baguio Gold Mining Companyand the director of Mines in the court of First Instance in Baguio, seekingjudgement declared said plaintiffs to be the owners of certain parcels of landsituated in Sitio Binanga, Barrio of Tuding, Itogon, Benguet, Mt. Province, to annulthe declarations of location of certain mineral claims of the Baguio GoldMining Company, overlapping the parcels claimed by plaintiffs, and to recoverdamages from the company. The complaint also sought to enjoin the director ofmines from proceeding with the lode patent applications of the Mining Companyand to have the mine buildings erected on the land in question demolished atthe latters expense. The defendant BGMC, claiming the virtue of valid locationsof the claims since 1925 to 1930.

 

Issue:   WoN an applicant who was previously deniedclaims of ownership in revindicatory action cannot file for registration ofsame land involved.

Held:    If the record of theformer trial shows that the judgement could not have been rendered withoutdeciding the particular matter, it will be considered as having settled thatmatters as to all future actions between the parties, and if a judgement necessarilypresupposes certain premises, they are as conclusive as the judgement itself.

                Since there can beno registration of land without applicant being its owner, the final judgementof CA in the previous litigationdeclaring that the mining company’s title is superior to that of appellantsshould be conclusive on the question in the present case.

10(Lou)

SPOUSESFRANCISCO LAHORA and TORIBIA MORALIZON, vs. EMILIODAYANGHIRANG, JR., and THE DIRECTOR OF LANDS,

(37SCRA 346)

G.R.No. L-28565 (January 30, 1971)

FACTS:On 26 November 1965, appellants petitioned the Court of First Instance of Davaofor registration of nine (9) parcels of land located in barrio Zaragosa,municipality of Manay, province of Davao. Appellant (Toribia Moralizon) alleged one-half of the parcels of land having been acquired by inheritance,and the other half by purchase and by continuous, open, public and adversepossession in the concept of owner. One of the said parcels of land isidentified as lot No. 2228, plan SWO-36856, Manay Cadastre.

Oppositorsalleged that lands belonging to him and his wife were included in theapplication for registration, mentioning specifically Lot No. 2228 which wassaid to be already covered by Original Certificate of Title no.P-6055 in thename of oppositor's wife on 21 June 1956. Thus, Oppositors filed a motion forcorrection of the number of the certificate of title covering Lot No. 2228,erroneously referred to as OCT No. P-6055, when it should properly be OCT No.P-6053 and also prayed in the same motion that the petition be dismissed.

TheDirector of Lands also filed an opposition to the petition, contending that theapplicants or their predecessors-in-interest never had sufficient title overthe parcels of land sought to be registered, nor have they been in open,continuous, and notorious possession thereof for at least 30 years.

TheCourt of First Instance of Davao (in Land Reg. Case No. N-86), dismissing theirpetition with respect to Lot No. 2228 on the ground of previous registration,said appellants claiming that the question of the validity of a certificate oftitle based on a patent allegedly obtained by fraud can be raised by them in aland registration proceeding, contrary to the ruling of the court a quo.

ISSUE:W/N Lot No. 2228 can be the subject of two registration proceedings.

RULING:No, Lot No. 2228 cannot be the subject of two registration proceedings.

Inthe present case, Lot No. 2228 was registered and titled in the name ofoppositors' wife as of 21 June 1956, nine (9) years earlier. On 26 November1965, the appellants' petition for registration of the same parcel of landbased on the ground that the first certificate of title (OCT No. P-6053)covering the said property is a nullity, can no longer prosper.

Orderlyadministration of justice precludes that Lot 2228, of the Manay Cadastre,should be the subject of two registration proceedings. Having become registeredland under Act 496, for all legal purposes, by the issuance of the public landpatent and the recording thereof, further registration of the same would leadto the obviously undesirable result of two certificates of title being issuedfor the same piece of land, even if both certificates should be in the name ofthe same person. And if they were to be issued to different persons, theindefeasibility of the first title, which is the most valued characteristic ofTorrens titles, would be torn away.

Forthis reason, this Court has ruled in Pamintuan vs. San Agustin, 43Phil. 558, that in a cadastral case the court has no jurisdiction to decreeagain the registration of land already decreed in an earlier case; and that asecond decree for the same land would be null and void. Of course, if thepatent had been issued during the pendency of the registration proceedings, thesituation would be different.

WHEREFORE,finding no error in the order appealed from, the same is hereby affirmed, withcosts against the appellants.

Significance:Land Registration Act; Character of certificate of title for public landpatents. -- The rule in this jurisdiction,regarding public land patents and the character of the certificate of titlethat may be issued by virtue thereof, is that where land is granted by thegovernment to a private individual, the corresponding patent therefor isrecorded, and the certificate of title is issued to the grantee becomingentitled to all safeguards provided to the grantee becoming entitled to all thesafeguards provided in Section 38 of the land Registration Act, the titleissued to the grantee becoming entitled to all the safeguards provided inSection 38 of the said Act. In other words, upon expiration of one year fromits issuance, the certificate of title shall become irrevocable andindefeasible like a certificate issued in a registration proceeding.

Periodfor review of decree; Proper party and relief after expiration of period.-- Even assuming arguendo that there indeed exists a proper case forcancellation of the patent for intrinsic fraud, the action for review of thedecree should have been filed before the one year period had elapsed.Thereafter, the proper party to bring the action would only be the personprejudiced by the alleged fraudulent act -- the owner and grantor -- and notanother fraudulent act -- the owner and grantor -- and not another applicant orclaimant. Furthermore, the relief provided by the law in such instance may besecured by the aggrieved party, not in another registration, for land alreadyregistered in the name of a person cannot the subject of another registration,but in an appropriate action such as one for reconveyance or reversion, or fordamages in case the property has passed into the hands of an innocent purchaserfor value.

Correctionof typographical mistake in title number in motion.-- As regards the complaint against the alleged correction of the number of thecertificate of title covering Lot No. P-6053, it appearing that the motion wasintended to rectify a clearly typographical mistake, there is nothing irregularin the lower court's order granting the same.

NOTES.-- Effect of registration of administrative land patent. -- Aftera free patent application is granted and the corresponding certificate of titleis issued, the land covered thereby ceases to be a part of the public domainand becomes private property over which the Director of Lands has neithercontrol nor jurisdiction. (Sumail vs. Judge of First Instance of Cotobato,L-8278, April 30, 1955, 51 O.G. 2413)

 

 

         A certificate of title issued on registration of a homestead patent partakes ofthe nature of a certificate in judicial proceedings as long as the land isreally part of the disposable land of the public domain, and the certificatebecomes indefeasible and incontrovertible upon the expiration of one year fromits date of issuance ( Lucas vs. Durian, L-7886, Sept. 23, 1957). If theland was private, the grant is a nullity (Vital vs. Anore, L-4136, Feb. 29,1952, 53 O.G. 3739).

 

 

         As stated in Acierto vs. De los Santos, L-5828, Sept. 29, 1954,once a homestead patent is registered, as required by, the land become"registered land" within the meaning of Act no. 496 as to whichadverse title cannot be acquired by prescription.

         A homestead acquired in public land become registered land as soon as thepatent issued to the homesteader is recorded in the registry of deeds and thecertificate of title issued, thereafter being subject to the same rights andprivileges as other registered lands (Eugenio vs. Perdido, L-7083, May 19,1955)

11(Jess)

12(Diane)

ATOK BIG-WEDGE MINING COMPANY, PETITIONER, VS. HON. INTERMEDIATE APPELLATE COURT and TUKTUKAN SAINGAN

 [G.R. No. 63528. September 9, 1996]

 

FACTS: Atok Big Wedge Company and Tuktuktan Saingan are both claiming ownership over subject land situated in the barrio of Lucnab, Itogon, Benguet.  Atok Big Wedge Mining Company contended that the said parcel of land was being registered in the office of Mining Recorder in 1921 and 1931 pursuant to Philippine Bill of 1902. It is about sixteen years before TUKTUKAN declared the land in question for taxation purposes and thirty four (34) years before private respondent filed the land registration proceedings in 1965. They also showed the payment of annual assessment fees for the said land since 1931. They also claim that it is a mineral land. Tuktukan who was 70 years old at the time he testified shows that he acquired the land from his father-in-law, Dongail, when he married his daughter; that he was then 18 years old; that at the time of his acquisition, it was planted with camotes, casava, langka, gabi, coffee and avocados; that he lived on the land since his marriage up to the present; that he has been paying the taxes during the Japanese occupation and even before it; that he was never disturbed in his possession.  Supporting his oral testimony, he submitted tax declarations both dated March 20,

1948, the former for a rural land and the latter for urban land and improvement therein.

ISSUE: Whether or not Atok Wedge Mining Company has a better right on the subject land.

HELD: No. The SC ruled in favor of the Tuktukan because he has proven that he had in possession of the said land in a concept of an owner, continuously, open and uninterrupted for a period of more than 30 years. He had improved also almost 90% of the said parcel of land. He also had paid tax declaration of the said land since 1948 up to present. SC also said that payment of annual assessment fee is not enough proof. There must be an annual performance of labor or undertaking of improvements in the mine. When an ocular survey was made, it was evident that there was no improvements being made in the said land and there is any sign of mining had happened in the land.

13(April)

14(Daniel Eblahan)

Ignaciov. Director of Lands

GRno. L-12958, May 30, 1960

Montemayor,J.:

Facts:

               On January 25, 1950, FaustinoIgnacio applied to register a parcel of mangrove land in Gasac, Navotas, Rizal.This parcel of land, formed by accretion and alluvial deposits caused by theaction of the Manila Bay, adjoins a parcel of land acquired by Ignacio from thegovernment in 1936 and has occupied the land since

1935. The Director of Landsopposed this application because the land is public domain, a foreshore land.The trial court dismissed the application, ruling that the parcel of land isforeshore land.

Issue:

               Whether or not the parcel ofland in question is public domain

Held:

               Yes. The Court affirmed thetrial court in ruling that the land is public domain. The Court ruled that theland formed by the actions of the sea is property of the State and that the lawon accretion only applies to lands that are situated on the banks of rivers.The court disagreed with the petitioner that Manila bay is not a sea. It is apart of the sea, a mere indentation of the same.

               In this case, the petitionercannot claim private ownership of the land because the land in question is notsituated in a river bank, it is situated in Manila Bay. The alluvial depositsand accretion was caused by the actions of the Manila Bay, which is a part ofthe sea. The land formed is thus, part of the public domain.

15(Melodia)

_________________________________________________________________________________________________

16(Mel)

REPUBLIC VS. DE PORKAN

G.R. No. L-66866

June 18, 1987, 151 SCRA 88 

- Possession resulting in presumption of right to grant application 

FACTS

Minda de Porkan and Lolita Macatindog acquired Lots Nos. 1099 and 1546 from their predecessors-interests, who in turn acquired said lots though a grant by the government by virtue of their proven, open, exclusive and undisputed possession for more than 30 years. An issue over said lots arose when a certain Viola Azurin obtained from the then Philippine Fisheries Commission an Ordinary Fishpond Permit covering portions of Lots Nos. 1099 and 1546. Azurin filed with the Bureau of Lands a complaint for correction, amendment or cancellation of the Homestead Patent of De Porkan over Lot no. 1546 and the Free Patent of Macatindog over Lot No. 1099 alleging among others that the patentees secured their patents and titles through fraud, misrepresentation and illegal machinations. The Solicitor General sided with Azurin; when the case was brought to the Court of First Instance, the SG stated that the disputed portions of land were actually claimed by Azurin and that such lands could not be disposed by the Director of Lands under the Public Land Act. Hence, the patents and titles issued to de Porkan and Macatindog were void in so far as the portion occupied and covered by the fishpond permit of Azurin. After hearing however, the CFI dismissed the complaints and upheld the validity of the titles/patents of de Porkan & Macatindog over the lands in dispute. The SG in the present petition avers among others that the lots in dispute could not be the subject of disposition under the Homestead and Free Patent provisions of the Public Act since they are marshy and swampy, certified as such as more suitable for fishpond development, disposable only thru lease under the Public Land Act.

ISSUE:

Whether or not possession and cultivation of a land for more than 30 years will entitle the possessor thereof of a government grant and a certificate of title.

 

HELD:

Yes. As early as 1953, the respondents had already acquired by operation of law not only a right to a grant over Lot No. 1099, but a grant of the Government over the same alienable land by virtue of their proven, open, exclusive and undisputed possession for more than 30 years, since the Spanish colonial period.

The possession of a public land identified as Lot No. 1099 dates back to the time of the Spanish colonial period. Such possessions of the said public land has attained the character and duration prescribed by law as the equivalent of an express grant from the Government. The mandate of the law itself provides that possessors “shall be conclusively presumed to have performed all the conditions essential to a

government grant and shall be entitled to a certificate of title”. By legal fiction, the land ceases to be public and thus becomes a private land.

__________________________________________________________________________________________________

17(Rocky)

18(Maribeth)

REPUBLICOF THE PHILIPPINES vs CAet. al., 99 SCRA 742

FACTS:

OnMay 16, 1966, PerpetuoAlpuerto, filed with the CFI of Quezon (Branch II), amotion to reopenCadastral Case No. 97, LCR Cad. Rec. No. 1555, and to admit hisanswer over LotNo. 7718 of the Mauban (Quezon) cadastre. After trial, the lowercourt renderedits decision adjudicating to said Alpuerto Lot No. 7718 togetherwith itsimprovements, and ordered the issuance in his favor of thecorresponding decreeof registration, thus an order for the issuance of a decreeof registrationover the said lot. The Land RegistrationCommission then issuedDecree No. 127177 as the basis of the issuance of OCTNo. 0-13541 in his favor.Portions of the lot were subsequently transferred tovarious persons who wereissued their respective transfer certificates oftitle, among whom are privaterespondents.

OnApril 6, 1971, the Sol.Gen. filed for the government a complaint for annulment,cancellation of titlesand for reversion of Lot No. 7718 of Cadastral Survey ofMauban, Quezon to theState on the ground that the decision of the court adjudicatingLot No. 7718 toAlpuerto, its order for the issuance of the decree ofregistration as well asOCT No. 013541 and all its derivative titles are allnull and void and withoutlegal effect because the court had no jurisdiction toallocate the subjectland, which is inalienable.

HereinRespondents IndustrialMarketing and Investments Corporation, Henry O. AntonioAngeles and AureaAngeles filed a motion to dismiss the complaint alleging thatthe action isbarred by a prior judgment and that the court lacks jurisdictionover thenature of the action or suit. Defendant, Andres Laredo, likewise fileda motionfor dismissal of the complaint, based on the grounds that the complaintstatesno cause of action and that venue is improperly laid.

Afterhearing the motionsfor dismissal and the opposition thereto, the lower courtissued an orderdenying the motion to dismiss filed by Respondent Laredo, butgranting themotion to dismiss filed by Respondents Industrial Marketing andothers anddismissing the complaint filed by petitioner. The motion forreconsideration was further denied. Consequently,petitioner appealed the caseto the CA which affirmed the order of dismissal bythe lower court. The motionfor reconsideration was likewise denied, hence,this petition.

ISSUE: Whetheror not thecourt had jurisdiction to allocatethe subject land, which isinalienable?

HELD: No.It was held that where the land covered by the homestead applicationofpetitioner was still within the forest zone or under the jurisdiction of theBureauof Forestry, the Director of Lands had no jurisdiction to dispose of saidlandunder the provisions of the Public Land Law, and the petitioner acquirednoright to the land. It follows that "if a person obtains a title underthePublic Land Act which includes, by oversight, lands which cannot beregisteredunder the Torrens system, or when the Director of Lands did nothavejurisdiction over the same because it is a public forest, the grantee doesnot,by virtue of the said certificate of title alone, become the owner of thelandillegally included". The title thus issued is void at law, sincetheofficer who issued it had no authority to do so.

19(Ailyn)

20(Jen)

G.R.No. L-26127 June 28, 1974 Benin vs. Tuason, SupraFACTS:The plaintiffs in these three civil cases uniformly alleged, in theirrespective complaint, that sometime in the year 1951 while they were enjoyingthe peaceful possession of their lands, the defendants, particularly thedefendant J.M. Tuason and Co. Inc., through their agents and representatives,with the aid of armed men, by force and intimidation, using bulldozers andother demolishing equipment, illegally entered and started defacing,demolishing and destroying the dwellings and constructions of plaintiffs'lessees, as well as the improvements. They made inquiries regarding theprobable claim of defendants, and in 1953 they discovered for the first timethat

their lands, as described in their respective complaint, had either beenfraudulently or erroneously included, by direct or constructive fraud, in whatappears as Parcel No. 1 (known as Santa Mesa Estate) in Original Certificate ofTitle No. 735 of the Land Records of the province of Rizal in the names of theoriginal applicants for registration, now defendants, Mariano Severo Tuason yde la Paz, Teresa Eriberta Tuason y de la Paz, Juan Jose Tuason y de la Paz,Demetrio Asuncion Tuason y de la Paz, and Augusto Huberto Tuason y de la Paz.The plaintiffs in each of the three complaints also alleged that the registeredowners had applied for the registration of two parcels of land (known as theSanta Mesa Estate and the Diliman Estate; that the registration proceedingswere docketed as LRC No. 7681 of the Court of Land Registration; They allegethat the application for registration in LRC No. 7681, containing theboundaries, technical descriptions and areas of parcel No. 1 (Santa MesaEstate) and parcel No. 2 (Diliman Estate) was published in the OfficialGazette; that before the decision was handed down in LRC No. 7681, the area,boundaries and technical descriptions of parcel No. 1 were altered and amended;that the area of parcel No. 1 as mentioned in Decree No. 17431 is bigger thanthe area of parcel No. 1 appearing in the application for registration aspublished in the Official Gazette; that the amendments and alterations, whichwere made after the publication of the original application, were neverpublished; that on March 7, 1914 a decision was rendered in LRC No. 7681 basedon the amended plan; that pursuant to the decision of March 7, 1914 a decree ofregistration was issued on July 6, 1914, known as Decree No. 17431, decreeingthe registration in the names of the applicants of the two parcels of land(Santa Mesa Estate and Diliman Estate).Issue: W/N the LRC hadjurisdiction to render the decision for the reason that the amendment to theoriginal plan was not published. HELD: We believe that the lower courterred when it held that the Land Registration Court was without jurisdiction torender the decision in LRC No. 7681. Under Section 23 of Act 496, theregistration court may allow, or order, an amendment of the application forregistration when it appears to the court that the amendment is necessary andproper. Under Section 24 of the same act the court may at any time order an applicationto be amended by striking out one or more parcels or by severance of theapplication. The amendment may be made in the application or in the surveyplan, or in both, since the application and the survey plan go together. If theamendment consists in the inclusion in the application for registration of anarea or parcel of land not previously included in the original application, aspublished, a new publication of the amended application must be made. Thepurpose of the new publication is to give notice to all persons concernedregarding the amended application. Without a new publication the registrationcourt can not acquire jurisdiction over the area or parcel of land that isadded to the area covered by the original application, and the decision of theregistration court would be a nullity insofar as the decision concerns thenewly included land. The reason is because without a new publication, the lawis infringed with respect to the publicity that is required in registrationproceedings, and third parties who have not had the opportunity to presenttheir claim might be prejudiced in their rights because of failure of notice.But if the amendment consists in the exclusion of a portion of the area coveredby the original application and the original plan as previously published, anew publication is not necessary. In the latter case,

the jurisdiction of thecourt over the remaining area is not affected by the failure of a newpublication.

22(Nelson)

ROXASVS. CA

G.R.No. 118436

March21, 1997

FACTS: 

OnJuly 1990, herein private respondent Maguesun Management and DevelopmentCorporation (Maguesun Corporation) filed an Application for Registration of twoparcels of unregistered land located in Tagaytay City. In support of itsapplication for registration, Maguesun Corporation presented a Deed of AbsoluteSale dated June 10, 1990, executed by Zenaida Melliza as vendor and indicatingthe purchase price to be P170,000.00. Zenaida Melliza in turn, bought theproperty from the original petitioner herein (because she was substituted byher heirs in the proceedings upon her death), Trinidad de Leon vda. de Roxasfor P200,000.00 two and a half months earlier, as evidenced by a Deed of Saleand an Affidavit of Self-Adjudication.

Noticesof the initial hearing were sent by the Land Registration Authority (LRA) onthe basis of Maguesun Corporation’s application for registration enumeratingadjoining owners, occupants or adverse claimants; Since Trinidad de Leon vda.de Roxas was not named therein, she was not sent a notice of the proceedings.After an Order of general default was issued, the trial court proceeded to hearthe land registration case. Eventually, on February 1991 the RTCgranted Maguesun Corporation’s application for registration.

Itwas only when the caretaker of the property was being asked to vacate the landthat petitioner Trinidad de Leon Vda. de Roxas learned of its sale and theregistration of the lots in Maguesun Corporation’s name.

Hence,on April 1991, petitioner filed a petition for review before the RTC to setaside the decree of registration on the ground that Maguesun Corporationcommitted actual fraud. She alleged that the lots were among the properties sheinherited from her husband, former President Manuel A. Roxas and that her familyhad been in

open, continuous, adverse and uninterrupted possession of thesubject property in the concept of owner for more than thirty years before theyapplied for its registration under the Torrens System of land titling (in whichno decision has been rendered thereon). Petitioner further denied that she soldthe lots to Zenaida Melliza whom she had never met before and that hersignature was forged in both the Deed of Sale and the Affidavit ofSelf-Adjudication. She also claimed that Maguesun Corporation intentionallyomitted her name as an adverse claimant, occupant or adjoining owner in theapplication for registration submitted to the LRA such that the latter couldnot send her a Notice of Initial Hearing.

Adocument examiner from the PNP concluded that there was no forgery.Uponpetitioner’s motion, the signatures were re-examined by another expert fromNBI. The latter testified that the signatures on the questioned and sampledocuments were, however,  not written by the same person.

Despitethe foregoing testimonies and pronouncements, the trial courtdismissed thepetition for review of decree of registration. Placing greater weight on thefindings and testimony of the PNP document examiner, it concluded that thequestioned documents were not forged and if they were, it was Zenaida Melliza,and not Maguesun Corporation, who was responsible. Accordingly, MaguesunCorporation did not commit actual fraud.

Ina decision dated December 8, 1994, respondent court deniedthe petition for review and affirmed the findings of the trial court. TheCA held that petitioner failed to and demonstrate that there was actualor extrinsic fraud, not merely constructive or intrinsic fraud, a prerequisitefor purposes of annuling a judgment or reviewing a decree of registration.

Hence,the instant petition for review where it is alleged that the CA erred in rulingthat Maguesun Corporation did not commit actual fraud warranting the settingaside of the registration decree and in resolving the appeal on the basis ofMaguesun Corporation’s good faith. Petitioners pray that the registration ofthe subject lots in the name of Maguesun Corporation be cancelled, that saidproperty be adjudicated in favor of petitioners and that respondent corporationpay for damages.

ISSUE: WONprivate respondent Maguesun Corporation committed actual fraud (signatureforgery) in obtaining a decree of registration over the two parcels ofland, actual fraud being the only ground to reopen or review a decreeof registration.

HELD: WHEREFORE,the instant petition is hereby GRANTED. The Decision of the CA is herebyREVERSED AND SET AS

1.The Court here finds that respondent Maguesun Corporation committed actualfraud in obtaining the decree of registration sought to be reviewed bypetitioner. A close scrutiny of the evidence on record leads the Court to theirresistible conclusion that forgery was indeed attendant in the case at bar.Although there is no proof of respondent Maguesun Corporation’s directparticipation in the execution and preparation of the forged instruments, thereare sufficient indicia which proves that Maguesun Corporation is not the“innocent purchaser for value” who merits the protection of the law. Even to alayman’s eye, the documents, as well as the enlarged photographic exhibit of thesignatures, reveal forgery. Additionally, Zenaida Melliza’s non-appearanceraises doubt as to her existence

Petitionerand her family also own several other pieces of property, some of which areleased out as restaurants. This is an indication that petitioner is not unawareof the value of her properties. Hence, it is unlikely that indication that shewould sell over 13,000 sqm of prime property in Tagaytay City to a stranger fora measly P200,000.00. Would an ordinary person sell more than 13,000 sqm of primeproperty for P170,000.00 when it was earlier purchased for P200,000.00?

3.Petitioner Vda. de Roxas contended that Maguesun Corporation intentionallyomitted their name, or that of the Roxas family, as having a claim to or as anoccupant of the subject property.

Thenames in full and addresses, as far as known to the undersigned, of the ownersof all adjoining properties; of the persons mentioned in paragraphs 3 and 5(mortgagors, encumbrancers, and occupants) and of the person shown onthe plan (original application submitted in LRC No) asclaimants are as follows:

HilarioLuna, Jose Gil, Leon Luna, Provincial Road

allat Tagaytay City (no house No.) 30

Thehighlighted words are typed in with a different typewriter, with the first fiveletters of the word “provincial” typed over correction fluid. MaguesunCorporation, however, annexed a differently-worded application for thepetitionto review case. In the copy submitted to the trial court, the answer to thesame number is as follows:

HilarioLuna, Jose Gil, Leon Luna, Roxas.

Thediscrepancy which is unexplained appears intentional. If the word “Roxas” wereindeed erased and replaced with “Provincial Road all at Tagaytay City (no houseNo.)” in the original application submitted in LRC No. TG-373 BUT thecopy with the word “Roxas” was submitted to the trial court, it is reasonableto assume that the reason is to mislead the court into thinking that “Roxas”was placed in the original application as an adjoining owner, encumbrancer,occupant or claimant, the

same application which formed the basis for the LRAAuthority in sending out notices of initial hearing. (Section 15 of PD No. 1529actually requires the applicant for registration to state the full names andaddresses of all occupants of the land and those of adjoining owners, if knownand if not known, the extent of the search made to find them. Respondentcorporation likewise failed to comply with this requirement of law.)

Respondentcorporation’s intentional concealment and representation of petitioner’sinterest in the subject lots as possessor, occupant and claimantconstitutesactual fraud justifying the reopening and review of the decree of registration.Through such misfeasance, the Roxas family was kept ignorant of theregistration proceedings involving their property, thus effectively deprivingthem of their day in court

Thetruth is that the Roxas family had been in possession of the property uninterruptedlythrough their caretaker, Jose Ramirez. Respondent Maguesun Corporationalso declared in number 5 of the same application that the subject land wasunoccupied when in truth and in fact, the Roxas family caretaker resided in thesubject property.

Toconclude, it is quite clear that respondent corporation cannot tack itspossession to that of petitioner as predecessor-in-interest. Zenaida Mellizaconveyed not title over the subject parcels of land to Maguesun Corporation asshe was not the owner thereof.  Maguesun Corporation is thus notentitled to the registration decree which the trial court granted in itsdecision.

Petitionerhas not been interrupted in her more than thirty years of open, uninterrupted,exclusive and notorious possession in the concept of an owner over the subjectlots by the irregular transaction to Zenaida Melliza.She therefore retainstitle proper and sufficient for original registration over the twoparcels of land in question pursuant to Section 14 of PD No. 1529.

23(JM)

Beninv. Tuason

G.R.No. L-26127     June 28, 1974

Facts:

 Theplaintiffsalleged that they were the owners and possessors of the threeparcels ofagricultural lands, described in paragraph V of the complaint,located in thebarrio of LaLoma (now barrio of San Jose) in the municipality(now city) ofCaloocan, province of Rizal, that they inherited said parcels ofland from theirancestor Sixto Benin, who in turn inherited the same from hisfather, EugenioBenin; that they and their predecessors in interest hadpossessed these threeparcels of land openly, adversely, and peacefully,cultivated the same andexclusively enjoyed the fruits harvested therefrom;that Eugenio Benin,plaintiff’s grandfather, had said parcels of land surveyedon March 4 and 6,1894, that during the cadastral survey by the Bureau of Landsof the lands inBarrio San Jose in 1933 Sixto Benin and herein plaintiffs claimthe ownershipover said parcels of land; that they declared said lands fortaxation purposesin 1940 under Tax Declaration No. 2429; that after theoutbreak of the lastWorld War, or sometime in 1942 and subsequentlythereafter, evacuees from Manilaand other places, after having secured thepermission of the plaintiffs,constructed their houses thereon and paid monthlyrentals to plaintiffs. Onlydefendant J.M. Tuason & Co., Inc. was actuallyserved with summons. Theother defendants were ordered summoned by publicationin accordance withSections 16 and 17of the Rules of Court. Only defendant J.M.Tuason & Co.,Inc. appeared. The other defendants were all declared indefault.

Issue:

Doesan amendment due to exclusion does not requirea publication?

HELD:

Yes,it is presumed, therefore, that as occupantsproper notices thereof were servedon them and that they were aware of saidproceedings. . Therefore, thatthedecision of this Court, which affirmed the order of the Court of FirstInstanceof Rizal dismissing the complaint of Jose Alcantara, Elias Benin andPascualPili(along with four other plaintiffs) should apply not only against theheirs,of Elias Benin, against Jose Alcantara, and against Pascual Pili, asplaintiffsin Civil Cases Nos. 3621, 3622and 3623, respectively, but alsoagainst all theother plaintiffs in those cases. We find that the plaintiffs donot claim aright which is different from that claimed by Elias Benin.

24(Maris)

25(Reg)

26

G.R. No. L-32398               January 27, 1992

IN THE MATTER OF THE PETITION OF PO YO BI TO BE ADMITTED AS CITIZEN OF THE PHILIPPINES: PO YO BI, petitioner-appellee, vs. REPUBLIC OF THE PHILIPPINES, respondent-appellant.

FACTS:

On February 9, 1957, Po Yo Bi filed a petition for naturalization. On March 5, 1959, the trial court, through the Deputy Clerk of Court, issued a Notice of Petition for Philippine Citizenship setting the hearing of the petition to January 18, 1960 and ordered the publication and posting of the notice. Po Yo Bi filed for an amendment of information. An Amended Notice of Petition was issued on January 18, 1960 setting the hearing of the petition to October12, 1960 and ordering the publication of the said notice once a week for three consecutive weeks in the Official Gazette and in the YUHUM, newspaper of general circulation in the prov/city of Iloilo. Another amendment was filed (without claiming good moral character) and the Deputy Clerk of Court issued another Amended Notice of Petition for Philippine Citizenship setting the hearing to February 26, 1962 and directing publication of the order in the Official Gazette and in the Guardian, newspaper of gen. circulation in the prov/city of Iloilo. HOWEVER, the second amended petition ITSELF was not published or posted in a public and conspicuous place in the Office of the Clerk of Court or in the building where such office is located. The petition was granted after trial on October 15, 1963.

The Assistant City Fiscal of Iloilo, Vicente Gengos, on behalf of the SolGen, filed a motion to reconsider as Po Yo Bi had not complied with Sec.4 of the Revised Naturalization Law on filing his declaration of intention.

ISSUE:

Whether the court erred in not finding that the amended petition for naturalization was not published in accordance to the requirements of section 9 of CA 473, as amended.

HELD:

The second amended petition was not published. Neither were the original and the amended petitions. What the Office of the Clerk of Court did was to prepare and issue NOTICES of petition. IT was said notices alone which were ordered to be published and posted.

Section 9 of the Revised Naturalization Law requires that the petition itself must be published. This provision demands the compliance of the following requirements, namely: (1) the publication must be weekly; (2) it must be made three times; (3) and these must be consecutive. The Court further ruled that the publication is a jurisdictional requirement. Thus: “In short, non-compliance with the requirements thereof, relative to the publication of the petition, affects the jurisdiction of the court. It constitutes a FATAL defect, for it impairs the very root or foundation of the authority to decide the case, regardless of whether the one to blame therefor is the clerk of court or the petitioner or his counsel. Failure to raise this question in the lower court would not cure such defect.”

The Court held that the requirement that a copy of the petition to be posted and published should be textual or verbatim restatement of the petition as filed is jurisdictional. Non-compliance therewith nullifies the proceedings in the case, including the decision rendered in favor of the applicant.

27(Ed)

28(LJ)

29(Zax)

30(Clathem)

LEYVA V. JANDOC, 4 SCRA 595

FACTS: On September 10, 1958, ManuelaJ andoc applied, in the Court of First Instance of Cotabato, for theregistration of three (3) parcels of land situated in Cotabato. Eligio T. Leyvaobjected thereto on December 10, 1958, with respect to a portion of said landof about one (1) hectare which he claimed to have adversely possessed in goodfaith and under legal title since 1937. On or about December 17, 1958 his wife Eufemia L. Leyva filed another opposition alleging that she and her husband hadoccupied a portion of the land in question, which they had acquired from thedefunct "NARRA", and that both had similarly occupied as owners since 1937, another portion of said land of about 88 meters by 6.66 meters,"with their improvements ... made in good faith." Later, otheroppositions were filed, also, including one by the Bureau of Lands, whichclaimed the land applied for as part of the public domain.

The Court of Appeals denied the opposition of Leyvas declaring that the claim oft he Leyvas may be recognized only if the land in question is public land, and sincethe Director of Lands has already filed an opposition based on the ground that the land sought to be registered is public land, the court ruled that theLeyvas have no right to appear as independent oppositors, but could collaborate with the provincial fiscal. Leyva's counsel was allowed to stay in court to collaborate, as in fact he did collaborate, with the provincial fiscal.

Issue: whether or not the opposition of Leyvas will prosper.

Held: No.

The law provides that any person claiming any kind of interest to file an opposition to an application for registration must be based on a right of dominion or some other real right independent of, and not at all subordinate to, the rights of the Government.

However,in the case, since the petitioners are foreshore lessees of public land, their right is completely subordinate to the interests of the Government, and must necessarily be predicated upon the property in question being part of the public domain.

Invariably,it is incumbent upon the duly authorized representatives of the Government torepresent its interests as well as private claims intrinsically dependent uponit. It is well-settled that the interests of the Government cannot berepresented by private persons.

  

31(Lea)

LEYVA vs. JANDOC4 SCRA 595

FACTS: Respondent Laiz filed before the Court of First Instance of Cotabato a petition for cancellation of adverse claim annotated on the back of his transfer certificate of title covering a parcel of land registered in the name of Manuela Jandoc.Petitioner Leyva claimed that he executed a Compromise Agreement in 1963 (notarized in 1972) with Manuela Jandoc where the former will withdraw his opposition in the Land Registration case in the condition that certain properties automatically belong to him in case of favorable judgment for Jandoc. Respondent Laiz reasoned that a private agreement of sale was executed in 1959 between him and Respondent Jandoc.The Court of First Instance of South Cotabato ruled against petitioner and confirmed the sale between Laiz and Jandoc pertaining to the parcel of land. The CFI also denied petitioner’s Motion for Reconsideration. The Intermediate Appellate Court affirmed in toto the lower court’s decision.Hence a petition for review on certiorari was filed by petitioner contending that the IAC committed grave error in ruling that Laiz’s deed of absolute sale prevailed over Levya’s compromise agreement.

ISSUE:Whether petitioner Levya, basing his claim on a Compromise Agreement, has a better right over the parcel of land than respondent Laiz, who executed a Deed of Sale with respondent Jandoc.

RULING:The instant petition is without merit.Upon consideration of the foregoing, the Court of appeals concluded that the Compromise Agreement whether executed in 1963 or 1972 between Leyva and Jandoc cannot prevail over the Agreement of Sale between Laiz and Jandoc and that Laiz has a better right over the property in question than Leyva.But Leyva lays much stree on the findings of Francisco Cruz, Jr., a handwriting expert of the PC Crime Laboratory, Camp Crame who testified that the purported signature of Jandoc appeared to be of another person and of Col. Crispin

B. Garcia, formerly Chief of the Chemistry Branch in the PC Crime Laboratory who testified that the Agreement could have been written within the years 1964 to 1967.Both the trial court and the Court of Appeals gave more weight to the testimony of Fiscal Sarinas as corroborated by Catolico, one of the signatories in the Deed of Transfer, and Versoza, a notary public, upholding the validity of the assailed Sale Agreement.This Court has ruled in the case of Vda. De Roxas v. Roxas, that “the positive testimony of the three attesting witnesses ought to prevail over the expert opinions which cannot be mathematically precise but which on the contrary, are subject to inherent infirmities.” In any event, it is well established that the appellate court will not disturb the factual findings of the lower court for the latter is in a better position to gauge credibility of witnesses.

PartV

1.De Castro v. Marcos

January27, 1969 G.R. No. L-26093

SANCHEZ,J.:

FACTS:

               Privaterespondent Rufino Akiacommenced a petition before the Court of First Instanceof Baguio City, actingas a cadastral court, to register in his name 15, 922square meters of landsituated in the City of Baguio. However, petitionerVirginia L. de Castro movedto intervene. Her interest is in the 1,000 squaremeters is allegedly includedin the 15,922 square meters of land specified inAkia's petition.

               Itappears that De Castro wasthe highest bidder in the public sale of a 1,000square meter-parcel of land inQuezon Hill, Baguio City and such land was thereafterawarded in her favor. DeCastro then filed with the Bureau of Lands TownshipSales Application coveringthe parcel of land. Petitioner, as claimed, had beenpaying taxes on the lot.

               However,the Respondent Judgedismissed the opposition of de Castro on the ground a mereapplicant has nopersonality to sue but de Castro moved to reconsider. Shestressed the fact thatshe was not a mere applicant of public land but anequitable owner thereofbecause she paid to the government in full the value ofthe land she appliedfor.

ISSUE:Does petitioner Virginia L. de Castro have legalstanding in the proceedingsbelow?

HELD:

               YES.Mere citizens could haveno interest in public land and to give a partystanding in a court of landregistration, he must make some claim to theproperty. Homesteader, purchaser offriar land, and all persons who claimto be in possession prior the issuance oftheir titles or awards to public landare can be considered as equitable owners.To claim ownership, a homesteaderwho had not yet been issued his title but whohad fulfilled all the conditionsrequired by law, should be regarded as an equitableowner of theland. Similarly, a purchaser of friar land must have anequitable title tothe land before the issuance of the patent.

               Recently,we declared thatpersons who claim to be in possession of a tract of publicland and have appliedwith the Bureau of Lands for its purchase havethe necessary personality tooppose registratior. Since Petitioner Virginiade Castro is an awardee inthe public sale held upon her own township salesapplication. The award itselfis enough to prevent the reopening by respondentAkia as to the land disputed.Of course,the award up to now has not been fully implemented because she hasnot yetcomplied with one condition imposed on her. But, if the award is notapermanent disposition, it is at least a provisional one, enough topreventreopening by respondent Akia as to the land disputed.

               TheSupreme Court, accordingly,ruled that petitioner has legal standing before thecadastral court.

2(Charleen)

3(Gina)

ANA P. FERNANDEZvs. FELIZA ABORATIGUE

G.R. No.L-25313         December 28, 1970

Makalintal,J.:

Facts:

Theplaintiff, claiming ownership of a parcel of land situated in sitio Dipulao,barrio Central, township of Coron, Palawan, filed suit 1 in the Palawan Courtof First Instance to have the defendants vacate a portion of said land andcease exercising acts of ownership therein.

It wasalleged in the complaint that Vicente Aboratigue, father of defendant FelizaAboratigue, was during his lifetime employed as guard at one of the gates tothe property, and as such was allowed to plant fruit trees in a small portionin the immediate vicinity; and that after his death the said defendant,together with her husband Restituto Bacnan, were allowed to stay therein, butthat later on they claimed to be the owners of the said portion. In theiranswer to the complaint the defendants alleged, as basis of their claim ofownership, continuous possession and cultivation of the disputed area, bythemselves and through their predecessors-in-interest since the year 1901.

Issue:

Whetheror not the improvements on the disputed land belong to the Aboratigue.

Held:

Yes. Therule is that the owner of buildings and improvements should claim them duringthe proceedings for registration and the fact of ownership, if upheld by thecourt, must be noted on the face of the certificate. There is no such notationhere in favor of the appellants. However, inasmuch as the improvements in thedisputed area have been acknowledgment by plaintiff in her complaint asbelonging to the appellants' predecessor-in-interest, and the lower court'sdecision allowing them to recover the value of the improvements is not now inquestion, this right of the

appellants must be upheld. The trial court has leftthe determination of such value to mutual agreement between the parties. Thisdisposition should be modified in the sense that if they fail to agree, thematter should be submitted to the said court for hearing and adjudication.

4(Zhon)

Republic vs. Antonio Bacas

G.R. No. 182913, November 20, 2013

Facts:

In 1938, Commonwealth President Manuel Luis Quezon (Pres. Quezon) issued Presidential Proclamation No. 265, which took effect on March 31, 1938, reserving for the use of the Philippine Army 3 parcels of the public domain situated in the barrios of Bulua and Carmen, then Municipality of Cagayan, Misamis Oriental. The parcels of land were withdrawn from sale or settlement and reserved for military purposes, "subject to private rights, if any there be."

The Bacases filed their Application for Registration on November 12, 1964 covering a parcel of land [Lot No. 4354] which is a portion of those reserved in PP No. 265. They alleged, inter alia, the names of adjoining owners, as well as the occupancy of the PH army by mere tolerance. The LRC granted the application. The Republic failed to appeal said decision.

On May 8, 1974, the Chabons filed their application for Registration of Lot No. 4357, also reserved in PP No. 265. They indicated therein the names and addresses of the adjoining owners, but no mention was made with respect to the occupation, if any, by the Philippine Army. LRC granted the application. The Republic failed to appeal said decision.

Republic filed with RTC a civil case for annulment, cancellation of original certificate of title, reconveyance of lot or damages against Bacases [Civil Case No. 3494] and another against Chabons [Civil Case No. 5918].

The RTC dismissed the petition, holding the following: (1) the stated fact of occupancy by Camp Evangelista over certain portions of the subject lands in the

applications for registration by the respondents was a substantial compliance with the requirements of the law and the respondents did not commit fraud in filing their applications for registration and that the Republic was then given all the opportunity to be heard as it filed its opposition to the applications, appeared and participated in the proceedings so the Republic is estopped from questioning the proceeding; (2) assuming arguendo that respondents were guilty of fraud, the Republic lost its right to a relief for its failure to file a petition for review on the ground of fraud within one (1) year after the date of entry of the decree of registration; and (3) the subject parcels of land were exempted from the operation and effect of the Presidential Proclamation No. 265 pursuant to a proviso therein that the same would not apply to lands with existing"private rights." The presidential proclamation did not, and should not, apply to the respondents because they did not apply to acquire the parcels of land in question from the government, but simply for confirmation and affirmation of their rights to the properties so that the titles over them could be issued in their favor.

Upon appeal, CA affirmed the RTC decision.

 Issue:

1. whether or not the decisions of the LRC over the subject lands can still bequestioned

2. whether or not the applications for registration of the subject parcels ofland should be allowed

Held:

1. The Republic can question even final and executory judgment whenthere was fraud. Pursuant to , the application for registration shall state thename in full and the address of the applicant, and also the names andaddresses of all adjoining owners and occupants, if known; and, if notknown, it shall state what search has been made to find them. The purposeof this is to serve as notice and publication of the hearing that would enableall persons concerned, who may have any rights or interests in the property,to come forward and show to the court why the application for registrationthereof is not to be granted.

The Chabons did not make any mention of the ownership or occupancyby the Philippine Army. They also did not indicate any efforts or searchesthey had exerted in determining other occupants of the land. Such omissionconstituted fraud and deprived the Republic of its day in court. Not beingnotified, the Republic was not

able to file its opposition to the applicationand, naturally, it was not able to file an appeal either.

2. Even if the Bacases successfully allege the names and addresses of allpersons concerned who may have any rights or interests in the property, theGovernment can still question the registration of Bacas’ land on the groundthat the same is still under the public domain pursuant to Pres. Proc. No.265. As such the subject land is inalienable and cannot be registered.

Further, prescription or estoppel cannot lie against the governmentbecause it is a well-settled rule in our jurisdiction that the Republic or itsgovernment is usually not estopped by mistake or error on the part of itsofficials or agents.

5(May Ann)

6(Rhea B.)

G.R. Nos. 85991-94 July 3, 1991

REPUBLIC CEMENT CORPORATION, petitioner, vs. COURT OF APPEALS, MOISES CORREA and REGISTER OF DEEDS OF BULACAN, respondents.

FACTS:

Republic Cement Corp. filed a petition in the CFI of Bulacan for the registration in its name a parcel of land which they bought form persons who had occupied and cultivated it continuously since the Spanish regime. It was opposed by Rayo, Mangahas and Legaspi stating that they are the owners of the eastern and east central portion which is sought to be registered by petitioner, having been in actual, open, public, adverse, peaceful and uninterrupted possession and occupation thereof in the concept of owner for a period of over 60 years and having acquired ownership thereof by donation on the occasion of their marriage from the parents of Jose Rayo. They were later substituted by Moises Correa as subsequent purchaser of said parcel of land. The court ordered the registration of the 3 parcels of land in the name of Correa. Republic Cement Corp. contends that respondents failed to prove the genuineness of their title.

ISSUE: whether or not respondent established the identity of and title to the land sought to be registered in his name?

HELD:

Contrary to the claim of petitioner, private respondent, through his predecessors in interest, was able to establish the identity of and title to the land sought to be registered in his name. The technical description and the survey plan duly approved by the Director of Lands submitted in evidence by

private respondent fully describes the metes and bounds of the parcels of land involved.

Survey plan of the property showing its boundaries and total are clearly identifies and delineates the extent of the land.

Based on said transfers, petitioner is now seeking the registration of the whole of Lot No. 2880 in its name. This we cannot allow. The deeds of sale relied upon by petitioner do not constitute sufficient legal justification for petitioner's claim over all of Lot No. 2880. Petitioner's title over said lot, as the successor in interest of said heirs, is limited only to whatever rights the latter may have had therein. It is elementary that a grantor can convey no greater estate than what he has or in which he has an alienable title or interest.

7(Angel)

8(Janine)

9(Mike Jayson)

10(Lou)

11(Jess)

12(Diane)

FELIPA S. LARAGAN, INDEPENDENCIO SIBBALUCA, AURORA C. SIBBALUCA, and ZENAIDA S. VALDEZ vs. HONORABLE COURT OF APPEALS, TEODORO LEAÑO, TOMAS LEAÑO, FRANCISCO LEAÑO, and CONSOLACION LEAÑO

G.R. No. L-47644 August 21, 1987

 

FACTS: Felipa Laragan, Independencio Sibbaluca, Aurora Sibbaluca, and Zenaida Valdez filed an application with the CFI of Isabela for the registration of their title over a parcel of land situated in the Barrio of Sto. Tomas, Ilagan, Isabela. The applicants alleged that they acquired said parcel of land by way of an absolute deed of sale from the spouses Anastacio and Lucrecia Sibbaluca and that they have been in possession thereof for more than 34 years. Solicitor General filed a written opposition alleging that the applicants and their predecessor-in-interest do not have sufficient title to the parcel of land sought to be registered. He prayed that the land be declared public land. On 2 August 1969, Teodoro Leano, Tomas Leano, Vicente Leano, Francisco Leano, and Consolacion Leano filed their opposition to the application for registration. They claimed that they are the owners of the southern part of the land and that they have been in possession for more than 30 years.

The trial court rendered judgment in favor of Laragan. CA affirmed the decision but declared the southern part as public land.

ISSUE: Whether petitioners can register the subject land.

HELD: No. The application for registration, filed with the lower court, was for the confirmation of an imperfect title. The law applicable is Section 48 (b) of the Public Land Act.

It is an established rule that an applicant for registration is not necessarily entitled to have the land registered in his name simply because no one appears to oppose his title and to oppose the registration of the land. He must show, even in the absence of opposition, to the satisfaction of the court, that he is the absolute owner, in fee simple.

It would appear, however, that the possession and occupation of the land by the spouses Anastacio and Lucresia Sibbaluca are tainted with bad faith so that the petitioners are not entitled to the benefits of the provisions of Section 48 (b) of the Public Land Law.

NOTES:

Sec. 48. The following described citizens of the Philippines, occupying lands of the public domain or claiming to own any such lands or an interest therein, but whose titles have not been perfected or completed, may apply to the Court of First Instance of the province where the land is located for confirmation of their claims and the issuance of a certificate of title therefor, under the Land Registration Act, to wit:

 (b) Those who by themselves or through their predecessors-in- interest have been in open, continuous, exclusive, and notorious possession and occupation of agricultural lands of the public domain, under a bona fide claim of acquisition of ownership, for at least thirty years immediately preceding the filing of the application for confirmation of title except when prevented by way or force majeure. These shall be conclusively presumed to have performed all the conditions essential to a Government grant and shall be entitled to a certificate of title under the provisions of this chapter.

13(April)

14(Daniel Eblahan)

Director of Lands v. CAGR no. L-50260, July 29, 1992Romero, J.:Facts:  In 1914, Estanislao Patricio was a known to possess a 256 hectare tract of land within Timberland Block A in Masbate. He used it as pasture land. This land was then sold to Vicente Montenegro, along with 100 cattle and their offspring, corrals and enclosures. Montenegro used the land the same way Patricio did. He sold the land to Maria Calanog in 1938 and used it the same way, rearing up to 500 animals by 1941. On January 20, 1956, Calanog then applied to register the land but she was opposed by the Director of Lands and the Director of Forestry. The Directors alleged that neither Calanog nor her predecessors-in-interest had sufficient title over the land and that it is public domain. The court issued a general default. Thirty-seven other people petitioned to lift the general default, claiming two-thirds of the land, among them are two people issued with certificates of title. They also opposed the application of title by Calanog. The trial court granted the application, excluding the two other lots with certificates of title. The Court of Appeals affirmed the decision. Issue:  Whether or not there the respondent has a valid claim to the landHeld:  No. The Court ruled that respondent can claim neither the 256 hectare land disputed by the public petitioners nor the remaining land claimed by the thirty-seven oppositors. In order to register a land of public domain under the Public Land Act before its amendment, it required adverse possession from July 26, 1894 and must have open, notorious, and uninterrupted possession of the land and it must be proven by the applicant.  In this case, the Court found no adequate proof for the

respondent’s claim. It found out that in the deed of sale in 1928, only 200 hectares of the land was sold, very different to the 598 hectare land sought to be registered. No convincing proof was given by the respondent on the matter. The respondent also did not have proof of the required possession under the Public Land Act, the earliest evidence of possession by her predecessor-in-interest was only in 1914.  On the other hand, the Court found that both the public respondents and the oppositors have evidence to support their claims on the land. The public respondents had satisfactory proof that the 256 hectares of the land are timberland as evidenced by documents such as reports and sketch plans. The oppositors also had proof that they had been occupying and possessing the 342 hectares of the land as early as 1954 as evidenced by homestead applications, free patents, and transfer certificates of title.

15(Melodia)

________________________________________________________________________________________________

16(Mel)

Bureau of Forestry, Bureau of Lands and Philippine Fisheries Commission, petitioners

vs.

Court of Appeals and Filomeno Gallo, respondents

G.R. # L-37995 August 1987

 

FACTS:

                On July 11, 196,1 four (4) parcels of lands situated in Buenavista, Iloilo described in Plan PSU-150727 containing an approximate area of 30.5943 hectares were the subject of an application for registration by Mercesdes Diago who alleged among others that she herself occupied said parcel of lands having bought them from the estate of late Jose Ma. Nava who in his lifetime bought the lands in turn from Canuto Gustillo on June 21, 1934.

                On June 30, 1965, respondent Filomeno Gallo having purchased the subject parcels of land from Mercedes Diago on April 27, 1965, move to be substituted in place of the latter attaching to his motion an amended application for registration of title. Also on August 30, 1965, petitioner Philippine Fisheries Commission also

moved to be substituted in place of Bureau of Forestry as oppositor over portion of the land sought to be registered.

                On April 6, 1966, the trial court rendered its decision ordering the registration of the four (4) parcels of land in the name of respondent  Filomeno Gallo. Petitioners appealed from said decision to the respondent court of  appeals which in turn affirmed the Regional court decision.

ISSUE :

Whether or not the court has the power to classify or reclassify public lands into alienable and disposable lands.

 

HELD :

No, The supreme court ruled that the classification or reclassification of public lands into alienable or disposable, mineral or forest land is now a prerogative of the executive department of the government and not of the courts. With these rules, there should be no more room for doubt that is not the court which determines the classification of lands of public domain into agricultural, forest or mineral but the executive branch of government through the office of the president.

__________________________________________________________________________________________________

17(Rocky)

18(Maribeth)

 

REPUBLIC OF THE PHILIPPINES vs CAet. al., 99 SCRA 742 

FACTS:

On May 16, 1966, PerpetuoAlpuerto, filed with the CFI of Quezon (Branch II), a motion to reopenCadastral Case No. 97, LCR Cad. Rec. No. 1555, and to admit his answer over LotNo. 7718 of the Mauban (Quezon) cadastre. After trial, the lower court renderedits decision adjudicating to said Alpuerto Lot No. 7718 together with itsimprovements, and ordered the issuance in his favor of the corresponding decreeof registration, thus an order for the issuance of a decree of registrationover the said lot. The Land RegistrationCommission then issued Decree No. 127177 as the basis of the issuance of OCTNo. 0-13541 in his favor. Portions of the lot were subsequently transferred tovarious persons who were issued their respective transfer certificates oftitle, among whom are private respondents.

On April 6, 1971, the Sol.Gen. filed for the government a complaint for annulment, cancellation of titlesand for reversion of Lot No. 7718 of Cadastral Survey of Mauban, Quezon to theState on the ground that the decision of the court adjudicating Lot No. 7718 toAlpuerto, its order for the issuance of the decree of registration as well asOCT No. 013541 and all its derivative titles are all null and void and withoutlegal effect because the court had no jurisdiction to allocate the subjectland, which is inalienable.

Herein Respondents IndustrialMarketing and Investments Corporation, Henry O. Antonio Angeles and AureaAngeles filed a motion to dismiss the complaint alleging that the action isbarred by a prior judgment and that the court lacks jurisdiction over thenature of the action or suit. Defendant, Andres Laredo, likewise filed a motionfor dismissal of the complaint, based on the grounds that the complaint statesno cause of action and that venue is improperly laid.

After hearing the motionsfor dismissal and the opposition thereto, the lower court issued an orderdenying the motion to dismiss filed by Respondent Laredo, but granting themotion to dismiss filed by Respondents Industrial Marketing and others anddismissing the complaint filed by petitioner. The motion for reconsideration was further denied. Consequently,petitioner appealed the case to the CA which affirmed the order of dismissal bythe lower court. The motion for reconsideration was likewise denied, hence,this petition.

ISSUE: Whether or not thecourt had jurisdiction to allocatethe subject land, which is inalienable?

HELD: No. It was held that where the land covered by the homestead application ofpetitioner was still within the forest zone or under the jurisdiction of the Bureauof Forestry, the Director of Lands had no jurisdiction to dispose of said landunder the provisions of the Public Land Law, and the petitioner acquired noright to the land. It follows that "if a person obtains a title under thePublic Land Act which includes, by oversight, lands which cannot be registeredunder the Torrens system, or when the Director of Lands did not havejurisdiction over the same because it is a public forest, the grantee does not,by virtue of the said certificate of title alone, become the owner

of the landillegally included". The title thus issued is void at law, since theofficer who issued it had no authority to do so.

19(Ailyn)

20(Jen)

IHVCP vs. UP 200 SCRA 554, International Hardwood and Veneer Company of the Philippines, vs. University of the Philippines, and Jose Campos, Jr. G.R. No. L-52518 August 13, 1991Facts: IHVCP is a company engaged in the manufacture, processing and exportation of plywood. It renewed its timber license, which was granted by the government and shall be valid for 25 years, in early 1960. Said license authorizes the company to cut, collect and remove timber from the portion of timber land located in certain Municipalities of Laguna, including Paete.In 1964, the Congress enacted R.A. 3990, an act establishing an experiment station for UP. The said experiment station covers a portion of timberland in Paete, occupied by IHVCP. So UP, who claims ownership of said portion of timberland, demanded the latter to pay the forest charges to it, instead of the BIR. IHVCP rejected the demand and it filed a suit against UP, claiming that R.A. 3990 does not empower UP to scale, measure and seal the timber cut by it within the tract of land referred to in said Act, and collect the corresponding forest charges prescribed by the BIR.Issue: Whether or not UP is the owner of the portion of timberland in Paete.Held: Yes. The court ruled that R.A. 3990 ceded and transferred in full ownership to UP the area, which means that the Republic of the Philippines completely recovered it from the public domain. In respect o the areas covered by the timber license of IHVCP, the said Act removed and segregated it from being a public forest.The court further cited Sec. 3 of R.A. 3990, which provides that, “any incidental receipts or income therefrom shall pertain to the general fund of the University of the Philippines.” The provision of the Act is clear that UP, being the owner of the said land, has the right to collect forest charges and to supervise the operations of IHVCP insofar as the property of the UP within it is concerned.

22(Nelson)

G.R. No. 83609 October 26, 1989

 petitioner,  vs. COURT OF APPEALS, IBARRA BISNAR and AMELIA BISNAR, respondents.

Facts:

            Ibarra Bisnar and Amelia Bisnar applied for the registration of their two titles to two parcels of land, respectively containing 24 hectares and 34 hectares of land in barrio Gen. Hizon, Mun. of President Roxas, province of Capiz. The CA granted their application for confirmation and registration of the titles. The parcels of land in question here are forest lands but their predecessors-in-interest possessed the land for more than 50 years and in an open, continuous, exclusive and notorious possession over the subject land.

Issue:

            Whether or not the CA erred in granting the application of the claimant.

Held:

            Yes. It bears emphasizing that a positive act of the government is needed to declassify land which is classified as forest and to convert it into alienable or disposable land for agricultural or other purposes. Unless and until the land classified as forest is released in an official proclamation to that effect so that it may form part of the disposable agricultural lands of the public domain, the rules on confirmation of imperfect title do not apply.

Thus, possession of forest lands, however long, cannot ripen into private ownership. A parcel of forest land is within the exclusive jurisdiction of the Bureau of Forestry and beyond the power and jurisdiction of the cadastral court to register under the Torrens System.

23(JM)

HEIRS OF JOSE AMUNATEGUI, PETITIONERS, VS. DIRECTOROF FORESTRY, RESPONDENT.

 G.R. No.L-27873,

November 29, 1983

FACTS:

        Roque Borre and Melquiades Borre filedan application for a registration of a certain land located in Pilar, Capiz. Indue time, the heirs of Amunategui filed an opposition to the application of theBorres and prayed that a portion of the lot be conveyed to them. Anotheroppositor, Emeterio Bereber, filed his claim over another portion of the lot.Subsequently, the director of Forestry also filed an opposition regarding allthe claims of the land claiming that the land was a forest land. During thetrial Roque borre sold his rights to Angel alpasan who later on filed his claimon the said land. The CFI of Capiz ruled on the issue and favored Bereber,Alpasan and Melquiades Borre of their claims. Thereafter, the heirs ofamunategui and the Director of Forestry appealed the case to the CA. the CAissued a ruling denying all claims on the land and seconded the stand of theDirector of Forestry that the land is a forest land therefore it can not beowned privately. The heirs of Amunategui appealed the case to the SupremeCourt. They claim that the land is not a forest land since it is already swampyand mangroves have grown on the area. Further, there are no trees classified insection 1820 of the Revised Administrative Code which would make a land aforest land. In addition they claim that the land is more productive as anagricultural land than as a forest land. They further contend that they havebeen in possession of said land for many years however such claim was belliedby the Director of Forestry through his evidence that even Jose Amunategui evenapplied for permit to cut timber on said land.

ISSUE:

        Can a land classified as forest land,be subject to private ownership by the fact that it was used as an agriculturalland and that its actual state is no longer a forested area?

HELD:

        NO, a land classified as a forest landcan not be owned by private individuals by virtue of its actual state. A forestland is defined by the law even if it is stripped of its forest cover. It cannot be said that by virtue of the cultivation of private persons using suchland as an agricultural land changes its classification as a forest land. Classificationof lands is left to the executive department of the government. Devoid of anygovernment act, the land remains as a forest land thereby not subject toprivate ownership. The contention of the petitioners with regards to theprovision of Section 48, Commonwealth Act No. 141, as amended by Republic ActNo. 1942 is untenable since such act only apply to agricultural lands and notto forest lands. In the case at bar the land in question had never ceased to bea forest land.

24(Maris)

25(Reg)

26

G.R. No. L-50464               January 29, 1990

SUNBEAM CONVENIENCE FOODS INC., CORAL BEACH DEVELOPMENT CORP., and the REGISTER OF DEEDS OF BATAAN, petitioners, vs. HON. COURT OF APPEALS and THE REPUBLIC OF THE PHILIPPINES, respondents.

 

FACTS:

On April 29, 1963, a Sales Patent issued by the Director of Lands in favor of Sunbeam for parcels of land situated in Mariveles, Bataan. It was registered to the Register of Deeds who in turn issued Original Certificate of Title to Sunbeam and Transfer Certificate of Title in favor of Coral Beach. The SolGen instituted an action

for the reversal of the Civil Case No. 4062 favoring Sunbeam and Coral Beach claims with the CFI of Bataan.

Sunbeam and Coral Beach moved for dismissal and was approved. CA reversed the CFI decision and decides to give due course to the petition of the SolGen.

ISSUE:

Whether the classification of the subject lots are forest lands.

Whether the title conferred by the Director of Lands is valid for forest lands.

HELD:

Our adherence to the Regalian doctrine subjects all agricultural, timber, and mineral lands to the dominion of the State. Thus, before any land may be declassified from the forest group and converted into alienable or disposable land for agricultural or other purposes, there must be a positive act from the government. Even rules on the confirmation of imperfect titles do not apply unless and until the land classified as forest land is released in an official proclamation to that effect so that it may form part of the disposable agricultural lands of the public domain.

The mere fact that a title was issued by the Director of Lands does not confer any validity on such title if the property covered by the title or patent is part of the public forest.

Petition denied.

27(Ed)

28(LJ)

29(Zax)

30(Clathem)

REPUBLIC OF THE PHILIPPINES, DIRECTOR OF LANDS andDIRECTOR OF FORESTRY, petitionersvs.HON. COURT OF APPEALS, PAULINA PARAN, ELISAPARAN MAITIM and SINA PARAN, respondents. G.R. No. 48327 August 21, 1991

Facts:Private respondents are applicants for registration ofa parcel of land situated in Beckel La Trinidad,Benguet, covered by Survey Plan Psu-105218. In their application dated 13 February1970, private respondents claim to have acquired the land from their fatherDayotao Paran and by actual, physical, exclusive and open possession thereofsince time immemorial. Private respondents do notdispute that the land was formerly a part of the Central Cordillera ForestReserve, they contend that it had already been released therefrom. Privaterespondents submitted a certification 17 signed by Leopoldo Palacay,Chief of Land Classification Party No. 57 of the Bureau of Forest Development,Department of Agriculture and Natural Resources, showing the legal nature orstatus of the land as alienable or disposable.

On 18 November 1970, the Office of theSolicitor General filed on behalf of the Director of Lands anOpposition contending that: (1) private respondents have no registrabletitle; (2) the parcel of land sought to be registered is part of the publicdomain belonging to the Republic of the Philippines; and (3) the applicationfor registration was filed after expiration of the period provided for in R.A.No. 2061, hence the land registration court did not acquire jurisdiction overthe case.

The Office of the Provincial Fiscal ofBaguio and Benguet, on the other hand, filed a Motion to Dismiss. On 12November 1970, the land registration court issued anOrder 5 declaring a general default against the whole world exceptthe Bureau of Lands, the Reforestration Administration, and the Bureau ofForestry. Another Order 6 was then issued denying the motion todismiss filed by the Provincial Fiscal.

Issue: Whether or not the certificationsigned by the chief of Bureau of Forest Development is an adequate proof toreclassify the land.

Held: No. while the Court considered the reports of the District Forester and theDistrict Land Officer as adequate proof that the land applied for was no longerclassified as forestal. The difficulty in the instant case is that while thecertification of Leopoldo Palacay on which private respondents rely may,standing alone, be evidence that a reclassification had occurred, it iscontradicted by an official report of Luis Baker. Moreover, the privaterespondents' application for registration was in fact opposed by the Directorof Lands as well as the Director of Forestry.

Furthermore, the evidence of record thusappears unsatisfactory and insufficient to show clearly and positively that theland here involved had been officially released from the Central CordilleraForest Reserve to form part of the alienable and disposable lands of the publicdomain. Once a parcel of land is shown to have been included within a ForestReservation duly established by Executive Proclamation, as in the instant case,a presumption arises that the parcel of land continues to be part of suchReservation until clear and convincing evidence of subsequent withdrawaltherefrom or de-classification is shown. A simple, unsworn statement of a minorfunctionary of the Bureau of Forest Development is not, by itself, suchevidence.

31(Lea)

G.R. No. L-46048 November 29, 1988

 REPUBLIC vs. CA

 

 FACTS:

Spouses Jose Zurbito and Soledad Zurbito purchasedsmall parcels of land from various owners adjoining the 30 hectares of land located in Calulod Pauwa, Masbate inherited by Jose Zurbito from his parents. The land thus acquired by the spouses comprises the two parcels of land). The spouses occupied and cultivated these properties.Upon the death of Jose Zurbito, his wife Soledad Buencamino Zurbito and his children succeeded to his estate and continued his possession. The estate of Jose Zurbito was placed under administratix thereof.

Soledad vda. de Zurbito eventually sold her rights, title, interest and participation in the parcels of land subject of this registration proceedings in favor of the herein applicant and her son-in-law, Dr. Miguel Marcelo under a deed of absolute sale for and in consideration of the sum of P10,000.00. Then Soledad vda. de Zurbito, as administratix of the estate of the deceased Jose Zurbito and in behalf of her children co-signed with her other children a deed of sale with a right to repurchase wherein they sold to Miguel Marcelo for and in consideration of the sum of P12,000.00 the remaining one-half undivided portion of the properties mentioned in the previous deed of sale.

Under the provisions of the aforementioned deed, the vendors were given the option to repurchase the properties within five years from the date thereof (October 24, 1944) extendible for another period of one year at the option of vendors and that upon failure of the vendors to redeem the properties, within the alloted time, title shall automatically vest in the vendee, Dr. Miguel Marcelo. When the vendors-a-retro failed to repurchase the properties within the period agreed upon, Dr. Marcelo consolidated ownership of the properties by executing and registering in 1954 an affidavit of consolidation dated December 2,1953 in the office of the Register of Deeds of Masbate. However, he allowed his mother-in-law, Soledad vda. de Zurbito, to continue managing the properties. This arrangement was made in consideration of the personal relationship between the applicant and Soledad vda. de Zurbito.

The Opposition of the Director of Forestry is centered on the claim that approximately 22 hectares of the land is within the forest zone. The authentic document evidencing the classification of the land applied for registration as a forest zone has been presented. CA confirmed and ordered the registration of the title over the property in the name of Miguel Marcelo, married to Celia Zurbito, Filipino, of legal age and residing at 687 General Geronimo, Sampaloc, Manila.The Director of Forestry (now Director of Forest Development) and the Heirs of Jose Zurbito, as oppositors, appealed the aforequoted decision to respondent Court of Appeals.CA rendered judgment declaring the applicant and the private oppositors, now the private respondents herein, as co-owners, in stated shares, of the entire property involved while denying a motion for reconsideration filed by oppositor-appellant Director of Forestry.

ISSUE:Whether the 22 hectares area which forms part of the land applied for registration by and decreed in favor of herein private respondents is disposable agricultural land.

RULING:

 Yes.

“Forest reserves of public land can be established as provided by law. When the claim of the citizen and the claim of the Government as to a particular piece of property collide, if the Government desires to demonstrate that the land is in reality a forest, the Director of Forestry should submit to the court convincing proof that the land is not more valuable for agricultural than for forest purposes. Great consideration, it may be stated, should, and undoubtedly will be, paid by the courts to the opinion of the technical expert who speaks with authority on forestry matters. But a mere formal opposition on the part of the Attorney-General for the Director of Forestry, unsupported by satisfactory evidence, will not stop the courts from giving title to the claimant. “

It is not disputed that the aforesaid Land Classification Project No. 3, classifying the 22-hectare area as timberland, was certified by the Director of Lands only on December 22, 1924, whereas the possession thereof by private respondents and their predecessor-in-interest commenced as early as 1909. While the Government has the right to classify portions of Public land, the primary right of a private individual who Possessed and cultivated the land in good faith much prior to such classification must be recognized and should not be prejudiced by after-events which could not have been anticipated.

Thus, We have held that the Government, in the first instance may, by reservation, decide for itself what portions of public land shall be considered forestry land, unless private interests have intervened before such reservation is made. 

Part VI

1.  REPUBLIC v. CA (135 SCRA 156)

135 SCRA 156 January 31, 1985

FACTS:

                Spouses Maxino andTarciana Morales registered their parcel of land located at Barrio Cambuga(Anonang), Mulanay with the Court of First Instance at Gumaca, Quezon. On March21, 1961, Judge Vicente del Rosario rendered a decision, ordering theregistration of said land. The decision became final and executory. A decreeand an original certificate of title were issued.

                More than eightyears later, the Republic of the Philippines filed an amended petition tonullify the decision, decree, and title on the ground that the decision wasvoid because the land in question was still a part of the unclassified publicforest. It is axiomatic that public forestal land is not registerable. Itsinclusion in the public forest was certified by Director of Forestry FlorencioTamesis . The certification was reiterated by the Director of Forestry on May20, 1948.

                 On the other hand, the spouses contended thatthe former declaration that the land is alienable estopped the Government toannul the decision on the ground that the said land was discovered to be apublic forest and thereafter cannot be registered.

ISSUE: Whether or not the mistake of the lower court in granting theland title in favor of the spouses estopped the Government from declaring theland not alienable and not susceptible of private appropriation.

HELD:

                NO, the Governmentshould not be estopped by the mistakes or error of its agents.It isincontestable that the area registered by the Maxinos, is within the publicforest and therefore not alienable and disposable nor susceptible of privateappropriation. Possession of public forestal lands, however long, cannot ripeninto private ownership

2(Charleen)

3(Gina)

Directorof Lands vs. Court of Appeals

G.R. No.                May 31,1984

Facts:

The landin question, Identified as Lot 2347, Cad-302 is situated in Obando, Bulacan,and has an area of approximately 9.3 hectares. It adjoins the Kailogan Riverand private respondents have converted it into a fishpond.

In theirapplication for registration filed on May 10, 1976, private respondents(Applicants, for brevity) claimed that they are the co-owners in fee simple ofthe land applied for partly through inheritance in 1918 and partly by purchaseon May 2, 1958; that it is not within any forest zone or military reservation;and that the same is assessed for taxation purposes in their names.

TheRepublic of the Philippines, represented by the Director of the Bureau ofForest Development opposed the application on the principal ground that theland applied for is within the unclassified region of Obando, Bulacan, per BFMap LC No. 637 dated March 1, 1927; and that areas within the unclassified regionare denominated as forest lands and do not form part of the disposable andalienable portion of the public domain.

Afterhearing, the Trial Court ordered registration of the subject land in favor ofthe Applicants. This was affirmed on appeal by respondent Appellate Court,which found that "through indubitable evidence (Applicants) and theirpredecessors-in-interest have been in open, public, continuous, peaceful andadverse possession of the subject parcel of land under a bona fide claim ofownership for more than 30 years

prior to the filing of the application"and are, therefore, entitled to registration. It further opined that"since the subject property is entirely devoted to fishpond purposes, itcannot be categorized as part of forest lands. "

Issue:

Whetheror not applicants are entitled to judicial confirmation of title.

Held:

No. Ineffect, what the Courts a quo have done is to release the subject property fromthe unclassified category, which is beyond their competence and jurisdiction.The classification of public lands is an exclusive prerogative of the ExecutiveDepartment of the Government and not of the Courts. In the absence of suchclassification, the land remains as unclassified land until it is releasedtherefrom and rendered open to disposition. This should be so undertime-honored Constitutional precepts. This is also in consonance with theRegalian doctrine that all lands of the public domain belong to the State, andthat the State is the source of any asserted right to ownership in land andcharged with the conservation of such patrimony.

Sincethe subject property is still unclassified, whatever possession Applicants mayhave had, and, however long, cannot ripen into private ownership.

4(Zhon)

Republic vs. Francisco Bacus

G.R. No. 73261, August 11, 1989

Facts:

Respondent Bacus claims to be the owner of a parcel of land consisting of 496 square meters and situated in Manga, Tinago, Ozamis City. Although classified as forest land, the subject land has become highly developed residential-commercial land.On September 14, 1981, Bacus filed an application for its registration in his name with the Court of First Instance of Misamis Occidental. The application was opposed by the Republic of the Philippines through the Director of Lands on the grounds that the applicant did not have title to the property nor was it available for private appropriation since it was still part of the public domain.

The registration court ruled in favor of the applicant [Bacus]. The Republic appealed but CA affirmed in toto the appealed decision, prompting the instant petition.

Issue: w/n the subject land has changed its status as forest land due to its development into residential and commercial land

Held:

The subject land is still a forest land despite it being used as residential and commercial lands. It is for the claimant to show that it has been released for private appropriation, but this he has failed to do. No evidence has been offered to prove that the lot had earlier been declassified as forest land by the proper authority, to wit, the President of the Philippines.

Nota bene:

1. Forest lands are not alienable as such and can be the subject of private appropriation only when they are declassified and declared as alienable. As long as they remain forest lands, no court has jurisdiction to order their registration in the name of a private person.

2. Sec. 6 of Act No. 141 provides,The President, upon the recommendation of the Secretary of Agriculture and Natural Resources, shall from time to time classify the lands of the public domain into:(a) Alienable or disposable, (b) Timber, and (c) Mineral lands,and may at any time and in a like manner transfer such lands from one class to another, for the purposes of their administration and disposition.

3. Sec. 1827 of Revised Administrative Code provides,Lands in public forests, not including forest reserves, upon the certification of the Director of Forestry that said

lands are better adapted and more valuable for agricultural than for forest purposes and not required by the public interests to be kept under forest, shall be declared by the Department Head to be agricultural lands.

5(May Ann)

6(Rhea B.)

REPUBLIC vs SAYO

Facts:

The respondent spouses filed an original application for registration of a tract of land having an area of 33,950 hectares. Oppositions were filed by the Government, through the Director of Lands and the Director of Forestry, and some others. The case dragged on for about twenty (20) years. The remaining area of 5,500 hectares was, under the compromise agreement, adjudicated to and acknowledged as owned by the Heirs of Casiano Sandoval, but out of this area, 1,500 hectares were assigned by the Casiano Heirs to their counsel, Jose C. Reyes, in payment of his attorney's fees. In a decision rendered on 1981, the respondent Judge approved the compromise agreement and confirmed the title and ownership of the parties in accordance with its terms.

The Solicitor General contends that no evidence whatever was adduced by the parties in support of their petitions for registration; neither the Director of Lands nor the Director of Forest Development had legal authority to enter into the compromise agreement; as counsel of the Republic, he should have been but was not given notice of the compromise agreement or otherwise accorded an

opportunity to take part therein; that he was not even served with notice of the decision approving the compromise; it was the Sangguniang Panlalawigan of Quirino Province that drew his attention to the "patently erroneous decision" and requested him to take immediate remedial measures to bring about its annulment.

The respondents contended that the Solicitor General's arguments are premised on the proposition that the disputed land is public land, but it is not.

Issue:

Whether there was no evidence adduced by the parties in support of their petitions for registration

Held:

Yes. There was no competent evidence adduced by the parties in support of their petitions for registration.

The assent of the Directors of Lands and Forest Development to the compromise agreement did not and could not supply the absence of evidence of title required of the private respondents

It thus appears that the decision of the Registration Court a quo is based solely on the compromise agreement of the parties. But that compromise agreement included private persons who had not adduced any competent evidence of their ownership over the land subject of the registration proceeding. Portions of the land in controversy were assigned to persons or entities who had presented nothing whatever to prove their ownership of any part of the land. What was done was to consider the compromise agreement as proof of title of the parties taking part therein, a totally unacceptable proposition. The result has been the adjudication of lands of no little extension to persons who had not submitted any substantiation at all of their pretensions to ownership, founded on nothing but the agreement among themselves that they had rights and interests over the land.

In the proceeding at bar, it appears that the principal document relied upon and presented by the applicants for registration, to prove the private character of the large tract of land subject of their application, was a photocopy of a certification of the National Library. But, as this Court has already had occasion to rule, that Spanish document cannot be considered a title to property, it not being one of the grants made during the Spanish regime, and obviously not constituting primary evidence of ownership. It is an inefficacious document on which to base any finding of the private character of the land in question.

It thus appears that the compromise agreement and the judgment approving it must be, as they are hereby, declared null and void, and set aside. Considerations of fairness however indicate the remand of the case to the Registration Court so that the private parties may be afforded an opportunity to establish by competent evidence their respective claims to the property.

The decision of the respondent Judge complained of is annulled and set aside.

7(Angel)

8(Janine)

9(Mike Jayson)

10(Lou)

11(Jess)

12(Diane)

BERNARDO CARABOT, JUAN BANQUILES, LEONIDA V. ENDIAPE, LEON VILLANUEVA, OLIMPIA L. BANQUILES, FELISA BANQUILES, SEBASTIAN VILLANUEVA AND JUAN BANDAYRIL vs. THE Hon. COURT OF APPEALS, (FIRST DIVISION), SAMUEL PIMENTEL in his own behalf and in behalf of the minors, ALEXANDER MANUEL, JR., ALMABELLA and CLARIBEL all surnamed, PIMENTEL, NATIVIDAD RIOFLORIDO, assisted by her husband GREGORIO DINGLASAN

G.R. No. L-50622-23 November 10, 1986

 

FACTS: Samuel Pimentel and his children, as plaintiffs, alleged that they are the pro-indiviso owners of a parcel of land in Bo. Tala, San Narciso, Quezon. The Pimentels claimed that during the lifetime of Estrella Ribargoso, she placed as tenants on portions of the land the herein petitioners and that, after the death of Estrella, the Carabots and the Villanuevas asserted interest adverse to the Pimentels. The Pimentels prayed that the Carabots and the Villanuevas "be compelled to disclose the facts on which they base their claims" and be declared to have no title to or interest of any kind in the property. 

Natividad Rioflorido assisted by her husband Gregorio Dinglasan, claimed ownership of a "portion of the land in Bo. Tala, San Narciso, Quezon. She alleged that the herein petitioners, "who have been hired as tenants on the land have been illegally occupying for more than one year now several portions of the said property, pretending or assuming to be public land applicants." Ribargoso prayed that she be declared the lawful owner of the property and that her possession over the occupied portions be restored.

In both cases, the herein petitioners, as defendants, asserted that they have never been tenants of the private respondents, that they occupied the land as their

homesteads and that they have already applied for patents under the Public Land Law.

The trial court ruled that the homestead and free patent titles of the petitioners are null and void. CA affirmed.

ISSUE: Whether or not the parcels of land occupied by the petitioners and titled in their names as a result of homestead and/or free patents were already private property.

HELD: No. The parcels of land are not private property. It appears, then, that the titles of Dona Agripina Paguia were secured pursuant to the provisions of the Royal Decree of December 26, 1884. There was no proof regarding the validity of the Titulo de Composition con El Estado, granting that the existence of such documents has been proven by the certification of the Registrador de Titulos. SC has already ruled that "In order that natural boundaries of land may be accepted for the purpose of varying the extent of the land included in a deed of conveyance the evidence as to such natural boundaries must be clear and convincing. Petitioners failed to do so. In this regard the Supreme Court has ruled that:

All lands that were not acquired from the Government, either by purchase or by grant, belong to the public domain. An exception to the rule would be any land that should have been in the possession of an occupant and of his predecessors-in-interest since time immemorial; for such possession would justify the presumption that the land had never been part of the public domain or that it had been a private property even before the Spanish conquest.

Furthermore, as previously stated, there is no sufficient proof to show that what was described in the Spanish titles was precisely the same land in dispute. 

13(April)

14(Daniel Eblahan)

Tan v. RepublicGR no. 177797, December 4, 2008Chico-Nazario J.:Facts:  Spouses Tan acquired an unregistered parcel of land, a land forming part of the public

domain that became alienable on December 31, 1925, in Misamis Oriental from the Neris through a sale dated on June 26, 1970. The spouses immediately possessed the property and planted trees and fruit bearing plants. The spouses sought to register the land and was allowed by the RTC. The Republic assailed the decision in its appeal to the Court of Appeals and contended that the Tans did not have an open, continuous and notorious possession of the land because their predecessors-in-interest did not possess it before June 12 1945, making the property public domain. The Court of Appeals ordered the Tans to return the property to the Republic.Issue:  Whether or not the Tans have an open, continuous, exclusive, and notorious possession of the subject property.Held:  No, the Court denied the petition because the spouses failed to satisfy the requirements of the Public Land Act. The Court ruled that in the Public Land Act, persons occupying or claiming to own public domain should prove that they or their predecessors-in-interest have been in open, continuous, exclusive, and notorious possession and occupation of agricultural lands of the public domain since June 12, 1945. To prove this, the possession and occupation of the land must have started from June 12, 1945 or earlier.  In this case, the spouses failed to prove said requirement. Even if their predecessors-in-interest have possessed the land for thirty years or more, the earliest evidence of possession was a tax declaration from 1952, and there is no other evidence showing acts of ownership exercised by their predecessors-in-interest. The mere showing of possession for thirty years or more is not sufficient. Moreover, tax declarations are not conclusive evidence of ownership in the absence of actual, public, and adverse possession.Significance:Proof of ownership: tax declarations are not conclusive evidence of ownership.

15(Melodia)

16(Mel)

HEIRS OF MARIO MALABANAN vs. REPUBLIC OF THE PHILIPPINES 

GR No. 179987April 29, 2009

Facts:

On February 20 1998, Mario Malabanan filed an application for land registration before the RTC of Cavite, covering a parcel of land situated in Silang Cavite, consisting of 71,324 sq. meters. Malabanan claimed that he had purchased the

property from Eduardo Velazco, and that he and his predecessors-in-interest had been in open, notorious, and continuous adverse and peaceful possession of the land for more than 30 years. Velazco testified that the property was originally belonged to a twenty-two hectare property owned by his great-grandfather, Lino Velazco. Lino had four sons– Benedicto, Gregorio, Eduardo and Esteban–the fourth being Aristedes’s grandfather. Upon Lino’s death, his four sons inherited the property and divided it among themselves. But by 1966, Esteban’s wife, Magdalena, had become the administrator of all the properties inherited by the Velazco sons from their father, Lino. After the death of Esteban and Magdalena, their son Virgilio succeeded them in administering the properties, including Lot 9864-A, which originally belonged to his uncle, Eduardo Velazco. It was this property that was sold by Eduardo Velazco to Malabanan. Among the evidence presented by Malabanan during trial was a Certification dated 11 June 2001, issued by the Community Environment & Natural Resources Office, Department of Environment and Natural Resources (CENRO-DENR), which stated that the subject property was “verified to be within the Alienable or Disposable land per Land Classification Map No.3013 established under project no. 20-A and approved as such under FAO 4-1656 on March 15, 1982.”  On December 3, 2002, the RTC approved the application for registration.

The Republic interposed an appeal to the Court of Appeals, arguing that Malabanan had failed to prove that the property belonged to the alienable and disposable land of the public domain, and that the RTC had erred in finding that he had been in possession of the property in the manner and for the length of time required by law for confirmation of imperfect title. On February 23, 2007, the Court of Appeals reversed the RTC ruling and dismissed the application of Malabanan.

Issues:

1. In order that an alienable and disposable land of the public domain may be registered under Section 14(1) of Presidential Decree No. 1529, otherwise known as the Property Registration Decree, should the land be classified as alienable and disposable as of June 12, 1945 or is it sufficient that such classification occur at any time prior to the filing of the applicant for registration provided that it is established that the applicant has been in open, continuous,exclusive and notorious possession of the land under a bona fide claim of ownership since June12, 1945 or earlier?

2. For purposes of Section 14(2) of the Property Registration Decree, may a parcel of land classified as alienable and disposable be deemed private land and therefore susceptible to acquisition by prescription in accordance with the Civil Code?

3. May a parcel of land established as agricultural in character either because of its use or  because its slope is below that of forest lands be registrable under Section 14(2) of the Property Registration Decree in relation to the provisions of the Civil Code on acquisitive prescription?

4. Are petitioners entitled to the registration of the subject land in their names under Section14 (1) or Section 14(2) of the Property Registration Decree or both? 

Ruling:

The Petition is denied.

In connection with Section 14(1) of the Property Registration Decree, Section 48 (b) of the Public Land Act recognizes and confirms that “those who by themselves or through their  predecessors in interest have been in open, continuous, exclusive, and notorious possession and occupation of alienable and disposable lands of the public domain, under a bona fide claim of acquisition of ownership, since June 12, 1945” have acquired ownership of, and registrable title to, such lands based on the length and quality of their possession. (a) Since Section 48(b) merely requires possession since 12 June 1945 and does not require that the lands should have been alienable and disposable during the entire period of possession, the possessor is entitled to secure judicial confirmation of his title thereto as soon as it is declared alienable and disposable, subject to the timeframe imposed by Section 47 of the Public Land Act. (b) The right to register granted under Section 48 (b) of the Public Land Act is further confirmed by Section 14 (1) of the Property Registration Decree.

In complying with Section 14 (2) of the Property Registration Decree, consider that under the Civil Code, prescription is recognized as a mode of acquiring ownership of patrimonial property. However, public domain lands become only patrimonial property not only with a declaration that these are alienable or disposable. There

must also be an express government manifestation that the property is already patrimonial or no longer retained for public service or the development of national wealth, under Article 422 of the Civil Code. And only when the property has become patrimonial can the prescriptive period for the acquisition of property of the public dominion begin to run. Patrimonial property is private property of the government. The person acquires ownership of patrimonial property by prescription under the Civil Code is entitled to secure registration thereof under Section 14 (2) of the Property Registration Decree.

17(Rocky)

18(Maribeth)

MUNICIPALITY OF SANTIAGO vs. CA, 120 SCRA 734 

FACTS:

Lots Nos. 4976-A and 8000-A locatedin the Municipality of Santiago are being claimed by respondent Maria Cauiñan,the surviving spouse of Eulalio Bayaua, son of Liberato Bayaua, the allegedoriginal occupant thereof.  They claimthat they possessed said property in the concept of owners since timeimmemorial through their predecessors-in-interest. They base their claim ofownership on (1) a Spanish document entitled, Estadisticade los Terrenos de propiedad particular existantes en este pueblo de Carig,Provincia de la Isabela de Luzon Num. 6 del ano 1896" on file with theDivision of Archives, Bureau of Public Libraries, wherein the name of "DonLiberato Bayaua" was listed as "proprietario" over a 3hectare, 61 centare lot; (2) on Tax Declarations for the years 1921, 1938-1939,1945, 1957-1959 in the name of Eulalio Bayaua; and (3) on tax receipts. 5

During the cadastral surveyof the Municipality of Santiago from October 30, 1927 to March 23, 1932, andthe cadastral proceedings that followed in 1939, private respondents and theirpredecessors-in-interest did not file any claim to Lot 8000-A, which includedthe bigger portion of the market site.

Eulalio Bayaua filed a claimwith respect to Lot 4976 during the cadastral survey, but did not file anAnswer during the cadastral proceedings. The only ones who filed

theircadastral Answers with regards to Lot 4976 were the Municipality of Santiagoand one Antero Catabas.

After the suspension of thehearing due to war, respondent heirs of Eulalio Bayaua filed a Petition toAdmit their Answers to Lot No. 4976 and Lot No. 1-D, alleging that previousanswers had been filed by Eulalio within the statutory period, but due toaccident, mistake or excusable negligence, the same could not be found in theCourt records. Petitioner opposed admission. The Court denied admission of theAnswer. Private respondents did not move for the reconsideration of said Ordernor did they take other remedial steps to have the Answer admitted. The lower courtthen issued another Order declaring Lot No. 4976 public land subject, however, to whatever rightsthe Municipality of Santiago, Province of Isabela, may have by virtue andpursuant to Presidential Proclamation No. 131.

Private respondents did notmove for the reconsideration of the foregoing Order, nor did they appealtherefrom.

Apparently, privaterespondents had reiterated their petition to admit answer, for, on July 19,1968, another Order was issued in the cadastral case denying admission on theground that Lot 4976 had been declared public land in the Order of September17, 1963, which Order had become final (Exhibit "6").

Private respondents appealedto the Court of Appeals which reversed the judgment of the Trial Court anddeclared private respondents the lawful owners and possessors of the disputedland through acquisitive prescription. Hence, the present appeal by certiorariby the Municipality.

ISSUE: Whether or not theproof of possession adduced by private respondents constitutes exclusive andnotorious possession under claim of ownership?

HELD: No. Private respondents failed to submit convincingevidence of actual, peaceful and adverse possession in the concept of owner ofthe entire area in question until the time they were allegedly dispossessed bythe Municipality sometime in 1951. As pointed out by the Trial Court, privaterespondents merely showed a "sporadic feeble cultivation of portionsthereof which does not amount to possession as owner". A mere casual cultivation of portionsof the land by the claimant does not constitute possession under claim ofownership. In that sense, possession is not exclusive and notorious so as togive rise to a presumptive grant from the State.

19(Ailyn)

20(Jen)

Municipality of Antipolo vs. Zapanta G.R. No. L-65334 December 26, 1984

FACTS: On August 8, 1977, a single application for the registration of two distinct parcels of land was filed by two distinct applicants before the then Court of First Instance of Rizal, Branch XV, Makati (the Registration Court, for short). One of the two applicants was Conrado Eniceo. He had applied for registration under the Torrens system of a parcel of land containing 258 square meters. The other applicant was "Heirs of Joaquin Avendaño", and the land they were applying for registration was a parcel (hereinafter called the DISPUTED PROPERTY) containing 9,826 square meters surveyed in the name of the Municipality of Antipolo (ANTIPOLO, for short). Both parcels were situated in the Municipality of Antipolo. The applications were approved by the Registration Court on February 26, 1980. ANTIPOLO took steps to interpose an appeal but because it failed to amend the Record on Appeal, its appeal was disallowed.On May 22, 1981, ANTIPOLO filed a complaint in Civil Case No. 41353, also of the Court of First Instance of Rizal, Branch XIII, Pasig (the CASE BELOW, for short) against named "Heirs of Joaquin Avendaño", and their assignees (hereinafter called the AVENDAÑO HEIRS) praying for nullification of the judgment rendered by the Registration Court. The defendants, in their Answer, pleaded a special defense of res judicata, After a preliminary hearing on the mentioned special defense, the CASE BELOW was dismissed. ANTIPOLO perfected an appeal to the then Court of Appeals.A notice to file Brief was issued by the Appellate Court, which ANTIPOLO claimed it had not received. Upon motion of the defendants-appellees to dismiss on the ground that ANTIPOLO had not filed its Brief within the reglementary period, the appeal was dismissed despite the fact that before the dismissal, ANTIPOLO had submitted its Appellant's Brief.Issue: Whether or not the registration court did not have jurisdiction over the land subject of registration, it being property of the Municipality of Antipolo, used long before the war as a public market and other public purposes, and hence actually devoted to public use and service.Held: The claim of the AVENDAÑO HEIRS that they merely tolerated occupancy by ANTIPOLO which had borrowed the DISPUTED PROPERTY from them, since they had been in possession, since as far back as 1916, erroneously presupposes ownership thereof since that time. They forget that all lands are presumed to be public lands until the contrary is established. The fact that the DISPUTED PROPERTY may have been declared for taxation purposes in their names or of their predecessors-in-interest as early as 1918 does not necessarily prove ownership. They are merely indicia of a claim of ownership. ANTIPOLO had also declared the DISPUTED PROPERTY as its own in Tax Declarations Nos. 909, 993 and 454.Since the Land Registration Court had no jurisdiction to entertain the application for registration of public property of ANTIPOLO, its Decision adjudicating the DISPUTED PROPERTY as of private ownership is null and void. It

never attained finality, and can be attacked at any time. It was not a bar to the action brought by ANTIPOLO for its annulment by reason of res judicata. It follows that the titles issued in favor of the AVENDAÑO HEIRS must also be held to be null and void. They were issued by a Court with no jurisdiction over the subject matter. Perforce, they must be ordered cancelled.

22(Nelson) HINDI KO PO SURE KUNG TAMA UNG CASE

G.R. No. L-61462 July 31, 1984

REPUBLIC OF THE PHILIPPINES (Director of Lands), petitioner,  vs. COURT OF APPEALS and TEDITA INFANTE TAYAG, respondents.

Facts:

This is an application for registration of land with an area of more than 11 hectares located at Barrio Tambo, Buhi, Camarines Sur which was opposed by the Director of Lands.

Tedita, who claimed the land on 1976, contended that the subject land was possessed by her father Froilan Infante, who died in 1937. Tedita knew the land but not aware of its history. The only witness of Tedita was  Abraham Morandarte, who testified that he was the overseer of Tedita’s father and he supported the claim of Tedita. Furthermore, The realty taxes of the subject land from 1946 up to 1976 were only paid in 1976 by Tedita’s brother-in-law.

Issue:

            Whether or not the evidence presented by Tedita is enough to grant the application for registration.

Held:

            No. Under that law, the applicant must prove that he, by himself and through his predecessors in interest, have been in the open, continuous, exclusive and notorious possession and occupation of public agricultural land, under abona fide claim of acquisition of ownership, for at least thirty years immediately preceding the filing of the application for confirmation of title except when prevented by force majeure.

After a study of the records, we find that contention to be meritorious. The testimonies of Mrs. Tayag (who does not know the boundary owners and the area of the land) and Morandarte, her overseer since 1973, are not sufficient to prove the

alleged thirty years' possession in the concept of owner by the applicant, her sister, mother and father.

The taxes for 31 years, 1946 to 1976, were paid only in 1976, a few months prior to the filing of the application.

The applicant failed to satisfy the requirements for judicial confirmation of her alleged title (Maloles vs. Director of Lands, 25 Phil. 548). The said land must be presumed to be still a part of the public domain (Oh Cho vs. Director of Lands, 75 Phil. 890).

23(JM)

VICENTE C. REYES vs.FRANCISCO SIERRA et. al., 93 SCRA 472 (G.R.No. L-28658 October 18, 1979)

 

Facts:

  

Vicente Reyes filed an application for registration ofhis title to a parcel of land in Rizal. He declared that he inherited the landfrom his father Vicente Reyes Sr. who died in 1944. An opposition was filed byFrancisco Sierra and Emilio Sierra. The land was originally owned by BasiliaBeltran’s parents. Basilia borrowed from Vicente’s father and secured the loanof P 100 with the piece of land in question. Since the execution of thedocument, the father of Vicente began paying taxes up to his death. Vicenteclaimed it was a contract of sale.

However, the court is of opinion that it is mortgagecontract. The intention of the parties at the time of execution of the contractmust prevail. Registering the property in his own name because of the failureof the mortgagor to redeem the property would constitute pactum commissorium.

Although the document is a contract of mortgage and not acontract of sale, the trial court ruled that the petitioner acquired ownershipover the property by prescription having been in constructive possession of theland since 1926.

The trial court also ruled that applicant and his father’scontinuous payment of the realty taxes constitutes strong corroboratingevidence of applicant's adverse possession. 

 

Issue:

 Whether payment of taxes is a sufficient evidence toprove entitlement to register the property

  

Held:

 

            No. In the case at bar, thepossession of applicant was not adverse, nor continuous.

When possession is asserted to convert itself intoownership, a new right is sought to be created, and the law becomes moreexacting and requires positive proof of title. Applicant failed to presentsufficient evidence to prove that he is entitled to register the property. Thetrial court's finding that since applicant and his father had been continuouslypaying the realty taxes, that fact "constitutes strong corroboratingevidence of applicant's adverse possession," does not carry much weight.Mere failure of the owner to pay the taxes does not warrant a conclusion thatthere was abandonment of a right to the property. The payment of taxes onproperty does not alone constitute sufficient evidence of title.

 

24(Maris)

25(Reg)

26

G.R. Nos. 85991-94          July 3, 1991

REPUBLIC CEMENT CORPORATION, petitioner, vs. COURT OF APPEALS, MOISES CORREA and REGISTER OF DEEDS OF BULACAN, respondents.

 

FACTS:

Republic Cement Corporation filed a petition in the CFI Bulacan, Branch V for the registration in its name of a parcel of land located in Bario Minuyan, Norzagaray, Bulacan. Owners of the east central portion of the subject land opposed the application. A counter application was filed for the two oppositors. The three applications were tried jointly. The CFI rendered in Land Registration Cases in favor of the oppositors, ordering the registration of the claimed lots for Legaspi, spouses Rayo and Mangahas. Mangahas was later substituted by Moises Correa. RCC appealed the CFI decision. The CA upheld the decision of the trial court.

ISSUES:

Whether Legaspi, spouses Rayo and Mangahas were able to prove the genuineness of his title and the identity of the lands he claims for his own in the manner and with the degree of evidence required by law.

HELD:

Petition lacks merit.

Contrary to the belief of RCC, Correa, through his predecessors-in interest, was able to establish the identity of and title of the land sought to be registered in his name. The technical description and the survey plan duly approved by the Director of Lands submitted in evidence by private respondent fully describes the metes and bounds of the parcels of land involved. Survey plan of the property, showing its boundaries and total area, clearly identifies and delineates the extent of the land.

An applicant for registration of land, if he relies on a document evidencing his title thereto, must prove not only the genuineness of said title but also the identity of the land therein referred to. If he only claims a portion of what is included in his title, he must clearly prove that the property sought to be registered is included in that title.

27(Ed)

28(LJ)

29(Zax)

30 

REPUBLIC OF THE PHILIPPINES, represented bythe MINDANAO MEDICAL CENTER, petioner, vs.HON. COURT OF APPEALS and ALEJANDRO Y DE JESUS, respondents. G.R. No.L-40912 September 30, 1976

Facts: On January 22, 1921, Eugenio de Jesus,the father of respondent Alejandro de Jesus, applied with the Bureau of Landsfor Sales Patent situated Davao City.

On January 23, 1934, the Bureau of Lands,through its Davao District Land Officer, accepted sealed bids for the purchaseof the subject land. The Director of Lands, however, annulled the auction salefor the reason that the sales applicant, Eugenio de Jesus, failed toparticipate in the bidding for non-service of notice on him of the scheduledbidding. In lieu of that sale, another bidding was held on October 4, 1934.Sales applicant Eugenio de Jesus was the lone bidder.

On September 7, 1936, President Manuel L.Quezon issued Proclaimation No. 85 withdrawing Lot No. 1176-B-2 from sale andsettlement and reserving the same for military purposes, under theadministration of the Chief of Staff, Philippine Army.

On October 9 1956, President Magsaysayrevoked this Proclamation No. 328 and reserved the same Lot No. 1176-B-2 formedical center site purposes under the administration of the Director ofHospital.

Whereupon, on December 6, 1969, petitionerMindanao Medical Center applied for the Torrens registration of the12.8081-hectare Lot 1176-B-2 with the Court of First

Instance of Davao. TheMedical Center claimed "fee simple" title to the land on the strengthof proclamation No. 350 reserving the area for medical center site purposes.

The Court of appeals denied the petition ofMindanao Medical Center.

Issue: whether or not petitioner MindanaoMedical Center has registerable title over a full 12.8081-hectare land byvirtue of an executive proclamation in 1956 reserving the area for medicalcenter site purposes.

Held: Yes.

Proclamation no. 350 is free of any legalinfirmity. It proceeds from the recognized competence of the president toreserve by executive proclamation alienable lands of the public domain for aspecific public use or service.

Petitioner Mindanao Medical Center hasregisterable title over the whole contested area of 12.8081 hectares,designated Lot No. 1176-B-2, and not only on a portion thereof occupied by theMedical Center, its nervous disease pavilion and their reasonableappurtenances. Such land grant is constitutive of a "fee simple" tileor absolute title in favor of petitioner Mindanao Medical Center. Thus, Section122 of the Act, which governs the registration of grants or patents involvingpublic lands, provides that "Whenever public lands in the PhilippineIslands belonging to the Government of the Philippines arealienated, granted,or conveyed to persons or to public or private corporations, the same shall bebrought forthwith under the operation of this Act [Land Registration Act, Act496] and shall become registered lands."

31(Lea)

DIRECTOR OF LANDS vs. IAC195 SCRA 38FACTS: The application for the registration of the subject lot, which allegedly contained an area of 1,036,172 square meters, was filed by Espartinez on May 17, 1972. He alleged therein that he acquired lot by purchase from Sotera Llacer. He invoked Section 48 of Commonwealth Act No.

141, as amended by Republic Act No. 1942, should the Land Registration Act be not applicable.The jurisdictional requirements of publication of notice of initial hearing and posting of such notices in conspicuous places in the parcel of land involved and in the municipal building having been complied with, and considering that only the Bureau of Lands and the Bureau of Forestry represented by the fiscal had appeared, the lower court issued an order of general default with the exception of said government agencies. Thereafter, one Perpetua Llarena appeared and, together with the fiscal, she was required to file an opposition to the application.Inasmuch as both the fiscal and Llarena failed to file their respective oppositions within the period set by the court, on December 12, 1972, it commissioned the clerk of court to receive evidence. On the same day, however, the Solicitor General entered his appearance for the government and at the same time, filed an opposition to the application for registration. He alleged therein that neither Espartinez nor his predecessors-in-interest had sufficient title to acquire ownership of the land the same not having been acquired by means of any of the various types of title issued by the Spanish government or any other recognized mode of acquisition of title over realty under pertinent laws; that neither Espartinez nor his predecessors-in-interest were in open, continuous, exclusive and notorious possession of the land for at least thirty (30) years prior to the filing of the application; that Espartinez may not avail of the provisions of Section 48 of the Public Land Act for failure to fulfill the requisites prescribed therein; and that the parcel of land involved is part of the public domain and therefore, not subject to private appropriation.ISSUE:Whether the subject parcel of land is alienable and disposable. RULING:No.Espartinez' reliance on Section 48(b) of Commonwealth Act No. 141 is misplaced. That law is premised on the prior classification of the land involved as a disposable agricultural land.Anyone who applies for confirmation of imperfect title under this provision has, under the ruling in Heirs of Amunategui vs. Director of Forestry, the burden of overcoming the presumption that the land sought to be registered forms part of the public domain. Espartinez having failed to present any proof that the land in question has been classified as and forms part of the disposable public domain, whatever possession he might have had, and however long, cannot ripen into private and his failure to adduce clear and convincing evidence of his claim over the land has given rise to the presumption that Lot 6783 is still part of the public domain (Director of Lands vs. Heirs of Juana Carolino, supra).PREMISES CONSIDERED, the land subject of the application for registration and confirmation of imperfect title is hereby DECLARED as part of the public domain.

Part VII

1. REPUBLIC V. CA

154 SCRA 476

 

Facts:

In 1968,Martina Carantes for and in behalf of the heirs of Salming Piraso filed in theCFI of Baguio an application for the registration of the land, which the latterclaimed to be in their possession and occupation openly, continuously,exclusively, notoriously since 1915. The Director of lands, through theSolicitor General and the Director of Forestry, opposed the application on theground that the said portion land is within the Central Cordillera ForestReserve as shown in the reports and testimonies of the district foresters. TheCFI granted the application, which was also affirmed by the CA. Thegovernment’s failure to show that the disputed land is more valuable for forestpurposes is one of the reasons for the CA’s ruling. It also noted the failureto prove that trees are thriving in the land.

Issue:

Whetherthe land in dispute is alienable and disposable.

Ruling:

No. TheCourt ruled that the petitioner clearly proved thru the reports and testimoniesof the district foresters that the land applied for registration is a part of aforestland. As to the claim of the applicants that they have been in possessionof the land since 1915, the court cited its decision in Director of Forestry v.Munoz (23 SCRA 1184),where it stated that possession of forest lands, no matterhow long, cannot ripen into private ownership.

In itsdecision, the Court also addressed the CA’s ruling by citing its decision inHeirs of Amunatequi v. Director of Forestry (126 SCRA 69, 75), where it ruled,―A forested area classified as forest land of the public domain does not losesuch classification simply because loggers or settlers may have stripped it ofits forest cover.

Forestlandsdo not have to be on mountains or in out of the way places. The classificationis descriptive of its legal nature or status and does not have to bedescriptive of what the land actually looks like.

The Courtagain reiterated that there must first be a formal Government declaration thatthe forestland has been re-classified into alienable and disposableagricultural land, before private persons in accordance with the various modesof acquiring public agricultural lands can acquire it.

2. GOMEZ VSCOURT OF APPEALS

168 SCRA503

 

FACTS:

Acourt ruling (Philippine Islands vs Abran) settled that 12 parcels of landbelonged to one Consolacion Gomez. Consolacion later died and the 12 parcels ofland were inherited by Gomez et al – her heirs. The heirs agreed to divide theproperty among them. After notice and publication, and there being noopposition to the application, the trial court issued an order of generaldefault. On 5 August 1981, the court rendered its decision adjudicating thesubject lots in Gomez et al’s favor. The decision became final and executoryhence the court directed the Chief of the General Land Registration Office toissue the corresponding decrees of registration over the lots adjudicated.

GLROChief Silverio Perez opposed the adjudication and petitioned for its settingaside. He discovered that the 12 parcels of land were formerly part of a titledland which was already granted by homestead patent in 1929.   Under the law, land already granted byhomestead patent can no longer be the subject of another registration. Thelower court granted Silverio’s recommendation. Gomez et al invoked Sec. 30 and32 of PD 1529 (Land Registration Act) which provides that after judgment has become final andexecutory, the court shall forthwith issue an order to the Commissioner of LandRegistration for the issuance of the decree of registration and certificate oftitle. That once the judgment becomes final and executory under Sec 30, thedecree of registration must issue as a matter of course.

 

ISSUE:

Whetheror not to set aside the lower court’s initial ruling on approving theadjudication even after it had become final and executory.

HELD:

Yes.Unlike ordinary civil actions, the adjudication of land in a cadastral or landregistration proceeding does not become final, in the sense ofincontrovertibility until after the expiration of one (1) year after the entryof the final decree of registration. TheSupreme Court has held that as long as a final decree has not been entered bythe Land Registration Commission (now NLTDRA) and the period of one (1) yearhas not elapsed from date of entry of such decree, the title is not finally adjudicatedand the decision in the registration proceeding continues to be under thecontrol and sound discretion of the court rendering it.

 

HENCE,the case may still be reopened and the decision set aside when granted.

 3. MENDOZA VS. CA

84 scra 76

Facts:

In 1964, it was proven that aparcel of land located in Sta. Maria, Bulacan, is owned by Mendoza. Mendozaapplied for a title. During pendency of the application before the landregistration court, Mendoza sold the land to Daniel Cruz. The contract of salewas admitted in court in lieu of the pending application for land title. Theregistration court rendered a decision in July 1965, ordering the registrationof the two parcels of land in the name of Cruz subject to the usufructuaryrights of Mendoza.

The decision became final andexecutory. In 1968, however, uponfailure of Cruz to pay Mendoza, Mendoza petitioned that the title issued in thename of Cruz be cancelled. The land registration court ruled in favor ofMendoza on the ground that the court erred in its earlier decision in issuingthe land title to Cruz – who was not a party to the application of titleinitiated by Mendoza. Cruz appealed. The Court of Appeals ruled in favor ofCruz.

Issue:

 

Whether the title can be dealt with in the name of a“third party”.

Ruling:

Yes. The Court ofAppeals ruling must be sustained. First of all, it was proven that Mendozacaused the registration in the name of Cruz pursuant to their contract of sale.Second, Mendoza overlooks Section 29 of the Land Registration Act whichexpressly authorizes the registration of the land subject matter of aregistration proceeding in the name of the buyer (Cruz) or of the person towhom the land has been conveyed by an instrument executed during the intervalof time between the filing of the application for registration and the issuanceof the decree of title.

“SEC. 29. After the filing of theapplication and before the issuance of the decree of title by the Chief of theGeneral Land Registration Office, the land therein described may be dealt withand instruments relating thereto shall be recorded in the office of theregister of deeds at any time before issuance of the decree of title, in thesame manner as if no application had been made. The interested party may,however, present such instruments to the Court of First Instance instead ofpresenting them to the office of the register of deeds, together with a motionthat the same be considered in relation with the application, and the courtafter notice to the parties, shall order such land registered subject to theencumbrance created by a said instruments, or order the decree of registrationissued in the name of the buyer or of the person to whom the property has beenconveyed by said instruments. . . .

A stranger or a third party maybe dealt with in the land registration proceedings. The only requirements ofthe law are: (1) that the instrument be presented to the court by theinterested party together with a motion that the same be considered in relationwith the application; and (2) that prior notice be given to the parties to thecase. And the peculiar facts and circumstances obtaining in this case show thatthese requirements have been complied with in this case.

 4. JULIA CARAGAY-LAYNO VS CA

26 DECEMBER 1984, 133 SCRA 718

 

Facts:

 

Mariano De Vera died in 1951. Hiswidow administered his property until her death in 1966. De Vera’s nephew(Salvador Estrada) took over as administrator of De Vera’s estate. Prior to thewidow’s death, she made an inventory showing that De Vera’s property (locatedin Calasiao, Pangasinan) measures 5417 sq. m (more or less). Estrada howevernoticed that the Torrens title under De Vera indicated that his propertymeasures 8752 sq. m. He learned that the discrepancy is the 3732 sq. m. beingoccupied by Juliana. Estrada sued to evict Juliana. 

 

Juliana averred that she and herfather have been in open, continuous, exclusive and notorious possession and inthe concept of an owner of the land since 1921; that they’ve been paying taxes;that the title held by Estrada was registered in 1947 but it only took them toinitiate an action in 1967 therefore laches has set in. 

 

Issue:

 

Whether the disputed portionshould be adjudged in favor of De Vera’s estate.

Ruling:

 

 No. The inclusion ofJuliana’s land in De Vera’s title was erroneously done. It was shown thatJuliana, an unlettered woman, agreed to have Mariano de Vera borrow her titlefor the purposes of Mariano obtaining a loan during de Vera’s lifetime; thatwhen de Vera registered his portion of land adjoined to that of Juliana, thelatter’s land was erroneously included. 

The error ishighlighted by the fact that de Vera’s widow, in her inventory before she died,attested that de Vera’s portion of land is only 5417 sq. m. more or less. Thediscrepancy approximates the portion of land actually being occupied byJuliana.

By that, the only portion that can be adjudged in favor of de Vera’sestate is that which was being claimed by the widow (in her inventory). Arecalculation must however be made to specify the exact measure of landbelonging to each: 3732 sq m should be retained by Juliana (portion which sheactually occupies) and 5020 sq. m. should go to de Vera’s estate.

 5. DIRECTOR OFLANDS VS THE COURT OF APPEALS

G.R. No. L-17696           

Facts:

On November 19, 1926 a salesapplication was filed with the Bureau of Lands by Benito Tolentino for a tractof public agricultural land with an area of 5 hectares, situated in BarrioCallang Municipality of Gamu, Province of Isabela. In accordance with theapplication, which was given number 8706, the land was advertised for sale tothe highest bidder and on February 15, 1928 was duly awarded to the applicantpursuant to the provisions of Chapter V of the Public Land Act On January 19,1950, Tolentino having complied with the legal requirements as to actualoccupancy, cultivation and improvement of the area applied for as well as thepayment of the purchase price, the Director of Lands signed the correspondingorder for the issuance of a patent in his favor. Upon investigation, however,the applicant discovered that a portion of the land covered by his applicationwith an area of 2.3506 hectares, specifically that portion identified as LotNo. 8091 Pls-62, had been applied for as a homestead by the herein defendantBraulio Cosme on March 22, 1949 and that Homestead Patent No. V-19 had beenissued to him by the Bureau of Lands on the following August 19, pursuant towhich he obtained Original Certificate of Title No. P-880 from the Office ofthe Register of Deeds for the Province of Isabela on November 10, 1949. Thehomestead settlement application had been filed with the now defunct NationalLand Settlement administration pursuant to Executive Proclamation No. 610promulgated in 1940, under which certain areas of public agricultural land inIsabela were reserved for settlement purposes, and it was upon recommendationof that office that the patent was issued by the Director of Lands.

Upon protest by Benito Tolentinofiled with the Bureau of Lands, an investigation was conducted by the DistrictLand Officer for Isabela, and when it was verified that the land covered by thehomestead patent was embraced within the area awarded to Tolentino in 1928, theDirector of Lands filed the present action on November 27, 1953, for thecancellation of the homestead patent and the original certificate of titleissued to the defendant Braulio Cosme.

Their principal contention isthat after the certificate of title was issued on November 10, 1949 by virtueof Homestead Patent No V-19 the land in question came under the operation ofthe Land Registration Act as provided in Section 122 thereof, and that upon theexpiration of one year from the date of its issuance, the said title becameincontrovertible.

Issue:

Whether the court can invalidatethe lands covered by titles based on homestead,free or sales patent.

Ruling:

No. Where a portion of a landsubject of a land registration case are covered by titles based on homestead,free or sales patent, the court cannot simply invalidate them.

A certificate of title based on a patent, even after the expiration ofone year from the issuance thereof, isstill subject to certain conditions and restriction.As a matter of fact, in appropriate cases and after prioradministrative investigations by the Director of Lands, properactions may be instituted by said official which may lead to the cancellationof the patent and the title, and the consequent reversion of the land to theGovernment.On the other hand certificate of title issued pursuant to Act2259, after the lapse of one (1) year, becomes incontrovertible.The inescapableconclusion, therefore, is that, while with the due registration and issuance ofa certificate of title over a land acquired pursuant to the Public Land Law,said property becomes registered in contemplation of Act 496,in view of itsnature and manner of acquisition, such certificate of title, when in conflictwith one obtained on the same date through judicial proceedings, must give wayto the latter.7

A certificate of title issuedpursuant to a homestead patent partakes of the patent to a certificate issuedin a judicial proceeding, as long as the land disposed of is really a partof the disposable land of the public domain (El Hogar Filipino vs. Olviga, 60Phil. 22; Ramoso vs. Obligado, 70 Phil. 86 and others), and becomesindefeasible and incontrovertible upon the expiration of one year from the dateof the issuance thereof, ... a certificate of title, be it original or aduplicate, may only be ordered cancelled under special circumstances, andone of them is when the title is void. And a title will be consideredvoid if it is procured through fraud, as when a person applies for theregistration of a land in his name although he knows that the property belongsto another.

 6. REPUBLIC OF THE PHILIPPINES VS. HON.SOFRONIO G. SAYO

 

G.R. No. L-60413

 

Facts:

 

Spouses Casiano Sandoval and Luz Marquez filed anoriginal application for registration of 33,950 hectares tract of land but wasopposed by the government including the Heirs of Liberato Bayaua . The land wasformerly part of Santiago, Isabela, but had been transferred to Nueva Vizcayain virtue of Republic Act No. 236.

Then, an order of general default was enteredagainst the whole world except the oppositors.

After about 20 years, a compromise agreement wasentered into by the parties. Under the compromise agreement, the Heirs ofCasiano Sandoval renounced their claims and ceded —

1) in favor ofthe Bureau of Lands, an area of 4,109 hectares;

2) in favor ofthe Bureau of Forest Development, 12,341 hectares;

3) in favor ofthe Heirs of Liberato Bayaua, 4,000 hectares; and

4) in favor ofPhilippine Cacao & Farm Products, Inc., 8,000 hectares.

The remaining area of 5,500 hectares was adjudicated to and acknowledgedas owned by the Heirs of Sandoval, but out of this area, 1,500 hectares wereassigned to their counsel, Jose C. Reyes, in payment of his attorney's fees.

The respondent Judge approved the compromise agreement and confirmed thetitle and ownership of the parties in accordance with its terms.

The applicants for registration presented a mere photocopy of acertification of the National Library stating that the property in question wasregistered under the Spanish system of land registration as private property.But, that Spanish document cannotbe considered a title to property, it not being one of the grants made duringthe Spanish regime, and obviously not constituting primary evidence ofownership.

 

Issues:

Whether the Registration Court correctly rendereddecision based from the

compromise agreement of the parties.

Whether informacion posesoria is a prima facieevidence of possession.

Ruling:

The compromise agreement and the judgmentapproving it is declared null and void.

The decision of the Registration Court aquo is based solely on thecompromise agreement of the parties but such included private persons who hadnot adduced any competent evidence of their ownership over the land subject ofthe registration proceeding. Portions of the land in controversy were assignedto persons or entities who had presented nothing whatever to prove their ownershipof any part of the land. What was done was to consider the compromise agreementas proof of title of the parties taking part therein, a totally unacceptableproposition. The result has been the adjudication of lands of no littleextension to persons who had not submitted any substantiation at all of theirpretensions to ownership, founded on nothing but the agreement among themselvesthat they had rights and interests over the land.

Under the Spanish Mortgage Law, informacion posesoria was considered a mode of acquiringtitle to public lands, subject to two conditions: first, the inscriptionthereof in the Registry of Property, and second, actual, public, adverse, anduninterrupted possession of the land for 20 years; but where, as here, proof offulfillment of these conditions is absent.

7. THE DIRECTOR OF LANDS et.al. VS HON. SALVADOR C. REYES 

G.R. No. L-27594

Facts:

 

On February 24, 1964,the applicant Alipio Alinsunurin, claiming ownership in fee simple byinheritance from the late Maria Padilla, sought the registration of title underAct 496, as amended, of a vast tract of land, situated at the municipality ofLaur, province of Nueva Ecija, admittedly inside the boundary of the militaryreservation of Fort Magsaysay.

The Director of Lands,Director of Forestry, and the Armed Forces of the Philippines opposed theapplication, claiming that the applicant was without sufficient title and wasnot in open, exclusive, continuous and notorious possession and occupation ofthe land in question for at least thirty (30) years immediately preceding thefiling of the application; that approximately 13,957 hectares of said landconsist of the military reservation of Fort Magsaysay established underProclamation No. 237, dated December 10, 1955 of the President.

The applicant AlipioAlinsunurin filed a motion for substitution of parties, requesting that theParañaque Investment and Development Corporation be considered as the applicantin his place, it having acquired all his rights, interests, ownership anddominion over the property subject matter of the application. The motion was granted by the lowercourt.

It is beyond disputethat the land subject of the application is included within the area reservedfor military purposes under Proclamation No. 237, dated December 19, 1955, ofthe President. The land is largely uncultivated, mountainous and thicklyforested with a heavy growth of timber of commercial quantities. 

It is claimed by theapplicant that Melecio Padilla acquired the land by virtue of a possessoryinformation title issued during the Spanish regime on March 5, 1895, and uponhis death in 1900, he transmitted the ownership and possession thereof to hisdaughter and sole heir, Maria Padilla. The latter in turn continued tocultivate the land thru tenants and utilized portions for pasture, until herdeath sometime in 1944.

On November 19, 1966,the lower court rendered decision holding that the parcel of land applied foris adjudicated to and ordered to be registered in favor of:

(a) Parañaque Investment and DevelopmentCorporation, a Philippine corporation wholly owned by Filipino citizens, withaddress at Manila, Philippines, two-thirds (2/3) portion, subject to the rightsof Ariosto Santos and

(b) Roman C. Tamayo, Filipino citizen, married,resident of Cullit, Lallo, Cagayan, one-third (1/3) portion of the saidproperty.

The oppositors Director of Lands, Director ofForestry and the Armed Forces of the Philippines filed a Notice of Appeal fromthe said decision to the Supreme Court.

By an order, the lower court required theProvincial Fiscal to file an Amended Record on Appeal, so as to include thereincertain orders and pleadings, within ten days from receipt of the order. OnMarch 16, 1967, the Amended Record on Appeal was duly filed and copies servedupon the appellees.

Pending the approval of the Record on Appeal, theapplicant Parañaque Investment and Development Corporation filed a motion forthe issuance of a decree of registration pending appeal. Likewise, Roman C.Tamayo, thru counsel, filed a motion for the issuance of a decree ofregistration. Both motions were opposed by the Government.

On March 11, 1967, the lower court, ruling thatits decision of November 19, 1966 had become final as to the share of Roman C.Tamayo, directed the issuance of a decree of registration of the entire land,one-third (1/3) pro-indiviso in favor of Roman C. Tamayo, andtwo-thirds (2/3) proindiviso in favor ofParañaque Investment and Development Corporation, subject to the final outcomeof the appeal.

On March 14, 1967, the Commissioner of LandRegistration issued Decree No. 113485 pursuant to the said order, and, on March15, 1967, the Register of Deeds issued Original Certificate of Title No. 0-3151of the Register of Deeds of the Province of Nueva Ecija.

On April 12, 1967, the lower court approved theAmended Record on Appeal which, together with the evidence and transcripts, wasforwarded to this Court in due course of appeal.

As the lower court denied reconsideration of theorder directing the issuance of a decree of registration, the petitionersinstituted before the Supreme Court a special civil action for certiorari andmandamus with preliminary injunction, seeking to nullify the order dated March11, 1967, the decree of registration issued pursuant thereto and OriginalCertificate of Title of the Register of Deeds for the province of Nueva Ecija.

Issue:

 

Whether thelower court acted without jurisdiction or exceeded its jurisdiction in orderingthe issuance of a decree of registration despite the appeal timely taken fromthe entire decision a quo. 

 

Ruling:

 

The appeal takenby the Government was from the entire decision, which is not severable. Thus,the appeal affects the whole decision.  

In any event, Werule that execution pending appeal is not applicable in a land registrationproceeding. It is fraught with dangerous consequences. Innocent purchasers maybe misled into purchasing real properties upon reliance on a judgment which maybe reversed on appeal. 

A Torrens titleissued on the basis of a judgment that is not final is a nullity, as it isviolative of the explicit provisions of the Land Registration Act whichrequires that a decree shall be issued only after the decision adjudicating thetitle becomes final and executory, and it is on the basis of said decree thatthe Register of Deeds concerned issues the corresponding certificate of title. 

Consequently,the lower court acted without jurisdiction or exceeded its jurisdiction inordering the issuance of a decree of registration despite the appeal timelytaken from the entire decision a quo. 

 8. ATTY. JOSE S. GOMEZ  et,al. VS HON. COURT OF APPEALS,

 

G.R. No. 77770

 

Facts:

Petitionersapplied for registration of several lots situated in Bayambang, Pangasinan onAugust 30, 1968.The lots were among those involved in the case of Governmentof the Philippine Islands vs. Abran, whereinthe Supreme Court declared Consolacion M. Gomez owner of certain lots in SitioPoponto Bayambang, Pangasinan. Petitioners are the heirs of Teodoro Y. Gomez(father of Consolacion).

Afternotice and publication, and there being no opposition to the application, thetrial court issued an order of general default. On 5 August 1981, the courtrendered its decision adjudicating the subject lots in petitioners' favor.

OnOctober 6, 1981, the trial court issued an order expressly stating that the decisionof August 5, 1981 had become final anddirected the Chief of the General Land Registration Office to issue the correspondingdecrees of registration over the lots adjudicated in the decision of August 5,1981.

OnJuly 11, 1984, respondent Silverio G. Perez, Chief of the Division of OriginalRegistration, Land Registration Commission submitted a report to the court aquo stating that portionsof the land sought for registration were covered by homestead patents issued in1928 and 1929 and registered under the Land Registration Act. He recommendedthat the decision of August 5, 1981 and the order of October 6, 1981 be setaside. Petitioners opposed the report, pointing out that no opposition wasraised by the Bureau of Lands during the registration proceedings and that thedecision of August 5, 1981 should be implemented because it had long becomefinal and executory.

Afterhearing, the lower court rendered a second decision setting aside the decisiondated August 5, 1981 and the order dated October 6, 1981 for the issuance ofdecrees. Petitionersmoved for reconsideration but the motion was denied. AHence, this recourse.

Issue:

Whetherthe decision dated August 5, 1981 had become final and executory, that it mayno longer be reopened, reviewed, or set aside.

Ruling:

Petitionersanchor their claim on section 30 of P.D. No. 1529 (Property RegistrationDecree) which provides that, after judgment has become final and executory, thecourt shall forthwith issue an order to the Commissioner of Land Registrationfor the issuance of the decree of registration and certificate of title.Petitioners contend that section 30 should be read in relation to section 32 ofP.D. 1529 in that, once the judgment becomes final and executory under section30, the decree of registration must issue as a matter of course. This being thelaw, petitioners assert, when respondent Judge set aside in his decision, dated25 March 1985, the decision of 5 August 1981 and the order of 6 October 1981,he clearly acted without jurisdiction.

Petitioners'contention is not correct. Unlike ordinary civil actions, the adjudication ofland in a cadastral or land registration proceeding does not become final, inthe sense of incontrovertibility until after the expiration of one (1) yearafter the entry of the final decree of registration. 

Petition denied.

 9. CAYANAN V. DE LOS SANTOS

21 SCRA 1348

 

Facts:

The title of appellee De Los Santos to Lot 56 of Porac Cadastrewas confirmed by Judge Santos of the Court of First Instance of Pampanga.

In the same year, there was a petition for review for the saidlot. It was alleged that said lot was registered in the name of appellee De losSantos through actual fraud, through deceit and through intentionalomission of facts. It was stated further that a simulated Deed ofAbsolute Sale was executed in favor of the other respondent appellee,Felix Camaya.

It prayed further for the opening of the decree ofregistration, the cancellation of the Original Certificate of Title, aswell as the Transfer Certificate of Title and the adjudication of said lotin favor of petitioners, now appellant Cayanan, however it wasdenied.

Issue:

 

Whether the case may still be reopened and the decision set asidewhen granted

Ruling:

 

Yes,it may.

As long as the final decree is not issued by theChief of the General Land Registration Office in accordance with the law, andthe period of one year filed for the review thereof has not elapsed, the titleis not finally adjudicated and the decision therein rendered continues to beunder the control and sound discretion of the court rendering it.

10. HEIRS OF CRISTOBAL MARCOS vs. DE BANUVAR

G.R. No. L-22110.

FACTS:

On March 24, 1938 the CFI rendered a decision confirming thetitles of La Urbana, Inc. over lot 5 and lot 1, Psu-56145, with certainreservations, and ordered the registration of these lots in favor of thelatter.On May 17, 1960 de Erquiaga, one of the successors-in- interest of LaUrbana, Inc. filed a petition for reconstitution of the decision of March 24,1938. During the pendency of the reconstitution proceedings, the respondent DeBanuvar acquired lot 1 from de Erquiaga, who was thus substituted as a partyfor the latter.

 However, the petitionersopposed the application for the reconstitution, on a claim that they have beenin actual, adverse, open and uninterrupted possession and occupation of thesaid parcel in the concept of owners since time immemorial, long before theWorld War II, and have introduced improvements thereon. They prayed that theapplication for the reconstitution of records be denied and that "theparcels of land in question be ordered registered in the respective names ofthe herein oppositors or declare the same as public land and be subdivided tooppositors who are landless." OnJune 13, 1963 the respondent court ordered the issuance of a decree in favor ofDe Banuvar with respect to lot 1 only, after finding that the decision in theland registration case had already become final and executory. Moreover, thecourt issued another order granting a writ of possession in favor of De Banuvarand "against all persons who have entered and occupied portions of lot 1,Psu-56145 before the issuance of the decree."

ISSUE:

Whether the issuanceof the order is proper.

RULING:

 In the opposition to thepetition for reconstitution, the petitioners' alleged possession and occupationof portions of lot 1 arose prior to or during the registration proceedings. Forthis reason, the order of granting a writ of possession in favor of De Banuvaragainst the petitioners, is proper and justified. The petitioners hereinadmittedly took possession and occupation of portions of lot 1 prior to July 1,1963 when the decree in question was issued. The fundamental rule is that awrit of possession can be issued not only against the original oppositors in aland registration case and their representatives and successors-in-interest,but also against any person unlawfully and adversely occupying said lot at anytime before and up to the issuance of the final decree. 

 Furthermore, there is noprovision in the Land Registration Act similar to Sec. 6, Rule 39, regardingthe execution of a judgment in civil action, except the proceedings to placethe winner in possession by virtue of a writ of possession. The decision in aland registration case unless the adverse or losing party is in possession,becomes final without any further action, upon the expiration of the period forperfecting an appeal. 

There is nothing in the law that limits the period within whichthe court may order or issue a decree. The reason is ... that the judgment ismerely declaratory in character and does not need to be asserted or enforcedagainst the adverse party. Furthermore, the issuance of a decree is aministerial duty both of the judge and of the Land Registration Commission;failure of the court or of the clerk to issue the decree for the reason that nomotion therefor has been filed cannot prejudice the owner, or the person inwhom the land is ordered to be registered.

 11. EUFEMIA VILLANUEVA VDA. DE BARROGA,et al. VS.ANGEL ALBANO,et al.

G.R. No. L-43445 January 20, 1988

 

Facts:

On July 31, 1941, CFI Ilocos Norte rendered a decision inCadastral Proceeding No. 44 adjudicating a parcel of land known as Lot No. 9821in favor of Delfina Aquino.One of the oppositors was Ruperta Pascual, who wasdeclared in default. However, thedecree of registration was only issued on October 14, 1955; and it was only onNovember 17, 1979, that an original certificate of title was issued in DelfinaAquino's name.On August 11, 1970, EufemiaBarroga and Saturnina Padaca,the children and heirs of Ruperta Pascual, broughtsuit in the same Court of First Instance praying that Delfina Aquino's title bevoided and cancelled, and that title be adjudicated in their favor. Barroga'sand Padaca's complaint was denied by the court. Thereafter, the Court of FirstInstance ordered execution of the judgment on December 6, 1973. On August 8,1975, the Cadastral Court promulgated an order granting the motion of AngelAlbano, et al. for a writ of possession as regards Lot No. 9821. A writ ofpossession dated August 28, 1975 was issued. The plaintiffs appealed. Themotion was thereafter denied by the court by Order dated September 22, 1975. Hence, this petition.

Issue:

Whether theCourt could still issue orders despite lapse of long period of time from entryof judgment.

Held:

The judgment of the cadastral court was ajudgment in rem,binding generally upon the whole world, inclusive of persons not partiesthereto, and particularly uponthose who had actually taken part in the proceeding as well as their successorsin interest by title subsequent to the commencement of the action or specialproceeding, litigating for the same thing and under the same title and in thesame capacity. The writ ofpossession could properly issue despite the not inconsiderable period of timethat had elapsed from the date of the registration decree, since the right tothe same does not prescribe. The appellees are entitled to said writ ofpossession, despite the lapse of many years, their right thereto beingimprescriptible at least as against the persons who were parties to thecadastral case or their successors-in-interest.

 12. MAMERTA DE LA MERCED VS.COURT OF APPEALS

5 SCRA 240

Facts:

Ezequiel Santos (and his wife) is claiming ownership of LotNo. 395 of the Rizal Cadastre by virtue of an adjudication of the cadastralcourt in favor of his father. Defendants, in their answer, resisted plaintiffs'claim and asserted their ownership over said property as evidenced by an OCTissued to their predecessor Juan de la Merced and their continuous possessionof the land for more than 30 years. Mamerta de la Merced, a legitimate daughterof Juan de la Merced, was allowed to intervene and make common cause with thedefendants. The court rendered a decision for the plaintiffs after making afinding the lot was part of the OCT in the name of the spouses Inocencio de losSantos and Victorina Macapagal, parents of plaintiff Ezequiel Santos. Thecadastral court declared the lot a public land, as a consequence of which Juande la Merced, after filing a homestead application therefor, was able to obtainOriginal Certificate of Title. Holding that the cadastral court had nojurisdiction to issue the order declaring the lot public land, and, therefore,the same as well as the certificate of title issued thereafter was null andvoid, the court ordered the cancellation of OCT in the name of Juan de laMerced; directed defendants to vacate Lot. While the court held that the landhaving ceased to be part of the public domain, the Director of Lands no longerhad authority to grant the homestead patent over the same to Juan de la Merced,Plaintiffs interposed an appeal to the Court of Appeals.. Hence, the filing of theinstant petition for review.

Issue:

Whether the property thereby affected still be lost byadverse possession.

Ruling: 

A decree of registration and a certificate of title, under Act496, are two different things. And it is the decree of registration, to beissued by the Land Registration Commissioner, which shall be the basis of thecertificate of title to be issued subsequently by the corresponding register ofdeeds, that quiets title to and bindsthe land. The title of ownership on the land is vested upon the owner upon theexpiration of the period to appeal from the decision or adjudication by thecadastral court, without such an appeal having been perfected. The certificateof title would then be necessary for purposes of effecting registration of subsequentdisposition of the land where court proceedings would no longer be necessary.

As we have here a decree issued by the cadastral court, orderingthe issuance to Inocencio de los Santos of the certificate of title over LotNo. 395 after the decision adjudicating ownership to him of the said propertyhad already become final, and there being no imputation of irregularity in thesaid cadastral proceedings, title of ownership on the said adjudicatee wasvested as of the date of the issuance of such judicial decree. The land, forall intents and purposes, had become, from that time, registered property whichcould not be acquired by adverse possession.

13(Douglas)

14(Daniel Eblahan)

Eland Philippines Inc. v. GarciaGR no. 173289, February 17, 2010Peralta, J.:Facts:  The respondents owned a parcel of land in Tagaytay City. They were occupying the possessing said land continuously, publicly, and adversely for at least thirty years, under the Public Land Act. They later found out that the petitioner has applied to register the land with the Regional Trial Court and that the same court has already issued a decree. They filed a Quieting of Title with the same court against Eland Philippines. In a summary judgment, the same trial court ruled in favor of the respondents as the absolute owners and rightful possessors of the land and set aside its earlier decree. The Court of Appeals dismissed the case. Issue:  Whether or not the summary judgment was properHeld:  No. The Court reversed the lower courts’ decision, ruling that summary judgment is appropriate when there are no genuine issues of fact. In this case, a genuine issue of fact has been raised by the respondents that tend to prove the invalidity of the petitioner’s claim. These issues can only be resolved in a full trial.  Also, the Court ruled that the quieting of title was not the proper remedy but a petition for review of the decree of registration. This petition may be availed a person deprived of land or any interest or estate by reason of fraud when the following requisites are satisfied:  1. Petitioner must have an estate or interest in the land.  2. He must show actual fraud in the procurement of the decree of registration  3. The petition must be filed within one year from the issuance of the final decree by the Land Registration Authority  4. The property has not yet passed to an innocent purchaser for value.  In this case, the period of one year before the decree becomes final and incontrovertible has not yet lapsed when they filed the complaint for the quieting of title and because the decree was not yet issued by the Land Registration Administrator.Significance:A review of a decree is allowed when the person is deprived of land or his interest or estate within due to fraud. The Court also laid down the rules on how to avail said remedy. T Court also

noted that the petition may be filed after the decision of the court up until before the one year expiration period from the entry of the final decree issued by the Land Registration Administrator.

15(Douglas)

16(Mel)

HEIRS OF CRISTOBAL MARCOS vs. DE BANUVAR

G.R. No. L-22110.

FACTS:

On March 24, 1938 the CFI rendered a decision confirming the titles of La Urbana, Inc. over lot 5 and lot 1, Psu-56145, with certain reservations, and ordered the registration of these lots in favor of the latter.On May 17, 1960 de Erquiaga, one of the successors-in- interest of La Urbana, Inc. filed a petition for reconstitution of the decision of March 24, 1938. During the pendency of the reconstitution proceedings, the respondent De Banuvar acquired lot 1 from de Erquiaga, who was thus substituted as a party for the latter.

 However, the petitioners opposed the application for the reconstitution, on a claim that they have been in actual, adverse, open and uninterrupted possession and occupation of the said parcel in the concept of owners since time immemorial, long before the World War II, and have introduced improvements thereon. They prayed that the application for the reconstitution of records be denied and that "the parcels of land in question be ordered registered in the respective names of the herein oppositors or declare the same as public land and be subdivided to oppositors who are landless."  On June 13, 1963 the respondent court ordered the issuance of a decree in favor of De Banuvar with respect to lot 1 only, after finding that the decision in the land registration case had already become final and executory. Moreover, the court issued another order granting a writ of possession in favor of De Banuvar and "against all persons who have entered and occupied portions of lot 1, Psu-56145 before the issuance of the decree."

ISSUE:

Whether the issuance of the order is proper.

RULING:

 In the opposition to the petition for reconstitution, the petitioners' alleged possession and occupation of portions of lot 1 arose prior to or during the registration proceedings. For this reason, the order of granting a writ of possession in favor of De Banuvar against the petitioners, is proper and justified. The petitioners herein admittedly took possession and occupation of portions of lot 1 prior to July 1, 1963 when the decree in question was issued. The fundamental rule is that a writ of possession can be issued not only against the original oppositors in a land registration case and their representatives and successors-in-interest, but also against any person unlawfully and adversely occupying said lot at any time before and up to the issuance of the final decree. 

 Furthermore, there is no provision in the Land Registration Act similar to Sec. 6, Rule 39, regarding the execution of a judgment in civil action, except the proceedings to place the winner in possession by virtue of a writ of possession. The decision in a land registration case unless the adverse or losing party is in possession, becomes final without any further action, upon the expiration of the period for perfecting an appeal. 

There is nothing in the law that limits the period within which the court may order or issue a decree. The reason is ... that the judgment is merely declaratory in character and does not need to be asserted or enforced against the adverse party. Furthermore, the issuance of a decree is a ministerial duty both of the judge and of the Land Registration Commission; failure of the court or of the clerk to issue the decree for the reason that no motion therefor has been filed cannot prejudice the owner, or the person in whom the land is ordered to be registered.

17(Douglas)

18(Maribeth)

Manotok vs. Barque, G.R.No. 162335, December 12, 2005

FACTS:

Respondents Barque, filed a petitionwith the LRA for administrative reconstitution of the original copy of TCT No.210177 issued in the name of Homer L. Barque, which was destroyed in the firethat gutted the Quezon City Hall, including the Office of the Register of Deedsof Quezon City. In support of said petition, they submitted the owner’sduplicate copy of TCT No. 210177, real estate tax receipts, tax declarationsand the Plan FLS 3168 D covering the property.

Upon being notified of the petition foradministrative reconstitution, Manotok filed their opposition thereto claimingthat the lot covered by the title under reconstitution forms part of the landcovered by their reconstituted title TCT No. RT-22481, and alleging that TCTNo. 210177 in the name of petitioners’ predecessors-in-interest is spurious.

On June 30, 1997, Atty. Benjamin M.Bustos, as reconstituting officer, denied the reconstitution of TCT No.210177 on grounds that:

1. Lots 823-A and 823-B, Fls-3168-D,containing areas of 171,473 Sq. Mtrs. and 171,472 Sq. Mtrs., respectively,covered by TCT No. 210177, appear to duplicate Lot 823 Piedad Estate,containing an area of 342,945 Sq. Mtrs., covered by TCT No. 372302 registeredin the name of Severino M. Manotok, et. al., reconstituted under Adm.Reconstitution No. Q-213 dated February 01, 1991;

2. The submitted plan Fls-3168-D is aspurious document as categorically stated by Engr. Privadi J.G. Dalire, Chief,Geodetic Surveys Division, Land Management Bureau, in his letter dated February19, 1997.<a></a><a>6</a>

Respondents’ motion for reconsiderationwas denied hence they appealed to the LRA which ruled that the reconstitutingofficer should not have required the submission of documents other than theowner’s duplicate certificate of title as bases in denying the petition andshould have confined himself with the owner’s duplicate certificate oftitle. The LRA further declared that based on the documents presented, theBarques have established by clear and convincing evidence that TCT NO. 210177,at the time of the destruction thereof, was valid, genuine, authentic andeffective. They duly presented the original of the owner’s duplicate copy ofTCT No. 210177. The logbook of the Register of Deeds of Quezon City lists TCTNo. 210177 as among the titles lost. The Register of Deeds of Quezon Cityhimself acknowledged the existence and authenticity of TCT No. 210177 when heissued a certification to the effect that TCT No. 210177 was one of the titlesdestroyed and not salvaged from the fire that gutted the Quezon City Hall.

It is likewise noteworthy that thetechnical description and boundaries of the lot reflected in TCT No. 210177absolutely conform to the technical description and boundaries of Lot 823Piedad Estate ... as indicated in the B. L. Form No. 28-37-R dated 11-8-94 andB. L. Form No. 31-10 duly issued by the Bureau of Lands.

It therefore becomes evident that theexistence, validity, authenticity and effectivity of TCT No. 210177 wasestablished indubitably and irrefutably by the petitioners. Under suchcircumstances, the reconstitution thereof should be given due course and thesame is mandatory.

Nevertheless, notwithstanding the conclusionthat Respondents’ title was fraudulently reconstituted, the LRA noted that itis only the Regional Trial Court (RTC) which can declare that the same wasindeed fraudulently reconstituted. It thus opined that respondents’ title mayonly be reconstituted after a judicial declaration that petitioners’ title wasvoid and should therefore be cancelled.

Petitioners’ filed a motion forreconsideration which was opposed by respondents with a prayer thatreconstitution be ordered immediately. Both were denied.

From the foregoing, respondents filed apetition for review with the Court of Appeals docketed as CA-G.R. SP No.66700 and praying that the LRA be directed to immediately reconstitute TCT No.210177 without being subjected to the condition that petitioners’ TCT No.RT-22481 [372302] should first be cancelled by a court of competentjurisdiction. Petitioners likewise filed a petition for review with theCourt of Appeals docketed as CA-G.R. SP No. 66642.

In CA-G.R. SP No. 66700, the SecondDivision of the Court of Appeals rendered a Decision the dispositive portion ofwhich reads:

WHEREFORE, the foregoing premisesconsidered the assailed Resolution of the LRA dated June 24, 1998 is AFFIRMEDin toto and the petition for review is ordered DISMISSED.Respondents moved for reconsideration. On November 7, 2003, the SpecialDivision of Five of the Former Second Division rendered an Amended Decision inCA-G.R. SP No. 66700, the dispositive portion of which reads:

WHEREFORE, our decision dated 13September 2002 is hereby reconsidered. Accordingly, the Register of Deeds ofQuezon City is hereby directed to cancel TCT No. RT-22481 of privaterespondents and the LRA is hereby directed to reconstitute forthwithpetitioners’ valid, genuine and existing Certificate of Title No. T-210177.

Petitioners’ motion for reconsiderationof the amended decision in CA-G.R. SP No. 66700 was denied, hence, thepetition docketed as G.R. No. 162605.

Meanwhile, in CA-G.R. SP No. 66642, theThird Division of the Court of Appeals rendered a Decision on October 29,2003, the dispositive portion of which reads:

WHEREFORE, the petition is herebyDENIED. The Resolution of the LRA dated 24 June 1998 is hereby AFFIRMED.

In so ruling, the Third Division of theCourt of Appeals declared that the LRA correctly deferred in giving due courseto the petition for reconstitution since there is yet no final judgmentupholding or annulling respondents’ title.<a></a><a></a>

Respondents’ motion for reconsiderationwas granted by the Third Division of the Court of Appeals on February 24, 2004,thus:

WHEREFORE, the Motion forReconsideration is hereby GRANTED. The Decision of this Court dated 29 October2003 is RECONSIDERED and a new one is entered ordering the Register of Deeds ofQuezon City to cancel petitioners’ TCT No. RT-22481 and directing the LRA toreconstitute forthwith respondents’ TCT No. T-210177.

From the foregoing decisions of theCourt of Appeals in CA-G.R. SP No. 66700 and CA-G.R. SP No. 66642, petitionersfiled separate petitions for review before the Supreme Court docketed as G.R.No. 162605 and G.R. No. 162335, respectively.

ISSUE: Whether or not the LRA could proceed with thereconstitution of the tile?

HELD: Yes. The court held that the LRA properly ruled that the reconstitutingofficer should have confined himself to the owner’s duplicate certificate oftitle prior to the reconstitution. Section 3 of Republic Act (RA) No. 26 clearlyprovides:

Section 3. Transfer certificates oftitle shall be reconstituted from such of the sources hereunder enumerated asmay be available, in the following order:

(a) The owner’sduplicate of the certificate of title; 

When respondents filed the petition forreconstitution, they submitted in support thereof the owner’s duplicatecertificate of title, real estate tax receipts and tax declaration. Plainly,the same should have more than sufficed as sources for the reconstitutionpursuant to Section 3 of RA No. 26 which explicitly mandates that thereconstitution shall be made following the hierarchy of sources asenumerated by law. In addition, Section 12 of the same law requires that thepetition shall be accompanied with a plan and technical description of theproperty only if the source of the reconstitution is Section 3(f)of RA No. 26. Thus:

Section 12. … Provided, That in casethe reconstitution is to be made exclusively from sourcesenumerated in section 2(f) or 3(f) of this Act, the petition shall further beaccompanied with a plan and technical description of the property duly approvedby the Chief of the General Land Registration Office, or with a certified copyof the description taken from a prior certificate of title covering the sameproperty.<a></a><a>29</a>

Since respondents’ source ofreconstitution is the owner’s duplicate certificate of title, there is no needfor the reconstituting officer to require the submission of the

plan, much lessdeny the petition on the ground that the submitted plan appears to be spurious.By enumerating the hierarchy of sources to be used for the reconstitution, itis the intent of the law to give more weight and preference to the owner’sduplicate certificate of title over the other enumerated sources.

19(Ailyn) 

NATURE OF RECLAIMED LANDS

G.R. No. 133250 July 9,2002

FRANCISCO I. CHAVEZ, PETITIONER, VS. PUBLIC ESTATES AUTHORITY AND AMARICOASTAL BAY DEVELOPMENT CORPORATION, RESPONDENTS

FACTS

On November 20, 1973, the government, through theCommissioner of Public Highways, signed a contract with the Construction andDevelopment Corporation of the Philippines (CDCP) to reclaim certain foreshoreand offshore areas of Manila Bay. On February 4, 1977, then President FerdinandE. Marcos issued Presidential Decree No. 1084 creating PEA - tasked “to reclaim land, including foreshoreand submerged areas,” and “to develop, improve, acquire, x x x lease and sellany and all kinds of lands.” On the same date, then President Marcos issuedPresidential Decree No. 1085 transferring to PEA the “lands reclaimed in theforeshore and offshore of the Manila Bay” under the Manila-Cavite Coastal Roadand Reclamation Project (MCCRRP).

OnJanuary 19, 1988, then President Corazon C. Aquino issued Special Patent No.3517, granting and transferring to PEA “the parcels of land so reclaimed underthe Manila-Cavite Coastal Road and Reclamation Project (MCCRRP) .Subsequently,on April 9, 1988, the Register of Deeds of the Municipality of Parañaque issuedTransfer Certificates of Title in the name of PEA, covering the three reclaimedislands known as the “Freedom Islands” located at the southern portion of theManila-Cavite Coastal Road, Parañaque City. On April 25, 1995, PEA entered intoa Joint Venture Agreement (JVA) with AMARI, a private corporation, to developthe Freedom Islands. PEA and AMARI entered into the JVA through negotiationwithout public bidding. On June 8, 1995, then President Fidel V. Ramos, throughthen Executive Secretary Ruben Torres, approved the JVA.

OnNovember 29, 1996, then Senate President Ernesto Maceda delivered a privilegespeech in the Senate and denounced the JVA as the “grandmother of all

scams.”As a result, the Senate Committee on Government Corporations and PublicEnterprises, and the Committee on Accountability of Public Officers andInvestigations, conducted a joint investigation. Among the conclusions of theirreport are: (1) the reclaimed lands PEA seeks to transfer to AMARI under theJVA are lands of the public domain which the government has not classified as alienablelands and therefore PEA cannot alienate these lands; (2) the certificates oftitle covering the Freedom Islands are thus void, and (3) the JVA itself isillegal.

OnDecember 5, 1997, then President Fidel V. Ramos issued PresidentialAdministrative Order No. 365 creating a Legal Task Force to conduct a study onthe legality of the JVA. The Legal Task Force upheld the legality of the JVA,contrary to the conclusions reached by the Senate Committees.

On April27, 1998, petitioner Frank I. Chavez (“Petitioner” for brevity) as a taxpayer,filed the instant Petition for Mandamus with Prayer for the Issuance of aWrit of Preliminary Injunction and Temporary Restraining Order. Petitionercontends the government stands to lose billions of pesos in the sale by PEA ofthe reclaimed lands to AMARI. Petitioner prays that PEA publicly disclose theterms of any renegotiation of the JVA, invoking Section 28, Article II, andSection 7, Article III, of the 1987 Constitution on the right of the people toinformation on matters of public concern. Petitioner assails the sale to AMARIof lands of the public domain as a blatant violation of Section 3, Article XIIof the 1987 Constitution prohibiting the sale of alienable lands of the publicdomain to private corporations. In a Resolution dated March 23, 1999, the Courtgave due course to the petition and required the parties to file theirrespective memoranda.

On March30, 1999, PEA and AMARI signed the Amended Joint Venture Agreement (Amended JVA).On May 28, 1999, the Office of the President under the administration of thenPresident Joseph E. Estrada approved the Amended JVA.

Due tothe approval of the Amended JVA by the Office of the President, petitioner nowprays that on “constitutional and statutory grounds the renegotiated contractbe declared null and void.”

ISSUE

WHETHERTHE Lands reclaimed from foreshore andsubmerged areas also form part of the public domain and are also inalienable

WHETHERTHE STIPULATIONS IN THE AMENDED JOINT VENTURE AGREEMENT FOR THE TRANSFER TOAMARI OF CERTAIN LANDS, RECLAIMED AND STILL TO BE RECLAIMED, VIOLATE THE 1987CONSTITUTION

RULING

The 1987 Constitution, like the 1935 and 1973 Constitutions before it,has adopted the Regalian doctrine. The 1987 Constitution declares that allnatural resources are “owned by the State,” and except foralienable agricultural lands of the public domain, natural resources cannot bealienated.

UnderSection 2, Article XII of the 1987 Constitution, the foreshore and submergedareas of Manila Bay are part of the “lands of the public domain, waters x x xand other natural resources” and consequently “owned by the State.” As such,foreshore and submerged areas “shall not be alienated,” unless they areclassified as “agricultural lands” of the public domain. The mere reclamationof these areas does not convert these inalienable natural resources of theState into alienable or disposable lands of the public domain. There must be alaw or presidential proclamation officially classifying these reclaimed landsas alienable or disposable and open to disposition or concession. Moreover,these reclaimed lands cannot be classified as alienable or disposable if thelaw has reserved them for some public or quasi-public use.

Landsreclaimed by the government are sui generis, not available forsale to private parties unlike other alienable public lands. Reclaimed landsretain their inherent potential as areas for public use or public service.

Clearly,the Amended JVA violates glaringly Sections 2 and 3, Article XII of the 1987Constitution. Under Article 1409 of the Civil Code, contracts whose “object orpurpose is contrary to law,” or whose “object is outside the commerce of men,”are “inexistent and void from the beginning.” The Court must perform its dutyto defend and uphold the Constitution, and therefore declares the Amended JVAnull and void ab initio.

 TAX DECLARATION

G.R. No. 177797, December 04, 2008

SPS. PEDRO TAN AND NENA ACERO TAN,PETITIONERS, VS. REPUBLIC OF THE PHILIPPINES, RESPONDENT

FACTS

The spouses Pedro Tan and Nena AceroTan were natural-born Filipino citizens, who became Australian citizens on 9February 1984. They seek to have thesubject property registered in their names. The subject property was declaredalienable and disposable on 31 December 1925, as established by a Certificationdated 14 August 2000 issued by the Department of Environment and NaturalResources (DENR),

Community Environment and Natural Resources Office (CENRO),Cagayan de Oro City. Prior to the spouses Tan, the subject property was in thepossession of Lucio and Juanito Neri and their respective spouses. Lucio and Juanito Neri had declared thesubject property for taxation purposes in their names.The spouses Tan acquiredthe subject property from Lucio and Juanito Neri and their spouses by virtue ofa duly notarized Deed of Sale of Unregistered Real Estate Property dated 26June 1970. The spouses Tan tookimmediate possession of the subject property on which they planted rubber,gemelina, and other fruit-bearing trees. They declared the subject property for taxation purposes in their names andpaid realty taxes thereon.

However, a certain Patermateo Casiño (Casiño) claimed aportion of the subject property, prompting the spouses Tan to file a Complaintfor Quieting of Title against him before the RTC of Cagayan de Oro City. On 29August 1989, the RTC rendered a Decision favoring the spouses Tan and declaringtheir title to the subject property thus "quieted." Casiño appealed the said RTC Decision to theCourt of Appeals . In a Resolution the appellate court dismissed for lack ofinterest to prosecute. Casiño elevatedhis case to this Court via a Petition for Review on Certiorari. In a Resolutiondated 13 March 1991  the Court deniedCasiño's Petition for being insufficient in form and substance. The said Resolution became final andexecutory on 3 June 1991. Refusing to give up, Casiño filed an Application forFree Patent on the subject property before the Bureau of Lands. On 8 December1999, Casiño's application was ordered cancelled by Officer Ruth G. Sabijon ofDENR-CENRO, Cagayan de Oro City, upon the request of herein petitioner PedroTan, the declared owner of the subject property pursuant to the 29 August 1989Decision of the RTC. In 2000, the spouses Tan filed their Application forRegistration of Title to the subject property before the RTC of Cagayan de OroCity. The application of the spouses Tan invoked the provisions of Act No. 496 and/orSection 48 of Commonwealth Act No. 141,as amended. In compliance with the request of the LandRegistration Authority (LRA) dated 29 August 2000, the spouses Tan filed on 5October 2000 an Amended Application for Registration of Title to the subjectproperty.

The Office of the Solicitor General (OSG) entered itsappearance on behalf of the Republic, but failed to submit a written oppositionto the application of the spouses Tan.

When no opposition to the application of the spouses Tan wasfiled by the time of the initial hearing the RTC issued on 23 April 2001 anorder of general default, except as

against the Republic. Thereafter, the spouses Tan were allowed topresent their evidence ex-parte.

After the establishment of the jurisdictional facts, the RTCheard the testimony of John B. Acero , nephew and lone witness of the spousesTan. After Acero's testimony, the spouses Tan already made a formal offer ofevidence, which was admitted by the court a quo. On 9 May 2001, the RTCrendered a Decision granting the application of the spouses Tan. The Republicappealed the RTC Decision to the Court of Appeals.

On 28 February 2006, the Court of Appeals rendered aDecision granting the appeal of the Republic, and reversing and setting asidethe 9 May 2001 Decision of the RTC on the ground that the spouses Tan failed tocomply with Section 48(b) of Commonwealth Act No. 141, otherwise known as thePublic Land Act, as amended by Presidential Decree No. 1073, which requirespossession of the subject property to start on or prior to 12 June 1945. Hence,the appellate court ordered the spouses Tan to return the subject property tothe Republic.

The spouses Tan filed a Motion for Reconsideration of theforegoing Decision of the Court of Appeals. To refute the finding of the appellate court that they and theirpredecessors-in-interest did not possess the subject property by 12 June 1945or earlier, the spouses Tan attached to their Motion a copy of Tax DeclarationNo. 4627 covering the subject property issued in 1948 in the name of theirpredecessor-in-interest, Lucio Neri. They called attention to the statement in Tax Declaration No. 4627 thatit cancelled Tax Declaration No. 2948. Unfortunately, no copy of Tax Declaration No. 2948 was available even inthe Office of the Archive of the Province of Misamis Oriental. The spouses Tanasserted that judicial notice may be taken of the fact that land assessment isrevised by the government every four years; and since Tax Declaration No. 4627was issued in the year 1948, it can be presupposed that Tax Declaration No.2948 was issued in the year 1944.

The Court of Appeals denied the Motion for Reconsiderationof the spouses Tan in a Resolution dated 12 April 2007.

The earliestevidence of possession and occupation of the subject property can be tracedback to a tax declaration issued in the name of their predecessors-in-interestonly in 1952. However, the spouses Tanare now asking the kind indulgence

of this Court to take into account TaxDeclaration No. 4627 issued in 1948, which they had attached to their Motionfor Reconsideration before the Court of Appeals but which the appellate courtrefused to consider. Just as they hadargued before the Court of Appeals, the spouses Tan point out that TaxDeclaration No. 4627 was not newly issued but cancelled Tax Declaration No.2948; and should the Court take judicial notice of the fact that taxassessments are revised every four years, then Tax Declaration No. 2948covering the subject property was issued as early as 1944.

ISSUE

WHETHER TAXDECLARATIONS AND RECEIPTS ARE CONCLUSIVE EVIDENCE OF OWNERSHIP

RULING

Tax declarations and receipts are not conclusive evidence ofownership. At most, they constitute mere prima facie proofs of ownershipof the property for which taxes have been paid. In the absence of actual,public and adverse possession, the declaration of the land for tax purposesdoes not prove ownership.They may be good supporting or collaboratingevidence together with other acts of possession and ownership; but bythemselves, tax declarations are inadequate to establish possession of the propertyin the nature and for the period required by statute for acquiring imperfect orincomplete title to the land.

The spouses Tan purchased the subject property and came intopossession of the same only in 1970. To justify their application for registrationof title, they had to tack their possession of the subject property to that oftheir predecessors-in-interest. Whilethe spouses Tan undoubtedly possessed and occupied the subject property openly,continuously, exclusively and notoriously, by immediately introducingimprovements on the said property, in addition to declaring the same and payingrealty tax thereon; in contrast, there was a dearth of evidence that theirpredecessors-in-interest possessed and occupied the subject property in thesame manner. The possession andoccupation of the subject property by the predecessors-in-interest of thespouses Tan were evidenced only by the tax declarations in the names of theformer, the earliest of which, Tax Declaration No. 4627, having been issued onlyin 1948. No other evidence was presentedby the spouses Tan to show specific acts of ownership exercised by theirpredecessors-in-interest over the subject property which may date back to 12June 1945 or earlier.

For failure of the Spouses Tan to satisfy the requirementsprescribed by Section 48(b) of the Public Land Act, as amended, this Court hasno other option but to deny their application for judicial confirmation andregistration of their title to the subject property.

DECREE OF REGISTRATION

G.R. No.123346, December 14, 2007

MANOTOK REALTY, INC. AND MANOTOK ESTATE CORPORATION,PETITIONERS, VS. CLT REALTY DEVELOPMENT CORPORATION, RESPONDENT

FACTS

            On 10August 1992, CLT Realty Development Corporation (CLT) sought to recover fromManotok Realty, Inc. and Manotok Estate Corporation (Manotoks) the possessionof Lot 26 of the Maysilo Estate in an action filed before the Regional TrialCourt of Caloocan City. CLT’s claim was anchored on Transfer Certificate ofTitle (TCT) issued in its name by the Caloocan City Register of Deeds, whichtitle in turn was derived from Estelita Hipolito (Hipolito) by virtue of a Deedof Sale with Real Estate Mortgage dated 10 December 1988. Hipolito’s titleemanated from Jose Dimson’s (Dimson) TCT , a title issued pursuant to an orderof the Court of First Instance (CFI) of Caloocan City. Dimson’s title appearsto have been sourced from OCT No. 994.

For their part, the Manotoks challenged the validity of thetitle relied on by CLT, claiming that Dimson’s title, the proximate source ofCLT’s title, was irregularly issued and, hence, the same and subsequent titlesflowing therefrom are likewise void. The Manotoks asserted their ownership overLot 26 and claimed that they derived it from several awardees and/or vendees ofthe National Housing Authority. The Manotok title likewise traced as itsprimary source OCT No. 994 which, on 9 September 1918, was transferred toAlejandro Ruiz and Mariano Leuterio who had previously acquired the property on21 August 1918 by virtue of an “Escritura de Venta” executed by Don TomasArguelles and Don Enrique Llopis. On 3 March 1920, Ruiz and Leuterio sold theproperty to Francisco Gonzalez who held title thereto until 22 August 1938 whenthe property was transferred to Jose Leon Gonzalez, Consuelo Susana Gonzalez,Juana Francisca Gonzalez, Maria Clara Gonzalez, Francisco Felipe Gonzalez andConcepcion Maria Gonzalez under TCT No. 35486. The lot was then, per annotationdated 21 November 1946, subdivided into seven (7) parcels each in the name ofeach of the Gonzalezes.

The trial court, ruling for CLT, adopted the factualfindings and conclusions arrived at by the majority commissioners appointed toresolve the conflict of titles. It was established that the entire MaysiloEstate was registered under Act No. 496 by virtue of which OCT No. 994 wasissued by the Register of Deeds of Rizal;that Lot 26 was transferred to CLT byHipolito whose title was derived from the Dimson title and that on the basis ofthe technical descriptions of the property appearing in the Manotok titles, thelatter’s property indeed encroached on the property described in CLT’s title.

The Manotoks appealed to the Court of Appeals, whichaffirmed the decision of the trial court. Their motion for reconsiderationhaving been denied, they filed a petition for review with the Supreme Court,ascribing error to the appellate court in upholding the trial court’s decisionwhich decided the case on the basis of the majority commissioners’ report andoverlooked relevant facts in the minority commissioner’s report.

ISSUE

whether the titles issued in thename of CLT IS valid

RULING

With respect to G.R. No. 123346, the Court upheld thevalidity of the trial court’s adoption of the commissioners’ majority report aspart of the decision. The Court pointed out that the titles of respondents inall three cases were derived from OCT No. 994 of the Registry of Deeds ofCaloocan City registered on 19 April 1917. The Manotoks filed their respectivemotions for reconsideration. On 5 June 2006, the cases were elevated to theCourt en banc. In the Manotok petition, CLT hadoriginally filed a complaint for annulment of the titles in the name of theManotoks, alleging that it was the registered owner of Lot 26 of the MaysiloEstate. It is evident from all three titles CCLT’s, Hipolito’s andDimson’s—that the properties they purport to cover were “originally registeredon the 19th day April 1917 in the Registration Book of the Office of theRegister of Deeds of Rizal.” As earlier established, there is no such OCT No.994 originally registered on 19 April 1917. None of these three titles can beaccorded recognition simply because the original title commonly referred totherein never existed. To conclude otherwise would constitute deliberatedisregard of the truth. These titles could be affirmed only if it can be proventhat OCT No. 994 registered on 19 April 1917 had actually existed. CLT and theDimsons were given the opportunity to submit such proof before this Court, butthey did not. In fact, CLT has specifically manifested that the OCT No. 994they concede as true is also the one which the Office of Solicitor Generalsubmitted as true, and that is OCT No. 994 issued on 3 May 1917.Given thisessential clarification, there is no sense in affirming the 2005 Decision whichsustained the complaints for annulment of title and/or recovery of possessionfiled by CLT and the Dimson when their causes of action are both founded on aninexistent mother title.

From these premises, the Court is able to make the followingbinding conclusions. First, there is only one OCT No. 994. As it appears on therecord, that mother title was received for transcription by the Register ofDeeds on 3 May 1917, and that should be the date which should be reckoned asthe date of registration of the title. It may also be acknowledged, as appearson the title, that OCT No. 994 resulted from the issuance of the decree ofregistration on 17 April 1917, although such date cannot be considered as thedate of the title or the date when the title took effect. Second,any title thattraces its source to OCT No. 994 dated 17 April 1917 is void, for such mothertitle is inexistent. This error alone is, in fact, sufficient to invalidate theDimson and CLT claims over the subject property if singular reliance is placedby them on the dates appearing on their respective titles.

The land becomes a registered landonly upon the transcription of the decree in the original registration book bythe register of deeds, the date and time of such transcription being set forthin the process and certified to at the foot of each entry or certificate oftitle. The issuance of the original and owner’s duplicate certificates arebasic for the valid existence of the title. Issuance of additional copies arepermissive and their non-existence does not affect the status of title. Acertificate of title is deemed as regularly issued with the issuance of theoriginal copy and owner’s duplicate

�7�L�%

20(Douglas)

21 (Douglas)

22(Douglas)

23(Cha)

24(Cha)

25(Cha)

26(Cha)

27(Cha)

28 (Cha)

29(Cha)

A writ of demolition is but a compliment of the writ of possession and may be issued by a special order of the court.Gawaran v. IAC 162 SCRA 154Facts:Petitioners oppose the application for registration and confirmation of title over Lot 2, PSU-173975 situated in Digman, Bacoor, Cavite, on which petitioners had their residential house and a "camarin." The trial court awarded the lot to the petitioners. The respondents appealed to the CA which reversed the decision of the trial court and confirmed the ownership of said Lot No. 2 in the names of private respondents. On motion of private respondents, the respondent RTC, on March 19, 1985, issued the questioned writ of possession with the complimentary directive for the oppositors to dismantle and remove their building and/or structure from Lot No. 2 under pain of demolition and to vacate the premises in favor of private respondents within thirty (30) days. The petitioners appealed but the petition was dismissed, and an order for the issuance of a decree in favor of private respondents was issued.Issue: WON the court is correct in issuing the writ of possession with a special order of demolitionHeld:The SC held that the respondent appellate court committed no reversible error in holding that the writ of possession issued by the trial court and it is a necessary consequence of the adjudication of ownership and the corresponding issuance of the Original Certificate of Title. In a registration case, the judgment confirming the title of the applicant and ordering its registration in his name necessarily carries with it the delivery of possession which is an inherent element of the right of ownership. Hence, a writ of possession may be issued not only against the person who has been defeated in a registration case, but also against any one unlawfully and adversely occupying the land or any portion thereof during the registration proceedings up to the issuance of the final decree. It is the duty of the registration court to issue said writ when asked for by the successful party.

9/7, 1:51pmMush TapaIn lieu of demolition, award of damages proper.Ayala Corp. v. Ray Burton Development Corp.294 SCRA 48Facts:Petitioner Ayala Corporation (AYALA) is the owner of the Ayala estate located in Makati City. The said estate was originally a raw land which was subdivided for sale into different lots devoted for residential, commercial and industrial purposes. On March 20, 1984, Karamfil Import-Export Company Ltd. (KARAMFIL) bought from AYALA a piece of land identified as Lot 26, Block 2 consisting of 1,188 square meters, located at what is now known as H.V. de la Costa Street, Salcedo Village, Makati City. The said land is now the subject of this case. The transaction was documented in a Deed of Sale of even date, which provides, among others, that the vendee would comply with certain special conditions and restrictions on the use or occupancy of the land.On February 18, 1988, KARAMFIL sold the lot to Palmcrest Development and Realty Corporation (PALMCREST) under a Deed of Absolute Sale of even date. AYALA gave its written conformity to the sale but reflecting in its approval the same special conditions/restrictions as in the previous sale. PALMCREST in turn sold the lot to Ray Burton Development Corporation (RBDC), now respondent, on April 11, 1988, with the agreement that AYALA retains possession of the Owner’s Duplicate copy of the title until a building is erected on said parcel of land in accordance with the requirements and/or restrictions of AYALA. As in the KARAMFIL-PALMCREST transaction, AYALA gave its conformity to the sale, subject to RBDC’s compliance with the special conditions/restrictions which were annotated in the deed of sale.Sometime in June of 1989, RBDC submitted to AYALA for approval a set of architectural plans for the construction of a 5-storey office building on the subject lot. The building was to be known as “Trafalgar Tower” but later renamed “Trafalgar Plaza.” Since the building was well within the 42-meter height restriction, AYALA approved the architectural plans.Early in June of 1990, RBDC made another set of building plans for “Trafalgar Plaza” and submitted the same for approval, this time to the Building Official of the Makati City Engineer’s Office, not to AYALA. In these plans, the building was to be 26-storey high, or a height of 98.60 meters, with a total gross floor area of 28,600 square meters. After having obtained the necessary building permits from the City Engineer’s Office, RBDC began to construct “Trafalgar Plaza” in accordance with these new plans.Issue: WON award of damages is proper, in lieu of demolitionHeld:Yes.In sum, there is more than ample evidence on record pinpointing RBDC’s violation of the applicable FAR restrictions in the Consolidated and Revised Deed Restrictions (CRDRs) when it constructed the 27-storey Trafalgar Plaza. The prayer of petitioner is that judgment be rendered, among others, ordering Ray Burton to comply with its contractual obligations in the

construction of ‘Trafalgar Plaza’ by removing or demolishing the portions of areas thereof constructed beyond or in excess of the approved height, as shown by the building plans submitted to, and approved by, Ayala, including any other portion of the building constructed not in accordance with the said building plans.However, the record reveals that construction of Trafalgar Plaza began in 1990, and a certificate of completion thereof was issued by the Makati City Engineer’s Office per ocular inspection on November 7, 1996. Apparently Trafalgar Plaza has been fully built, and we assume, is now fully tenanted. The alternative prayers of petitioner under the CRDRs, i.e., the demolition of excessively built space or to permanently restrict the use thereof, are no longer feasible. Thus, we perforce instead rule that RBDC may only be held alternatively liable for substitute performance of its obligations – the payment of damages. Ray Burton Development Corporation acted in bad faith in constructing Trafalgar Plaza in excess of the applicable restrictions upon a double submission of plans and exercising deceit upon both AYALA and the Makati Engineer's Office, and thus by way of example and correction, should be held liable to pay AYALA exemplary damages in the sum of P2,500,000.00.

9/7, 1:52pmMush TapaJUDICIAL CONFIRMATION OF IMPERFECT TITLERepublic v. DiloyG.R. No. 174633, 26 August 2008, 563 SCRA 413Unless the land has been classified as alienable and disposable, the rules on the confirmation of imperfect title shall not apply.Facts:Before this Court is a Petition for Review on Certiorari under Rule 45 of the 1997 Revised Rules of Civil Procedure seeking to set aside the Decision and Resolution of the Court of Appeals. The CA Decision denied the appeal filed before it by the Republic of the Philippines (Republic) and affirmed the Decision of the 2nd MCTC of Silang-Amadeo, Silang, Cavite, granting the application for registration of title filed before it by the respondent Gregoria L. Diloy over a parcel of land located in Barangay Dagatan, Municipality of Amadeo, Province of Cavite, covering an area of 22,249 square meters. The Resolution denied the MR filed by the Republic.The Republic persistently argues that the respondent's Application for Registration of Title should have been denied because the latter failed to comply with the period of possession required by law, i.e., Section 14 of Presidential Decree No. 1529.30 The Republic reveals that the subject property was only declared alienable and disposable on 15 March 1982 per Forestry Administration Office (FAO) No. 4-1650. From 1982 when the property was declared alienable and disposable to 1997, the respondent had only been in adverse possession of the subject property for a period of 15 years. Thus, there was no compliance with Section 14, PD No. 1529 because the subject property was not yet alienable and disposable on 12 June 1945, and respondent's possession lacked the required number of years (30 years) for her to acquire the same

through prescription. Hence, respondent did not acquire an imperfect title, which may be confirmed through a judicial proceeding.Issue: WON the respondent has acquired a registrable titleHeld:The Petition is meritorious.While this Court agrees with the lower courts that, indeed, respondent's possession of the subject property was open, continuous, exclusive and notorious, however, we hold that respondent failed to prove that she or her predecessors-in-interest were already in possession of the subject property under a bona fide claim of ownership since 12 June 1945 or earlier, which is the reckoning period specifically provided in Section 14(1) of PD No. 1529.As can be gleaned from the records, respondent's possession of the subject property started only in the year 1979 when her mother executed a Deed of Absolute Sale over the same in her favor. There was also no showing that her predecessors-in-interest had already been in possession or had already exercised acts of ownership over the subject property since 12 June 1945 or prior thereto, as her predecessors-in-interest declared the subject property for taxation purposes only in the year 1948. What was clearly established by the respondent was possession of the subject property by her predecessors-in-interest beginning 1948, which was short of three years from 12 June 1945. What is more telling is that the subject property became alienable and disposable only on 15 March 1982. Prior to its declaration as alienable land in 1982, any occupation or possession thereof could not be considered in the counting of the 30-year possession requirement. The period of possession by the respondent of the subject property cannot be considered to have started in 1979, when the same was conveyed to her by her mother. Neither can her possession of the subject property be tacked to that of her predecessors-in-interest, even if they had occupied and were in possession of the same since 1948, because during those periods, the subject property had not yet been classified as alienable and disposable land capable of private appropriation. Possession of the subject property could only start to ripen into ownership on 15 March 1982, when the same became alienable and disposable. Any period of possession prior to the date when the subject lot was classified as alienable and disposable is inconsequential and should be excluded from the computation of the period of possession; such possession can never ripen into ownership and, unless the land has been classified as alienable and disposable, the rules on the confirmation of imperfect title shall not apply thereto. The adverse possession which may be the basis of a grant of title or confirmation of an imperfect title refers only to alienable or disposable portions of the public domain. There can be no imperfect title to be confirmed over lands not yet classified as disposable or alienable. In the absence of such classification, the land remains unclassified public land until released therefrom and open to disposition. Possession of the land by the respondent under the circumstances, whether spanning decades or centuries, can never ripen into ownership.

9/7, 1:52pmMush TapaTan v. RepublicG.R. No. 177797, 04 December 2008, 573 SCRA 89Facts:This case is a Petition for Review on Certiorari under Rule 45 of the 1997 Revised Rules of Civil Procedure seeking to reverse and set aside the Decision and Resolution of the CA. In its assailed Decision, the appellate court reversed and set aside the Decision of the RTC of Misamis Oriental, 10th Judicial Region, Branch 39, Cagayan de Oro City, in LRC Case No. N-2000-055, and ordered petitioners, spouses Pedro and Nena Tan (spouses Tan), to return the parcel of land known, with an area of 215,698 square meters, located in Calingagan, Villanueva, Misamis Oriental (subject property) to respondent, Republic of the Philippines (Republic). In its assailed Resolution, the appellate court denied the spouses Tan’s MR.Issue:WON [the Spouses Tan] have been in open, continuous, exclusive and notorious possession and occupation of the subject [property], under a bona fide claim of acquisition or ownership, since [12 June 1945], or earlier, immediately preceding the filing of the application for confirmation of titleHeld:The Court rules in the negative and, thus, finds the present Petition devoid of merit.Through the years, Section 48(b) of the Public Land Act has been amended several times. As amended, Section 48(b) now reads:(b) Those who by themselves or through their predecessors-in- interest have been in open, continuous, exclusive, and notorious possession and occupation of agricultural lands of the public domain, under a bona fide claim of acquisition or ownership, since June 12, 1945 or earlier, immediately preceding the filing of the application for confirmation of title except when prevented by wars or force majeure. Those shall be conclusively presumed to have performed all the conditions essential to a Government grant and shall be entitled to a certificate of title under the provisions of this chapter.Section 48(b) of the Public Land Act, as amended by PD No. 1073, presently requires, for judicial confirmation of an imperfect or incomplete title, the possession and occupation of the piece of land by the applicants, by themselves or through their predecessors-in-interest, since 12 June 1945 or earlier. This provision is in total conformity with Section 14(1) of the Property Registration Decree heretofore cited. As the law now stands, a mere showing of possession for thirty years or more is not sufficient. It must be shown, too, that possession and occupation had started on 12 June 1945 or earlier.For failure of the Spouses Tan to satisfy the requirements prescribed by Section 48(b) of the Public Land Act, as amended, this Court has no other option but to deny their application for judicial confirmation and registration of their title to the subject property. Much as this Court wants to conform to the State’s policy of encouraging and promoting the distribution of alienable

public lands to spur economic growth and remain true to the ideal of social justice, our hands are tied by the law’s stringent safeguards against registering imperfect titles.

9/7, 1:53pmMush TapaRepublic v. Credit Corp.G.R. No. 173088, 25 June 2008, 555 SCRA 315Facts:Respondent Imperial Credit Corporation is a corporation duly organized and existing under the laws of the Philippines. On 07 March 1966, respondent purchased from a certain Jose Tajon a parcel of land situated in Barrio Colaique (now Barangay San Roque), Antipolo City, Rizal for the sum of P17,986.00 as evidenced by a Deed of Sale with Mortgage. Upon full payment of the balance of P1,909.00 through judicial consignation, ownership of the property was consolidated in the name of respondent and the mortgage constituted thereon released in December 1997. The property was thereafter privately surveyed under PSU-178075 and approved on 25 January 2000. On 14 February 2000, respondent filed before the RTC of Antipolo City an application for registration of a parcel of land, as shown on Plan PSU-178075 containing an area of 8,993 square meters. The application was docketed and raffled off to Branch 74 of said RTC. The application alleged, among others, that respondent "subrogated former owner Jose Tajon, who has been in open, continuous, exclusive and notorious possession and occupation of the parcel of land, being a part of the alienable and disposable lands of the public domain, under a bona fide claim of ownership since 12 June 1945, by virtue of Deed of Sale with Mortgage executed on 07 March 1966, After respondent presented evidence establishing the jurisdiction facts, the RTC issued an order of general default against the whole world allowing respondent to present its evidence ex parte. At the hearing, Ricardo Santos, respondent’s legal researcher and duly authorized attorney-in-fact, testified on the fact of respondent’s actual possession through its caretaker, Teodisia Palapus, who had been overseeing said property since its acquisition from Jose Tajon. Palapus also corroborated Santos’ testimony and added that except for some trespassers, no one else had laid possessory claim on the property. Aside from the transfer documents, the other documentary evidence submitted consisted of a 1993 tax declaration, the tracing cloth plan, survey description, a certification from the Land Management Sector in lieu of the geodetic engineer’s certificate and the report by the Community Environment and Natural Resources Office that the property falls within the alienable and disposable zone. On 21 November 2002, the RTC rendered judgment granting respondent’s application for registration. Petitioner Republic of the Philippines, through the Office of the Solicitor General (OSG), seasonably appealed from the RTC’s Decision to the CA, contending that respondent failed to present incontrovertible evidence that respondent and its predecessor-in-interest have been in open continuous,

exclusive and notorious possession and occupation of the property since 12 June 1945 or earlier. The CA rendered a Decision on 02 June 2006, dismissing the appeal by the OSG. Issue: Although petitioner concedes that respondent was able to show that the land applied for has been declassified from the forest or timber zone and is an alienable public agricultural land, respondent’s evidence failed to satisfy the requirement under paragraph (1) of Section 14, P.D. No. 1529, that is, respondent’s possession and occupation of the property for the length of time and in the manner required by law. Held:The petition is meritorious.Under the Regalian doctrine, the State is the source of any asserted right to ownership of land. This is premised on the basic doctrine that all lands not otherwise appearing to be clearly within private ownership are presumed to belong to the State. Any applicant for confirmation of imperfect title bears the burden of proving that he is qualified to have the land titled in his name.The reckoning date under the Public Land Act for the acquisition of ownership of public lands is June 12, 1945 or earlier, and that evidence of possession from that date or earlier is essential for a grant of an application for judicial confirmation of imperfect title.While a tax declaration by itself is not sufficient to prove ownership, it may serve as sufficient basis for inferring possession.

9/7, 1:54pmMush TapaMalabanan v. RepublicG.R. No. 179987, 29 April 2009The SC en banc upheld the ruling of the CA denying the application of the petitioners for the registration of a parcel of land situated in Barangay Tibig, Silang, Cavite on the ground that they had not established by sufficient evidence their right to the registration in accordance with either Section 14(1) or Section 14(2) of PD No. 1529 (Property Registration Decree).Facts:On 20 February 1998, Mario Malabanan filed an application for land registration covering a parcel of land in Silang Cavite. Malabanan claimed that he had purchased the property from Eduardo Velazco, and that he and his predecessors-in-interest had been in open, notorious, and continuous adverse and peaceful possession of the land for more than thirty years. The application was raffled to the RTC Cavite-Tagaytay City. The OSG duly designated the Jose Velazco, Jr. to appear on behalf of the State. Apart from presenting documentary evidence, Malabanan himself and his witness, Aristedes Velazco, who testified that the property was originally belonged to a twenty-two hectare property owned by his great-grandfather, Lino Velazco.Issues:1. In order that an alienable and disposable land of the public domain may be registered under Section 14(1) of PD No. 1529, otherwise known as

the Property Registration Decree, should the land be classified as alienable and disposable as of June 12, 1945 or is it sufficient that such classification occur at any time prior to the filing of the applicant for registration provided that it is established that the applicant has been in open, continuous, exclusive and notorious possession of the land under a bona fide claim of ownership since June 12, 1945 or earlier?2. For purposes of Section 14(2) of the Property Registration Decree may a parcel of land classified as alienable and disposable be deemed private land and therefore susceptible to acquisition by prescription in accordance with the Civil Code?3. May a parcel of land established as agricultural in character either because of its use or because its slope is below that of forest lands be registrable under Section 14(2) of the Property Registration Decree in relation to the provisions of the Civil Code on acquisitive prescription?4. Are petitioners entitled to the registration of the subject land in their names under Section 14(1) or Section 14(2) of the Property Registration Decree or both?Held:The Petition is denied.(1) In connection with Section 14(1) of the Property

Registration Decree, Section 48(b) of the Public Land Act recognizes and confirms that “those who by themselves or through their predecessors in interest have been in open, continuous, exclusive, and notorious possession and occupation of alienable and disposable lands of the public domain, under a bona fide claim of acquisition of ownership, since June 12, 1945” have acquired ownership of, and registrable title to, such lands based on the length and quality of their possession.(a) Since Section 48(b) merely requires possession since 12 June 1945 and does not require that the lands should have been alienable and disposable during the entire period of possession, the possessor is entitled to secure judicial confirmation of his title thereto as soon as it is declared alienable and disposable, subject to the timeframe imposed by Section 47 of the Public Land Act.(b) The right to register granted under Section 48(b) of the Public Land Act is further confirmed by Section 14(1) of the Property Registration Decree.(2) In complying with Section 14(2) of the Property Registration Decree, consider that under the Civil Code, prescription is recognized as a mode of acquiring ownership of patrimonial property. However, public domain lands become only patrimonial property not only with a declaration that these are alienable or disposable. There must also be an express government manifestation that the property is already patrimonial or no longer retained for public service or the development of national wealth, under Article 422 of the Civil Code. And only when the property has become patrimonial can the prescriptive period for the acquisition of property of the public dominion begin to run.(a) Patrimonial property is private property of the government. The person acquires ownership of patrimonial property by prescription under the Civil Code is entitled to secure registration thereof under Section 14(2) of the Property Registration Decree.(b) There are two kinds of prescription by which patrimonial property may be acquired, one ordinary and other extraordinary. Under ordinary acquisitive prescription, a person acquires ownership of a patrimonial property through possession for at least ten (10) years, in good faith and with just title. Under extraordinary acquisitive prescription, a person’s uninterrupted adverse possession of patrimonial property

for at least thirty (30) years, regardless of good faith or just title, ripens into ownership.It is clear that the evidence of petitioners is insufficient to establish that Malabanan has acquired ownership over the subject property under Section 48(b) of the Public Land Act. There is no substantive evidence to establish that Malabanan or petitioners as his predecessors-in-interest have been in possession of the property since 12 June 1945 or earlier. The earliest that petitioners can date back their possession, according to their own evidence—the Tax Declarations they presented in particular—is to the year 1948. Thus, they cannot avail themselves of registration under Section 14(1) of the Property Registration Decree.Neither can petitioners properly invoke Section 14(2) as basis for registration. While the subject property was declared as alienable or disposable in 1982, there is no competent evidence that is no longer intended for public use service or for the development of the national evidence, conformably with Article 422 of the Civil Code. The classification of the subject property as alienable and disposable land of the public domain does not change its status as property of the public dominion under Article 420(2) of the Civil Code. Thus, it is insusceptible to acquisition by prescription.

VAGILIDAD v. VAGILIDAD G.R. No. 161136(Part 1)Facts:That on January 20, 1987, the heirs of ZOILO, Loreto Labiao (hereafter LORETO), Efren Labiao (hereafter EFREN) and Priscilla Espanueva (hereafter PRISCILLA) executed an Extrajudicial Settlement of Estate adjudicating the entire Lot No. 1253, covering 4,280 square meters, to LORETO. That On January 29, 1987, Transfer Certificate of Title (TCT) No. T-16693 was issued in favor of LORETO, EFREN and PRISCILLA, but on even date, TCT No. T-16693 was cancelled and TCT No. T-16694, covering the said property, was issued in the name of LORETO alone.That on September 21, 1988, Gavino Vagilidad Jr ( hereafter GABINO JR.] paid real estate taxes on the land he bought from LORETO as per Tax Declaration No. 1038 where the property was specified as Lot No. 1253-B. GABINO JR. thereafter sold the same lot to Wilfredo Vagilidad (hereafter WILFREDO) as per Deed of Absolute Sale dated December 7, 1989. On even date, Deed of Absolute Sale of a Portion of Land involving the opt-described property was also executed by LORETO in favor of WILFREDO. The aforementioned deeds, which were both executed on December 7, 1989 [and] notarized by Atty. Warloo Cardenal[,] [appear] to have been given the same entry number in his notarial books as both contained the designation Document No. 236, Page No. 49, Book No. XI, Series of 1989[.] That on February 14, 1990, the sale of Lot No. 1253-B to WILFREDO was registered with the Registry of Deeds of the Province of Antique under Entry No. 180425. Consequently, TCT No. T-18023, cancelling TCT No. 16694, was issued in favor of WILFREDO pursuant to the Deed of Absolute Sale dated December 7, 1989.That on September 29, 1995, spouses GABINO and Ma. Dorothy Vagilidad (hereafter DOROTHY), as plaintiffs, filed a Complaint for Annulment of Document, Reconveyance and Damages, with the Regional

Trial Court of Antique, Sixth Judicial Region, Branch 11, against spouses WILFREDO and Lolita Vagilidad (hereafter LOLITA), docketed as Civil Case No. 2825. The plaintiffs claimed that they are the lawful owners of Lot No. 1253-B which was sold to him by LORETO in 1986. They alleged that [GABINO JR.] is a nephew of defendant WILFREDO. They likewise raised that when GABINO SR. died, defendant WILFREDO requested GABINO JR. to transfer the ownership of Lot No. 1253-B in defendant WILFREDOs name for loaning purposes with the agreement that the land will be returned when the plaintiffs need the same. They added that, pursuant to the mentioned agreement, plaintiff GABINO JR., without the knowledge and consent of his spouse, DOROTHY, executed the Deed of Sale dated December 7, 1989 in favor of defendant WILFREDO receiving nothing as payment therefor. Issue:Whether the Deed of Sale executed in favor of the petitioners ( spouses Gabino Jr and Dorothy Vagilidad) validly convey Lot No. 1253-B to them.Held:the evidence preponderates in favor of the petitioners ( spouses Gabino Jr and Dorothy Vagilidad). The Deed of Sale executed by LORETO did validly convey Lot No. 1253-B to GABINO, JR.First, the Deed of Absolute Sale of Portion of Land dated December 7, 1989 between LORETO and WILFREDO is tainted with blatant irregularities. Second, the testimony of a disinterested witness, Febe Mabuhay, established the irregularity. Mabuhay used to work as secretary for Atty. Cardenal and co-signed as witness in both Deeds. Third, the testimony of Atty. Ernesto Estoya, then Clerk of Court of the Regional Trial Court of Antique, supports the claim that there was bad faith in the execution of the Deed of Absolute Sale of Portion of Land. Fourth, we give credence to the testimony of GABINO, JR. that LORETO and WILFREDO had employed the scheme to deprive him and his wife of their lawful title to the subject property. With these corroborating circumstances and the following irrefragable documents on record, the evidence preponderates in favor of GABINO, JR. One, he acquired Lot No.1253-B from LORETO on May 12, 1986[20] by virtue of the Deed of Absolute Sale. Two, the Bureau of Internal Revenue issued a Certification, also on May 12, 1986, for the exemption from the payment of capital gains tax when LORETO sold to him the subject parcel. Three, GABINO, JR. paid the real estate tax on the subject parcel in 1987. Four, he filed a Petition for the Surrender of LORETOs title on July 31, 1987 so he could transfer the title of the property in his name.

9/7, 9:08pmMelodia Lawangen DagasenRepublic v. Marcos No. L-32941 July 31, 1973, 52 SCRA 238(Part 3)Issue:Whether or a not the respondent judge has the power to reopen Civil Registration Case No. 1 of the Court of Fisrt Instance of Baguio establishing the Baguio Townsite Reservation in favor of the registration of the private respondents.Facts:That the respondent judge at the instance of private respondents, did reopen

Civil Registrartion Case No. 1 of the Court of Fisrt Instance of Baguio establishing the Baguio Townsite Reservation, promulgated as far back as November 13, 1922, thus enabling private respondents the registration of an area of 74,017 square meters inside the Camp John Hay Leave and Recreation Center.That on November 9, 1968, the respondent judge ordered in favor of private respondent for the registration of the aforesaid area.That on August 22, 1969, the Solicitor-General filed a motion to annul the decision based on the lack of jUrisdiction over the subject matter of the proceedings as the land in question is a part of a duly established military reservation.That on December 8, 1969, the respondent judge denied the motion. His decision was impressed by the claim that the private respondents had been in the possession " since the Spanish regime," and thus within the protection on the words annotated on all survey plans of Camp John Hay, to wit: " subject to prior and existing private rights."Held.The respondent judge was without power to reopen Civil Registration Case No. 1. RA No. 931 is quiet explicit, only persons ' claiming title to parcels of lands that have been the subject of cadastral proceedings' are granted the right to petition for the re opening thereof if the other conditions named therein are successfully met. The lots now disputed was reserved for naval purposes dated back as far as October 10, 1910 through an executive order issued by then President William Taft and thus could not have been the object of the cad astral proceeding, which was decided on November 13, 1922.What is more, it is undeniable that the land in question, being a part of a duly established military camp or reservation, cannot be thus ordered registered in favor of private respondents.Wherefore, the writ of certiorari is granted annulling and settling aside the decision of the respondent judge.

9/7, 9:19pmMelodia Lawangen Dagasen(Part5)Sec. Of DENR vs. Yap, GR No. 167707, 8 October 2008, 568 SCRA 164Facts: That on November 10, 1978, then President Ferdinand Marcos issued Proclamation No. 1801 declaring Boracay Island, among other islands, caves and peninsulas in the Philippines, as tourist zones and marine reserves under the administration of the Philippine Tourism Authority (PTA). President Marcos later approved the issuance of PTA Circular 3-82 dated September 3, 1982, to implement Proclamation No. 1801. That Claiming that Proclamation No. 1801 and PTA Circular No 3-82 precluded them from filing an application for judicial confirmation of imperfect title or survey of land for titling purposes, respondents-claimants Mayor Jose S. Yap, Jr., Libertad Talapian, Mila Y. Sumndad, and Aniceto Yap filed a petition for declaratory relief with the RTC in Kalibo, Aklan. That in their petition, respondents-claimants alleged that Proclamation No.

1801 and PTA Circular No. 3-82 raised doubts on their right to secure titles over their occupied lands. They declared that they themselves, or through their predecessors-in-interest, had been in open, continuous, exclusive, and notorious possession and occupation in Boracay since June 12, 1945, or earlier since time immemorial. They declared their lands for tax purposes and paid realty taxes on them. That Respondents-claimants posited that Proclamation No. 1801 and its implementing Circular did not place Boracay beyond the commerce of man. Since the Island was classified as a tourist zone, it was susceptible of private ownership. Under Section 48(b) of Commonwealth Act (CA) No. 141, otherwise known as the Public Land Act, they had the right to have the lots registered in their names through judicial confirmation of imperfect titles. That the Republic, through the Office of the Solicitor General (OSG), opposed the petition for declaratory relief. The OSG countered that Boracay Island was an unclassified land of the public domain. It formed part of the mass of lands classified as public forest, which was not available for disposition pursuant to Section 3(a) of Presidential Decree (PD) No. 705 or the Revised Forestry Code, as amended. That the OSG maintained that respondents-claimants reliance on PD No. 1801 and PTA Circular No. 3-82 was misplaced. Their right to judicial confirmation of title was governed by CA No. 141 and PD No. 705. Since Boracay Island had not been classified as alienable and disposable, whatever possession they had cannot ripen into ownership. April 14, 1976, the Department of Environment and Natural Resources (DENR) approved the National Reservation Survey of BoracayIsland, which identified several lots as being occupied or claimed by named persons.Issue: whether Proclamation No. 1801 and PTA Circular No. 3-82 can be deemed the positive act needed to classify Boracay Island as alienable and disposable land Held: Private claimants cannot rely on Proclamation No. 1801 as basis for judicial confirmation of imperfect title. The proclamation did not convert Boracay into an agricultural land. There is nothing in the law or the Circular which made Boracay Island an agricultural land. The reference in Circular No. 3-82 to private lands and areas declared as alienable and disposable does not by itself classify the entire island as agricultural. Clearly, the reference in the Circular to both private and public lands merely recognizes that the island can be classified by the Executive department pursuant to its powers under CA No. 141.Therefore, Proclamation No. 1801 cannot be deemed the positive act needed to classify Boracay Island as alienable and disposable land. If President

Marcos intended to classify the island as alienable and disposable or forest, or both, he would have identified the specific limits of each, as President Arroyo did in Proclamation No. 1064. This was not done in Proclamation No. 1801.

The Whereas clauses of Proclamation No. 1801 also explain the rationale behind the declaration of Boracay Island, together with other islands, caves and peninsulas in the Philippines, as a tourist zone and marine reserve to be administered by the PTA to ensure the concentrated efforts of the public and private sectors in the development of the areas tourism potential with due regard for ecological balance in the marine environment. Simply put, the proclamation is aimed at administering the islands for tourism and ecological purposes. It does not address the areas alienability.

5) ANGELITA F. BUENAVENTURA and PRECIOSA F. BUENAVENTURA, Petitioners,vs.REPUBLIC OF THE PHILIPPINES, Respondent.G.R. No. 166865 March 2, 2007

Facts:

Petitioners Angelita and Preciosa filed an Application for registration of Title on June 5, 2000 before the RTC of Paranaque City for a particular Lot situated in San Dionisio, Paranaque. Said petitioners are heirs of Spouses Amado Buenaventura and Irene Flores from whom they acquired the same. The property was acquired by the Spouses Buenaventura from the Heirs of Lazaro De Leon even before World War II. However, it was only on January 30 1948 that the deed of sale was executed in favor of Spouses Buenaventura. After the Execution of the said Deed of Sale, the Spouses Buenaventura transferred the tax declaration in their name. In 1978, the Spouses Buenaventura transferred by way of Deed of Sale, the subject property to their children, among who are the petitioners in the instant case.

Petitioners contend that "they and their predecessors-in-interest acquired title to the said parcel of land thru inheritance, transfer and possession as owners of the same since time immemorial and/or within the period provided for by law". To buttress their claim, the petitioners presented 5 witnesses. They likewise presented and identified several documents to establish further the following: 1.) petitioners' fee simple title over the property; 2.) the nature of the possession and occupation of the property; 3.) its classification as part of the alienable zone of the government; and 4.) the improvements introduced thereon and the taxes paid on the subject property.

The trial Court, on 29, October 2001 rendered a decision granting the application for registration of title.

The Republic appealed the case with the Court of Appeals. Said court reversed the

trial court’s decision and explained that the petitioners failed to show possession and occupation of the subject property under a bona fide claim of ownership since June 12, 1945 or earlier as provided for in Section 14(1) of the Property Registration Decree. It was stated in the decision of the Court of Appeals that the petitioners and their predecessors-in-interest only had possession of the subject property in 1948.

ISSUE: Can the petitioners validly register the land under their names even though their possession commenced only after June 12, 1945?

Ruling:

Yes. Even if the possession of alienable lands of the public domain commenced only after June 12, 1945, application for registration of said property is still possible by virtue of Section 14(2) of the Property Registration Decree which speaks of prescription. Under the Civil Code, Prescription is one of the modes of acquiring ownership. Article 1137 of the Civil Code states that “Ownership and other real rights over immovables also prescribe through uninterrupted adverse possession thereof for thirty years, without need of title or of good faith".

In the present case, while petitioners' possession over the subject property can be reckoned only on January 3, 1968, the date when according to evidence, the subject property became alienable, they can still have the said property in their names by virtue of Section 14 (2) of the Property Registration Decree.

The records reveal that they were in possession of the property for 32 years, reckoned from the year 1968, when the subject property was finally declared alienable by the DENR to the time they filed an application for registration of title on June 5, 2000. Petitioner's possession of the subject property since 1968 has been characterized as open, continuous, exclusive and notorious possession and occupation in the concept of an owner. Furthermore, the voluntary declaration of a piece of property for taxation purposes and the presentation of their realty tax payments of said property are good indicia of possession in concept of an owner, for no one in his right mind would be paying taxes for a property that is not in his actual or constructive possession. Such an act strengthens one's bona fide claim of acquisition of ownership.

Case No. 24: Ching v. Malaya, supra

Facts: This is a petition for certiorari under Rule 65 of the Rules of Court. The petitioners alleged in their complaint for ejectment that the private respondents had forced their way into the disputed premises and refused to vacate the same despite repeated demands. The petitioners claim that they own the property by a virtue of valid sale while the respondents claim that the property belonged to them by right

of inheritance. The municipal court rendered judgment ordering the private respondents to vacate the disputed property. On appeal, this decision was set aside by the respondent judge claiming that the municipal court had no competence to resolve the case.

Issue: Whether the municipal court had the jurisdiction over the case.

Held: Yes. It is settled that the mere assertion of ownership by the defendant in an ejectment case will not oust the municipal court of its summary jurisdiction. "The mere circumstance that proof of title, or evidence of ownership, had been introduced during the trial before the Municipal Court would not deprive said court of jurisdiction to rule on the question of who had the prior physical possession." There is one exception, however, and that is where it appears during the trial that, by the nature of the evidence presented, the issue of possession cannot be decided without deciding the issue of ownership. In such a case, the jurisdiction of the municipal court is lost and the action should be dismissed. After examining the facts of this present case, the Court finds that it does not come under the exception to the rule. Finally, the fact that the petitioners themselves adduced evidence of ownership over the property in question did not, as claimed, have the effect of divesting the municipal court of its jurisdiction.

Acosta vs. Salazar

Facts:

On November 10, 1985, respondents Trinidad and Aniceta Salazar filed a petition for the cancellation of the entries of the late spouses, Juan Soriano and Vicenta Macaraeg, annotated at the back of the Original Title of Certificate (OCT) No. 40287 on the ground that these entries have no consolidation rights in the Registry of Deeds (RD) of Tarlac and that the Transfer Certificate Title (TCT) No. 9297 which supposedly cancelled the said OCT is non-existent. The RTC Branch 63 of Tarlac granted their petition and ordered the RD to recall all titles, to cancel all tax declarations of the said entries, and to direct the owners of the said property to appear before the court for objections. The RD complied with the RTC’s orders and transferred the entries of the late spouses under the Salazars’ names.

With this, the petitioners, together with the 27 titleholders of the disputed property filed a complaint contending that they had acquired their titles in good faith and that the court committed an abuse of discretion acting as a land registration court because it had no jurisdiction over issues of ownership. The Salazars then filed for quieting of title with the RTC Branch 64 of Tarlac but was denied and ruled in favour of the herein petitioners contending that the RTC Branch 63 had no jurisdiction over the matter and that the TCT registered under their names was null and void.

Unsatisfied, the Salazars then appealed to the Court of Appeals which ruled in their favour contending that the RTC Branch 64 of Tarlac had no authority to dismiss RTC

Branch 63 of Tarlac’s decision because it is a court of equal rank. Moreover, CA also struck down the arguments raised by the titleholders and contended that the proceeding was a land registration proceeding, which is an action in rem, and requires no personal notice to the owners or claimants of the land in order to vest the court with jurisdiction.

Issue:

Whether the Salazars’ filing of action with the RTC Branch 63 of Tarlac is valid

Held:

No. While it is true that a proceeding regarding a registration of land is a proceeding in rem, requiring no personal notice to the owners to acquire jurisdiction over the case, the proceedings instituted by the Salazars with the RTC Branches 63 and 64 of Tarlac can hardly be considered as actions in rem. This is because the petition for cancellation of entries should have been directed against specific persons: namely, the heirs of Juan Soriano and the successors-in-interest who have acquired different portions of the property. Since no indispensable parties were included in the petition and were not impleaded by the Salazars, the said petitions with the RTC are not quasi in rem. Moreover, the absence of indispensable parties over the issue should have been a ground for the RTC to dismiss the case.

Furthermore, the property in dispute is under the protection of the Torrens system and the Salazars don’t have the right to impugn its legality. Rarely will the court allow another person to attack the validity and indefeasibility of a Torrens certificate, unless there is compelling reason to do so and only upon a direct action filed in court proceeded in accordance with law.

With this, the Court of Appeal’s decision is set aside and the the RTC Branch 64 of Tarlac’s decision is reinstated.

G.R. No. L-21024 July 28, 1969CENON MATEO, petitioner-appellant, vs.HON. FLORENCIO MORENO, in his capacity as SECRETARY OF PUBLIC WORKS AND COMMUNICATIONS, defendant-appellee.

FACTS: Sometime in 1959 a number of residents of Guiguinto, Bulacan, sent a letter-complaint to the Highway District Engineer of that province asking that the Sapang Cabay, a public navigable stream, which had been blocked by means of dikes and dams and converted into fishponds, be ordered reopened and restored to its original condition. The letter was referred to the Secretary of Public Works and Communications, who caused an investigation to be conducted pursuant to Republic Act No. 2056. Acting on the report which the investigator submitted to him, the Secretary rendered his decision on August 10, 1959, finding that the Sapang Cabay

was a public navigable stream and ordering Cenon Mateo, the herein petitioner-appellant, who had in the meantime acquired the property inside which the said creek is situated, to remove the dikes and dams therein constructed within thirty days from notice; otherwise they would be removed at his expense. Mateo moved to reconsider but was turned down, whereupon he filed the basic petition to restrain the respondent Secretary from enforcing his decision. The petition, as already stated, was dismissed by the Court a quo. The certification of the appeal to us was upon motion of both parties in view of the constitutional question involved.

ISSUE: whether or not Republic Act No. 2056 is unconstitutional because it unduly delegates judicial power to the Secretary and unlawfully deprives the appellant and others similarly situated of their property without due process of law

HELD: The constitutionality of the aforesaid statute has been upheld by this Court in Lovina vs. Moreno, G.R. No. L-17821, November 29, 1963, shortly before the present appeal was submitted for decision. That case held, furthermore, that the absence of any mention of a navigable stream within a property covered by a certificate of title does not preclude a subsequent investigation and determination of its existence nor make it private property of the title holder; that the findings of fact made by the Secretary of Public Works and Communications should be respected in the absence of illegality, error of law, fraud or imposition, as long as such findings are supported by substantial evidence; and that the ownership of a navigable stream or of the bed thereof is not subject to acquisitive prescription.

Rubie:LEDESMA VS. VILLASEÑOR 13 SCRA 494

FACTS:Felix Villaseñor is the special administrator of the estate of his deceased father, Eusebio Villaseñor. He filed a petition with the RTC (CFI) of Negros Occidental to prohibit the Register of Deeds of the same province to register a Deed of Sale of two lots made by his father in favor of Jose Ledesma. This was because said Deed of Sale was allegedly fictitious and the father’s signature was forged. The court then issued a writ of preliminary injunction. Thereafter, the RTC eventually dismissed the petition and lifted the writ on the ground that Ledesma had not been impleaded as a party-defendant and he only intervened in the case.Two days later, Ledesma filed his own petition asking that the Register of Deeds be ordered to register the aforementioned Deed of Sale on the ground that the earlier case was already dismissed and the writ was dissolved. On the same day, the court granted the petition without notice to the Register of Deeds and to Villaseñor and issued the order for registration.The Register of Deeds hence cancelled the certificates of title and issued new ones in Ledesma’s name. Villaseñor moved for reconsideration but was denied. Hence, present action.

ISSUE: Whether the RTC erred in issuing the order for registration.

RULING: Yes.We are of the opinion that the lower court did commit the error attributed to it.The court had no authority to issue the orders after just 2 days after lifting the injunction and dismissing the civil case without notice to the Register of Deeds or appellant considering that it was not yet final.The least that the court could have done was to afford appellant proper notice and hearing as it knew of the pendency of that case and that the relief sought therein was precisely to prevent registration.It is one thing for the Register of Deeds, in the exercise of his ministerial duties under the law, to register an instrument which in his opinion is registrable, and quite another thing for the court itself to order the registration. The former does not contemplate notice to and hearing of interested parties such as are required in a judicial proceeding nor carry with it the solemnity and legal consequences of a court judgment. The court a quo, in anticipating the action of the Register of Deeds, unnecessarily took the matter out of his hands and at the same time preempted the question of registration still pending in the civil action filed by appellant.The orders appealed from are hereby set aside, with costs against appellee.

27.G.R. No. 164687 February 12, 2009SM PRIME HOLDINGS, INC., Petitioner, vs.ANGELA V. MADAYAG, Respondent.Facts: Petitioner sought the Department of Environment and Natural Resources for the suspension of the land registration proceedings filed by Madayag over seven parcels of land in Urdaneta, Pangasinan. During the pendency of the petition, SM Prime Holdings filed an Urgent Motion to Suspend Proceedings, stating that the administrative case is prejudicial to the land registration case. The RTC concurred with petitioner’s claims, citing that since the survey plan is a mandatory requirement in land registration proceedings, cancellation thereof would be prejudicial to the petition for land registration. The Court of Appeals upheld the lower court’s ruling. Issue: Whether the appellate court acted in grave abuse of discretion in ordering the suspension of the proceedings. Ruling: Yes, there was grave abuse of discretion. The power to stay proceedings is an incident to the power inherent in every court to control the disposition of the cases in its dockets, with economy of time and effort for the court, counsel and litigants. Hence, every order suspending proceedings must be guided by the following precepts: it shall be done in order to avoid multiplicity of suits and prevent vexatious litigations, conflicting judgments, confusion between litigants and courts,

or when the rights of parties to the second action cannot be properly determined until the questions raised in the first action are settled.Furthermore, P.D. No. 1529 eliminated the distinction between general jurisdiction vested in the RTC and the latter’s limited jurisdiction when acting merely as a land registration court. Hence, the court now has the power to hear and decide controversial cases which involve substantial issues, such as the case at bar.

Case 7

B. Effect of the Regalian Doctrine: All lands of the public domain belong to the State which is the source of any asserted right to an asserted ownership of land. Property of the public domain is beyond the commerce of man and not susceptible of the private appropriation and acquisitive prescription.

REPUBLIC VS HEIRS OF LACHICA-SINGR. NO. 157485

Facts:

On August 26, 1991, respondent heirs instituted in the RTC of Kalibo, Aklan a complaint against Lucio Arquisola (Superintendent of ANCF) for recovery of possession, quieting of title, and declaration of ownership with damages. Respondent heirs claim that a 41,231-square meter-portion of the property they inherited had been usurped by ANCF, creating a cloud of doubt with respect to their ownership over the parcel of land they wish to remove from the ANCF reservation.The ANCF Superintendent countered that the parcel of land being claimed by respondents was the subject of Proclamation No. 2074 of then President Ferdinand E. Marcos allocating 24.0551 hectares of land within the area, which included said portion of private respondents’ alleged property, as civil reservation for educational purposes of ANCF. The ANCF Superintendent averred that the subject parcel of land is timberland and therefore not susceptible of private ownership.The RTC remanded the case to the MCTC of New Washington and Batan, Aklan, in view of the enactment of Republic Act No. 7659 which expanded the jurisdiction of first-level courts.On June 19, 2000, the MCTC rendered its Decision in favor of respondents. The MCTC ruled that the claim of respondent heirs over the disputed land by virtue of their and their predecessors’ open, continuous, exclusive and notorious possession amounts to an imperfect title, which should be respected and protected.Petitioner, through the Solicitor General, appealed to the RTC of Kalibo, Aklan and the RTC rendered its Decision affirming the MCTC Judgment. Petitioner Republic elevated the case to the Court of Appeals through a Petition for Review and the Court of Appeals rendered its Decision dismissing the petition for lack of merit.

Issue:Whether or not the CA gravely erred on a question of law in upholding respondents’ claim to supposed “private rights” over subject land despite the DENR certification

that it is classified as timberland.

Ruling:The private right referred to is an alleged imperfect title, which respondents supposedly acquired by possession of the subject property, through their predecessors-in-interest, for 30 years before it was declared as a timberland on December 22, 1960. It must be noted that respondents have not filed an application for judicial confirmation of imperfect title under the Public Land Act or the Property Registration Decree.The Court held that there are two requisites for judicial confirmation of imperfect or incomplete title under CA No. 141, namely: (1) open, continuous, exclusive, and notorious possession and occupation of the subject land by himself or through his predecessors-in-interest under a bona fide claim of ownership since time immemorial or from June 12, 1945; and (2) the classification of the land as alienable and disposable land of the public domain. With respect to the second requisite, the courts a quo held that the disputed property was alienable and disposable before 1960, citing petitioner’s failure to show competent evidence that the subject land was declared a timberland before its formal classification as such on said year. In Heirs ofMalabanan vs Republic, the members of this Court were in disagreement as to whether lands declared alienable or disposable after June 12, 1945 may be subject to judicial confirmation of imperfect title. There was, however, no disagreement that there must be a declaration to that effect. In the case at bar, it is therefore the respondents which have the burden to identify a positive act of the government, such as an official proclamation, declassifying inalienable public land into disposable land for agricultural or other purposes. Since respondents failed to do so, the alleged possession by them and by their predecessors-in-interest is inconsequential and could never ripen into ownership. Accordingly, respondents cannot be considered to have private rights within the purview of Proclamation No. 2074 as to prevent the application of said proclamation to the subject property. The SC grant the prayer of petitioner Republic to dismiss the civil case for lack of merit.

G. R. No. 156888 November 20, 2006PEDRO R. SANTIAGO, Petitioner, vs.SUBIC BAY METROPOLITAN AUTHORITY, Respondent.

FACTS: This case stemmed from a Complaint for Recovery of Possession of Property, filed by Victoria M. Rodriguez, Armando G. Mateo and herein petitioner Pedro R. Santiago against respondent Subic Bay Metropolitan Authority

Victoria M. Rodriguez, Armando G. Mateo and petitioner Pedro R. Santiago, alleged that: Victoria M. Rodriguez is the sole heir and administrator of the estate of Hermogenes Rodriguez who, in his lifetime, was the owner of parcels of land registered in his name under a Spanish title. Victoria leased two parcels of land to

Santiago and Mateo for a period of 50 years. By virtue of the lease, Santiago is presently occupying the land. SBMA, on the other hand, is claiming possessory, if not proprietary, rights over the parcels of land, by using them for its own commercial and other purposes. It is now the desire of plaintiff Victoria Rodriguez to recover possession of the property from the defendant so that she could comply with her contractual commitments to her co-plaintiffs.

RTC: Plaintiffs’ complaint is anchored on a Spanish title which they claim is still a valid, subsisting and enforceable title. Despite the fact that said title was never registered under Act 496, the land Registration Act (later PD 1529), plaintiffs still claim that they have a cause of action. The court is not convinced.

Petitioner Santiago’s immediate resort is by way of a petition for review on certiorari under Rule 45 of the Rules of Court.

ISSUE:Whether or not Spanish Titles are still admissible as evidence of ownership of lands

RULING:No. Although PD 892 reads: “Whereas, Spanish titles to lands which have not yet been brought under the operation of the Torrens system, being subject to Prescription, are now ineffective to prove ownership unless accompanied by proof of actual possession…,” petitioners cannot claim that they can still present the Spanish title as proof of ownership since they were in actual possession. Actual proof of possession only becomes necessary because Spanish titles are subject to prescription. The holder of a Spanish title may still lose his ownership of the real property to the occupant who actually possesses the same for the required prescriptive period. Because of this inherent weakness, the applicant for registration of his Spanish title under the Torrens system must also submit proof that he is in actual possession of the real property by virtue of prescription. Taking the law as a whole, it has clearly set a deadline for the filing of applications for registration of ALL Spanish titles under the Torrens system (i.e., 6 months from its affectivity or on 14 August 1976), after which, the Spanish titles may no longer be presented to prove ownership. However, if such land registration proceeding was filed and initiated after 14 August 1976, the applicant could no longer present his Spanish title to the court to evidence his ownership of the real property, regardless of whether the real property was in his actual possession.

Therefore, the fact that petitioners were in actual possession of the property when they filed the complaint with the RTC on April 29, 1996 does not exclude them from the application of PD 892, and their Spanish title remain inadmissible as evidence of their ownership of the property, whether in a land registration proceeding or in an action to remove a cloud on or to quiet title. However, this does not bar holders of Spanish titles from claiming ownership of real property on some other basis, such as those provided in PD 1529 or in the Public Land Act. Petitioners though failed to allege any other basis for their titles in their Complaint aside from possession of the

Subject Property from time immemorial, which this Court has already controverted; and the Spanish title, which is already ineffective to prove ownership over the Subject Property.

For sure, Spanish titles can no longer be countenanced as indubitable evidence of land ownership. And, without legal or equitable title to the subject property, Victoria M. Rodriguez, Armando G. Mateo and petitioner Pedro R. Santiago lacked the personality to claim entitlement to possession of the same. Title to real property refers to that upon which ownership is based. It is the evidence of the right of the owner or the extent of his interest, by which means he can maintain control and, as a rule, assert right to exclusive possession and enjoyment of the property.Therefore, the RTC correctly dismissed the complaint.

5) SECRETARY OF DENR VS. YAP G.R. NO. 167707

Facts:

The DENR identified several lots in Boracay as being claimed by named persons. President Ferdinand Marcos issued Proclamation No. 1801 declaring Boracay Island, among other islands, caves and peninsulas as tourist zones and marine reserves under administration of the Philippine Tourism Authority. Respondents-claimants alleged that it raised doubts on their right to secure titles over their occupied lands. Respondents-claimants posited that Proclamation No. 1801 and its implementing Circular did not place Boracay beyond the commerce of man. Since the Island was classified as a tourist zone, it was susceptible of private ownership. The Office of the Solicitor General countered that Boracay Island was unclassified land of the public domain which formed part of the “public forest”. Issue: Whether or not the Proclamation No. 1801 has posed any legal impediment to the titling of lands in Boracay Islands.

Ruling: The RTC and CA ruled that the Proclamation No. 1801 was not a legal impediment in the titling of lands in Boracay Islands. The CA held that respondents-claimants could not be prejudiced by a declaration that the lands they occupied since time immemorial were part of a forest reserve.

Proclamation No. 1801 cannot be deemed the positive act needed to classify Boracay Island as alienable and disposable land. If President Marcos intended to classify the island as alienable and disposable or forest, or both, he would have identified the specific limits of each, as President Arroyo did in Proclamation No. 1064. This was not done in Proclamation No. 1801.

The Whereas clauses of Proclamation No. 1801 also explain the rationale behind the declaration of Boracay Island, together with other islands, caves and peninsulas in the Philippines, as a tourist zone and marine reserve to be administered by the PTA

– to ensure the concentrated efforts of the public and private sectors in the development of the areas’ tourism potential with due regard for ecological balance in the marine environment. Simply put, the proclamation is aimed at administering the islands for tourism and ecological purposes. It does not address the areas’ alienability. Thus the islands remain as public domain which is governed by the Regalian doctrine.

CASE NO. 13: Republic of the Philippines vs. Court of Appeals83 SCRA 453

FACTS:Prudencio Tesalona died in 1905. He was survived by his two children Maria and Lucila. The two heirs, without executing an extrajudicial settlement of Prudencio's estate and adjudicating the said 29-hectare land to themselves, executed an " absolute sale" of the land in favor of Tarciana Morales-Maxin, the wife of applicant Prudencio Maxino who was Maria's son and the grandson of Prudencio Tesalona. In Land Registration Case No. 81-G of the Court of First Instance at Gumaca, Quezon, Judge Vicente del Rosario on March 21, 1961 rendered a decision, ordering the registration of said land, Lot 1, allegedly located at Barrio Cambuga (Anonang), Mulanay, in the names of the spouses Prudencio Maxino and Tarciana Morales. The decision became final and executory. A decree and an original certificate of title were issued.More than eight years later, or on June 20, 1969, the Republic of the Philippines filed with the Gumaca court an amended petition to annul the decision, decree and title on the ground that they are void because the land in question was still a part of the unclassified public forest. Moreover, the possessory information title relied upon by the Maxino spouses covered only 29 hectares of land and not 885 hectares. The petition was verified by the Acting Director of Forestry.The Maxinos opposed the petition but it was denied by the Appellate Court, hence this petition. ISSUE: Whether or not the registration of the 885 hectares of land is valid.RULING:No. It is incontestable that Lot 1, the 885-hectare area registered by the Maxinos, is within the public forest, not alienable and disposable nor susceptible of private appropriation.Its inclusion in the public forest was certified by Director of Forestry Florencio Tamesis on July 6, 1940, as per Land Classification Map No. 1386, Tayabas Project No. 16-E of Mulanay, and as shown in the report and testimony of Lorenzo R. Tria, a forest station warde. Tria recommended that the title of the Maxino spouses be annulled.It is axiomatic that public forestal land is not registerable. Its inclusion in a title, whether the title be issued during the Spanish regime or under the Torrens system, nullifies the title. Possession of public forestal lands, however long, cannot ripen into private ownership. Thus, the registration of the 885 hectares is not valid.

Case No. 24: Republic v. Reyes, 155 SCRA 313: GR No. L-30263-5

Facts: Godofredo R. Eusebio and Urbano C. Lara filed their Free Patent Applications for their parcels of land situated in Napindan, Taguig, Rizal at the Bureau of Lands. The applications were approved and registered at the Register of Deeds of Sale. In an investigation conducted by the Anti-Graft and Corrupt Board of the Bureau of Lands it was discovered that the parcels of land patented were actually under water and form part of the Laguna de Bay and neither of the respondents were able to occupy or possess said lots. Respondents admitted their non-compliance with the requirements and agreed to have their patents be cancelled. The petitioner, represented by the Director of Lands, filed complaints at the Court of First Instance against Eusebio and Lara. However, due to the respondents’ failure to answer the complaint, CFI of Rizal rendered decisions where the respondents’ patent numbers were declared null and void and ordering the Register of Deeds to cancel the said patent titles. After the lapse of almost 5 years, respondents filed for the annulment of the decision of the CFI of Rizal alleging as grounds that the Court had no jurisdiction over their persons and that the decision was procured through fraud. Director of Lands had not been properly served with summons and thus, CFI of Rizal allowed the respondents, now plaintiffs, to adduce their evidence before the Special Clerk of Court. However, CFI of Rizal again rendered a decision declaring the patent numbers null and void and ordered the Director of Lands to reinstate the free patent numbers issued in the names of the respondent. Notwithstanding the fact that their patents and certificates were declared null and void, respondents executed separate deeds of absolute sale involving the alleged lands.

Issue: Whether the land patented is entitled to an absolute deed of sale.

Held: No. That it is well settled that any title issued on non-disposable lots even in the hands of an alleged innocent purchaser for value, shall be cancelled. In the case at bar, the free patents and certificates of title issued to Eusebio and Lara cover areas which form parts of Laguna de Bay. These are neither agricultural nor disposable. Subject patents and titles were erroneously issued due to misrepresentations and false reports and must therefore be cancelled. Any false statement in an application for public land shall ipso facto produce the cancellation of the title granted. This rule applies even after the issuance of the certificate of title. A certificate of title cannot be used as a shield to perpetuate fraud, and the doctrine of indefeasibility of torrens title does not apply to free patent secured through fraud. Likewise, the Court ruled that mere possession of land does not itself divest the land of its public character. Void free patents and certificates of title do not divest the state of its ownership of the land nor operate to change the public character of the land to private. It is apparent that the law on innocent purchasers for value does not apply insofar as non-disposable public lands are concerned.

27.) G.R. No. L-37682 March 29, 1974REPUBLIC OF THE PHILIPPINES, Represented by the DIRECTOR OF LANDS, petitioner, vs.HON. PEDRO SAMSON ANIMAS, in his capacity as Judge of CFI South Cotabato,

Branch I, General Santos City, ISAGANI DU TIMBOL and the REGISTER OF DEEDS OF GENERAL SANTOS CITY, respondent.Facts: The Republic assailed the decision of the Court of First instance through the respondent judge which granted the conveyance of a land to Isagani Timbul. The Court of First Instance, through respondent judge, averred that the State’s right to assail had already prescribed because over one year had already lapsed since the title transfer pursuant to Section 38 of the Land Registration Act, and that the free patent issued therefor is indefeasible. The petitioner argued that the land is a forest land; hence, it is of public dominion and can never be appropriated.Issue: Whether the land, even though issued as a free patent, may be alienated and disposed.Ruling:Yes. The land in question is a forest land, thus it is part of the public dominion and it may not be disposed and alienated. The title granted to the private respondent by virtue of the land registration proceeding is void. It is the Bureau of Forestry which has been vested the sole prerogative over the demarcation, administration, protection and occupancy of the country’s forestry. Moreover, since it is the State which assails the title, prescription may not be invoked against the same: The Statute of Limitations does not lie against the State.

Case 7

a. Properties of public dominion

REPUBLIC V. CA131 SCRA 532

Facts:

Respondents Benjamin Tancinco, Azucena Tancinco Reyes, Marina (should be "Maria") Tancinco Imperial and Mario C. Tancinco are registered owners of a parcel of land covered by Transfer Certificate of Title No. T-89709 situated at Barrio Ubihan, Meycauayan, Bulacan bordering on the Meycauayan and Bocaue rivers.

They filed an application for the registration of three lots adjacent to their fishpond property but the Bureau of Lands filed a written opposition to the application for registration.

The private respondents filed a partial withdrawal of the application for registration with respect to Lot 3 in line with the recommendation of the Commissioner appointed by the Court, hence it was ordered withdrawn from the application. and trial proceeded only with respect to Lots 1 and 2 covered by Plan Psu-131892.

On June 26, 1976, the lower court rendered a decision granting the application on the finding that the lands in question are accretions to the private respondents' fishponds covered by Transfer Certificate of Title No. 89709 however, the petitioner Republic appealed to the respondent Court of Appeals.

On August, 19, 1982, the respondent Court rendered a decision affirming in toto the decision of the lower court.

There are facts and circumstances in the record which render untenable the findings of the trial court and the Court of Appeals that the lands in question are accretions to the private respondents' fishponds.

Issue:

Whether the registration of the lots valid.

Ruling:

No, the registration of the lots is not valid.

The lower court cannot validly order the registration of Lots 1 & 2 in the names of the private respondents. These lots were portions of the bed of the Meycauayan river and are therefore classified as property of the public domain under Article 420 paragraph 1 and Article 502, paragraph 1 of the Civil Code of the Philippines. They are not open to registration under the Land Registration Act. The adjudication of the lands in question as private property in the names of the private respondents is null and void. The only valid conclusion therefore is that the said areas could not have been there in 1939. They existed only after the private respondents transferred their dikes towards the bed of the Meycauayan river in 1951. What private respondents claim as accretion is really an encroachment of a portion of the Meycauayan river by reclamation.

# 29[G.R. No. L-27873. November 29, 1983.]HEIRS OF JOSE AMUNATEGUI, Petitioners, v. DIRECTOR OF FORESTRY, Respondent.

Facts:

There were two petitions for review on certiorari questioning the decision of the Court of Appeals which declared the disputed property as forest land, not subject to titling in favor of private persons, Borre and Amunategui.

The Director of Forestry, through the Provincial Fiscal of Capiz, also filed an opposition to the application for registration of title claiming that the land was mangrove swamp which was still classified as forest land and part of the public domain.

Another oppositor, Emeterio Bereber filed his opposition insofar as a portion of Lot No. 885 containing 117,956 square meters was concerned and prayed that title to said portion be confirmed and registered in his name.During the progress of the trial, applicant-petitioner Roque Borre sold whatever rights and interests he may have on Lot No. 885 to Angel Alpasan. The latter also filed an opposition, claiming that he is entitled to have said lot registered in his name.

Issue:

WON the lot in question can be subject of registration and confirmation of title in the name of the private person.

Held:

NO. The opposition of the Director of Forestry was strengthened by the appellate court's finding that timber licenses had to be issued to certain licensees and even Jose Amunategui himself took the trouble to ask for a license to cut timber within the area. It was only sometime in 1950 that the property was converted into fishpond but only after a previous warning from the District Forester that the same could not be done because it was classified as "public forest”.

A forested area classified as forest land of the public domain does not lose such classification simply because loggers or settlers may have stripped it of its forest cover. "Forest lands" do not have to be on mountains or in out of the way places. Swampy areas covered by mangrove trees, nipa palms, and other trees growing in brackish or sea water may also be classified as forest land. The classification is descriptive of its legal nature or status and does not have to be descriptive of what the land actually looks like. Unless and until the land classified as "forest" is released in an official proclamation to that effect so that it may form part of the disposable agricultural lands of the public domain, the rules on confirmation of imperfect title do not apply.The possession of forest lands, no matter how long, cannot ripen into private ownership. Therefore, the lot in question never ceased to be classified as forest land of public domain.

5) REPUBLIC V. CA131 SCRA 532

Facts:

Respondents Benjamin Tancinco, Azucena Tancinco Reyes, Marina (should be "Maria") Tancinco Imperial and Mario C. Tancinco are registered owners of a parcel of land covered by Transfer Certificate of Title No. T-89709 situated at Barrio Ubihan, Meycauayan, Bulacan bordering on the Meycauayan and Bocaue rivers.

They filed an application for the registration of three lots adjacent to their fishpond property but the Bureau of Lands filed a written opposition to the application for registration.

The private respondents filed a partial withdrawal of the application for registration with respect to Lot 3 in line with the recommendation of the Commissioner appointed by the Court, hence it was ordered withdrawn from the application. and trial proceeded only with respect to Lots 1 and 2 covered by Plan Psu-131892.

On June 26, 1976, the lower court rendered a decision granting the application on the finding that the lands in question are accretions to the private respondents' fishponds covered by Transfer Certificate of Title No. 89709 however, the petitioner Republic appealed to the respondent Court of Appeals.

On August, 19, 1982, the respondent Court rendered a decision affirming in toto the decision of the lower court.

There are facts and circumstances in the record which render untenable the findings of the trial court and the Court of Appeals that the lands in question are accretions to the private respondents' fishponds.

Issue:

Whether the registration of the lots valid.

Ruling:

No, the registration of the lots is not valid.

The lower court cannot validly order the registration of Lots 1 & 2 in the names of the private respondents. These lots were portions of the bed of the Meycauayan river and are therefore classified as property of the public domain under Article 420 paragraph 1 and Article 502, paragraph 1 of the Civil Code of the Philippines. They are not open to registration under the Land Registration Act. The adjudication of the lands in question as private property in the names of the private respondents is null and void. The only valid conclusion therefore is that the said areas could not have been there in 1939. They existed only after the private respondents transferred their dikes towards the bed of the Meycauayan river in 1951. What private respondents claim as accretion is really an encroachment of a portion of the Meycauayan river by reclamation.

Case No. 24: Republic v. Sangalang, 159 SCRA 515: GR No. L-58822

Facts: The subject property was inherited by Kiangs from their father, old man Kiang who in turn inherited the land in possession of his parents since Spanish

times. Old man Kiang filed an application for registration of the land but it was dismissed by the CFI of Banquet, Mt Province due to Civil Reservation Case No. L- declaring as public lands all lands within the limits of the Baguio Townsite Reservation, with the exception of lands reserved for specific public purposes and those claimed and adjudicated private property. About 31 years later, Kiangs filed an application for registration of the parcels of land in question. The respondent court presided by Judge Pio R. Marcos rendered a decision adjudicating the land in favor of the respondents. The republic, represented by the Solicitor General, filed a complaint for the annulment of the decision of Judge Marcos.

Issue: Whether the Kiangs are entitled for the registration of the land.

Held: No. The exception provided for in the Section 79 of Commonwealth Act No. 141 refer to lands "claimed by or belonging to private parties." This exception cannot possibly apply to the respondents Kiangs since the land which was the subject can no longer be considered land "claimed by or belonging to private parties." It was held that the CFI of Baguio and Banquet had no jurisdiction to reopen the case on the ground that it did not par-take of the nature of cadastral proceedings as contemplated in Republic Act No. 931, as amended, and that lands within Government reservations cannot be registered in favor of private individuals. Recognizing that before the promulgation of said decision , large portions of the public land within the Baguio Towns; Reservation had been illegally decreed in favor of private individuals, PD 1271 was issued intended to protect title holders who, before the promulgation of the Supreme Court decision on July 31, 1973, had acted in good faith and relied, although mistakenly, on the indefeasibility of torrens certificates of titles anal had introduced substantial improvements on the land covered by said certificates. The proviso in favor of bona-fide holders of titles issued on or before July 31, 1973 does not apply to the case at bar, since the certificate of title of the respondents Kiangs was issued only on June 5, 1975. Neither is there any showing that respondents complied with the requirements of Section 1 of PD No. 1271 for the validation of their title.

8. RAMIREZ VS C.A.144 SCRA 292Facts:On September 15,1959, petitioner- spouses Hilario Ramirez and Valentina Bonifacio filed an application for registration of a parcel of Riceland in Pamplona, Las Pinas Rizal. The petitioners presented parol evidence that they acquired the land in question by purchase from Gregorio Pascual during the early part of the American regime but the corresponding contract of sale was lost and no copy or record of the same was available. Thereafter, the court ordered the issuance of OCT No. 2273 in the petitioners’ names. On March 30, 1960, the private respondents filed a petition to review the decree of registration on the ground of fraud. They alleged, among others, that in 1938 respondents obtained a loan of P400.00 from the petitioners which they secured with a mortgage on the land in question by way of antichresis and that for this reason, Tax Declaration No. 8777 was cancelled and substituted by

Tax Declaration Nos. 9522 and 2385 issued in the names of the petitioners. In their answer, the spouses Ramirez denied the material allegations of the petition, they based their claim to the land on two deeds of sale allegedly executed on April 15, 1937 and April 23, 1937 which they allegedly found accidentally in March 1960. After trial, the court found that the deeds of sale were spurious, and that the respondents took possession of the land as owners after the death of Agapita Bonifacio and in 1938, mortgaged it to the spouses Ramirez to secure the payment of a loan in the amount of P400.00. It was agreed that the respondents could not redeem the property within a period of five years and that the petitioners would take possession of the land, enjoy its fruits, and pay the land taxes thereon. Finding the claims of the herein respondents sustained by the evidence, it ordered the cancellation of Original Certificate of Title No. 2273 of the Register of Deeds of Rizal in the names of herein petitioners and the issuance in lieu thereof of another original certificate of title in the names of herein respondents.Issue:Whether or not an antichretic creditor can acquire by prescription the land surrendered to him by the debtor.

Ruling:NO. The Court ruled that the issue was submitted to the appellate court and was correctly resolved therein. The Court of Appeals stated:...The petition alleged that 'the applicants Hilario Ramirez and Valentina Bonifacio willfully and fraudulently suppressed the facts that the petitioners are the legal and rightful owners of the rice field in question and that they possess the said rice field merely as antichretic creditors as security for the loan of P400.00; that the applicants are guilty of fraudulent misrepresentation and concealment when they declared in their application, in the case at bar, that no other person had any claim or interest in the said land.' These we believe are sufficient allegations of extrinsic fraud.In the applicant's application for registration, which followed the form required by the Land Registration Act, the applicants alleged that 'to the best of our knowledge and belief, there is no mortgage or incumbrance of any kind whatsoever affecting said land, nor any other person having any estate or interest therein, legal or equitable, in possession, remainder, reversion or expectancy.' This allegation is false and made in bad faith, for, as We have found, the applicants are not the owners of the land sought to be registered and they are in possession thereof only as antichretic creditors.

Case No. 24: De Los Angeles v. Santos, 12 SCRA 622: GR No. L-19615

Facts: Leonor de los Angeles and seven co-applicants filed an application for registration of title to 12 parcels of land in Ampid, San Mateo, Rizal. It is alleged that the applicants are owners pro-indiviso and in fee simple of the land. The Director of Lands filed an opposition after the initial hearing stating that the land is a portion of the public domain. The Province of Rizal also interposed asserting the required 3.00 meters strips of public easement on lots along Ampid river and creek. Julio Hidalgo also filed an opposition claiming that they are the lawful owners of the parcels of

land in question for having acquired homestead patents over said lots.

Issue: Whether the court erred in dismissing the application for registration, over which a homestead patent was issued during the pendency of the registration proceeding.

Held: Yes. If applicants were to successfully prove the averment and show their alleged registrable title to the land, it could only result in the finding that when Julio Hidalgo’s homestead patent was issued over on the said lot was no longer public. The land registration court would have to order a decree of title issued in applicants' favor and declare the aforesaid homestead patent a nullity which vested no title in the patentee as against the real owners. Since the existence or non-existence of applicants' registrable title is decisive of the validity or nullity of the homestead patent issued as aforestated on said lot the court a quo's jurisdiction in the land registration proceedings could not have been divested by the homestead patent's issuance. A homestead patent, therefore, does not finally dispose of the public or private character of the land as far as courts upon proceedings in rem are concerned. Applicants should thus be given opportunity to prove registrable title to Lot 11.

8. OMICO MINING AND INDUSTRIAL CORPORATION and FREDERICK G. WEBBER vs. JUDGE AMADOR T. VALLEJOS, in his capacity as Judge of the Court of First Instance of Cavite, ALFREDO CATOLICO, and LEONARDO ALCID, in his capacity as City Sheriff of Manila

[G.R. No. L-38974, March 25, 1975]

Facts:

On June 1, 1973, Alfredo Catolico, filed against Omico Mining and Industrial Corporation and Frederick G. Webber, the latter in his personal capacity and as President and Chairman of the Board of Directors of said corporation, alleging first, for the return of ten (10) certificates of stock of the corporation borrowed from him by the defendants, and the second, for the payment of his services as legal counsel for the corporation. Defendants filed a motion to dismiss the complaint on two grounds: namely (1) improper venue, in that the case was filed in Cavite where plaintiff is not a resident, the truth being that he is a resident of Quezon City where he has his permanent family home; and, as to the second cause of action, the contract of personal and professional services between plaintiff and defendants was entered into in the City of Manila, and, therefore, the case should have been filed in Manila in accordance with Section I of Rule 4 of the Revised Rules of Court; and (2) lack of cause of action, in that with regard to the stock certificates the same are in the name of Vicente Resonda; and, with respect to the contract of personal and professional services wherein it was agreed that the plaintiff shall head the legal department of defendant Omico Mining & Industrial Corporation.On June 16, 1973, the date set for the hearing of the motion to dismiss, neither the

parties nor their respective counsels appeared in court. While the motion to dismiss was pending resolution by the court because defendants had not yet presented to the court the required proof of service, plaintiff, on January 11, 1974, filed a petition to declare the defendants in default that defendants had been served with summons and copies of the complaint on June 8, 1973; that as of January 11, 1974, or after a lapse of seven (7) months from the service of summons, defendants had not filed their answer to the complaint. The court granted the petitionand, consequently, it received ex parte the evidence of the plaintiff and rendered judgment in favor of Catolico. Defendants filed a motion for reconsideration but Catolico file a motion to postpone hearing of motion for reconsideration.On May 31, 1974, while defendants' motion for reconsideration was still pending before the court because the defendants had not filed yet their reply to the opposition as they had not received a copy, Catolico filed a motion for immediate execution of judgment, alleging that said judgment had already become final and executory because the defendants failed to have the order of default lifted; that the motion for reconsideration was filed out of time; that there was a "manifest attempt on the part of the defendants to delay the proceedings to afford them an opportunity to have all their assets and shares dissipated by continuous sale of the same to the prejudice". Thence, the court denied the defendants’ motion for reconsideration. Defendants’ filed their notice of appeal. On July 22, Pio R. Marcos, as President and Chairman of the Board of Directors of defendant Omico Mining and Industrial Corporation, wrote a letter to respondent Sheriff asking that the defendants be given a little chance to exhaust the legal remedies available to hold in abeyance the execution and garnishment for the reasons that defendants were not given a chance to have their day in court in the motion for immediate execution of judgment and that they have already appealed from the lower court's decision and order of immediate execution.

Issue:

Whether respondent Judge acted without or in excess of jurisdiction or with grave abuse of discretion in declaring the defendants in default, in receiving plaintiff's evidence ex parte and in rendering judgment.

Held:

The Supreme Court ruled that the respondent Judge acted with grave abuse of discretion when he declared the petitioners in default. The motion to dismiss was pending before the court when such declaration was made, and it is generally irregular to enter an order of default while a motion to dismiss remains pending and undisposed of. The irregularity of the order of default is evident from the fact that when the petitioners were declared in default, their time for filing an answer had not yet commenced to run anew because on said date, their counsel had not yet

received any notice of the action taken by the court on their motion to dismiss. There may be cases where the attendance of certain circumstances "may be considered substantive enough to truncate the adverse literal application of the pertinent rules violated." Inasmuch as petitioners were declared in default while their motion to dismiss was still pending resolution, they were, therefore, incorrectly declared in default, and the holding of the trial of the case on the merits, in their absence, without due notice to them of the date of hearing, was a denial of due process. Consequently, the order of default, the judgment and the order of execution are patent nullities.